Юный математик дополнительные занятия в 5 классе

Раздел Математика
Класс 5 класс
Тип Рабочие программы
Автор
Дата
Формат doc
Изображения Есть
For-Teacher.ru - все для учителя
Поделитесь с коллегами:

РАБОЧАЯ ПРОГРАММА

МАТЕМАТИЧЕСКОГО КРУЖКА «Юный математик»

В 5 «а» КЛАССЕ

Пояснительная записка

Математика занимает особое место в образовании человека, что определяется безусловной практической значимостью математики, её возможностями в развитии и формировании мышления человека, её вкладом в создание представлений о научных методах познания действительности. Являясь частью общего образования, среди предметов, формирующих интеллект, математика находится на первом месте.

Первоначальные математические познания должны входить с самых ранних лет в наше образование и воспитание. Результаты надёжны лишь тогда, когда введение в область математических знаний совершается в лёгкой и приятной форме, на предметах обыденной и повседневной обстановки, подобранных с надлежащим остроумием и занимательностью.

Программа кружка рассчитана на учащихся 5 классов, склонных к занятиям математикой и желающих повысить свой математический уровень. Именно в этом возрасте формируются математические способности и устойчивый интерес к математике.

Данная программа является частью интеллектуально-познавательного направления дополнительного образования и расширяет содержание программ общего образования.

Цель программы- способствовать воспитанию интереса учащихся к математике и формированию когнитивных умений в процессе занятий математического кружка способностей.

Образовательные задачи:

- углубление и расширение знаний учащихся по математике;

- привитие интереса учащимся к математике;

- активизировать познавательную деятельность;

- показать универсальность математики и её место среди других наук.

Воспитательные задачи:

- воспитание культуры личности;

- воспитание отношения к математике как к части общечеловеческой культуры;

- воспитание понимания значимости математики для научно - технического прогресса;

- воспитание настойчивости, инициативы, чувства ответственности, самодисциплины.

Развивающие задачи:

- развитие ясности и точности мысли, критичность мышления, интуиции, логического мышления, элементов алгоритмической культуры, пространственных представлений, способности к преодолению трудностей;

- формирование математического кругозора, исследовательских умений учащихся.

Программа содержит материал, как занимательного характера, так и дополняющий, расширяющий программу общеобразовательной школы по математике. Большое внимание в программе уделяется исто­рии математики и рассказам, связанным с математикой (запись цифр и чисел у других народов, математические фокусы, ребусы и др.), выполнению самостоятельных заданий творческого характера (составить рассказ, фокус, ребус, задачу с использованием изученных матема­тических свойств), изучению раз­личных арифметических методов решения задач (метод ре­шения «с конца» и др.), выполнению проектных работ. Уделяется внимание рассмотрению геометрического ма­териала, развитию пространственного воображения.

Программа кружка рассчитана на один год обучения.

Итогом реализации программы являются: успешные выступления кружковцев на олимпиадах всех уровней, математических конкурсах, а также создание брошюры «Математическая шкатулка»(банк нестандартных задач для учащихся 5 классов), где будут собраны задачи по темам всего курса, которые составлены учащимися или взяты из каких-либо источников (книги, журналы, интернет) и их решения, проектные работы учащихся.








  1. Учебно-тематический план

Раздел

Тема

Кол-во занятий

1

Занимательная арифметика

Тема1.Запись цифр и чисел у других народов

1

Тема 2.Числа - великаны и числа- малютки

1

Тема3. Приёмы быстрого счёта на пальцах

1

Тема4. Приёмы быстрого счёта при сложении

1

Тема5. Приёмы быстрого счёта при умножении

1

Тема6. Вычисление примеров используя приёмы быстрого счёта.

1

Тема7. Решение задач

1

Тема8. Решение олимпиадных задач

1

Тема 9. Подготовка докладов о математиках

1

Тема10. Чтение докладов о математиках на неделе МО

1

2

Занимательные задачи

Тема 1. Магические квадраты

1

Тема 2. Составление магических квадратов

1

Тема 3.Математические фокусы

1

Тема 4.Демонстрация математических фокусов

1

Тема 5.Математические ребусы

1

Тема 6.Разгадывание математических ребусов

1

Тема 7. Софизмы

1

Тема 8.Задачи с числами

1

Тема 9.Задачи шутки

1

Тема 10.Старинные задачи

1

Тема 11.Решение старинных задач

1

Тема 12.Проведение мини-олимпиады по решению задач

1

Тема 13.Участие в дистанционных олимпиадах

1

3

Логические задачи

Тема 1.Задачи, решаемые с конца

1

Тема 2.Круги Эйлера

1

Тема 3.Решение задач с применением кругов Эйлера

1

Тема 4.Простейшие графы

1

Тема 5.Решение задач с применением графов

1

Тема 6.Задачи на переливания

1

Тема 7.Решение задач на переливания

1

Тема 8.Задачи на взвешивания

1

Тема 9.Решение задач на взвешивания

1

Тема 10.Задачи на движение

1

Тема 11.Решение задач на движение

1

Тема 12.Проведение мини-олимпиады по решению задач

1

Тема 13. Решение дистанционных олимпиад

1

4

Геометрические задачи

Тема 1.Задачи на разрезание

1

Тема 2.Задачи со спичками

1

Тема 3.Решение задач со спичками

1

Тема 4. Геометрические головоломки

1

Тема 5. Решение геометрических головоломок.

1

Тема 6. Решение дистанционных олимпиад

1

5

Комбинаторика

Тема 1. Комбинаторные задачи

1

Тема 2. Факториал.

1

Тема 3. Решение уравнений с факториалами.

1

Тема 4. Перестановки элементов.

1

Тема 5. Решение задач на перестановки.

1

Тема 6. Размещение элементов.

1

Тема 7. Решение задач на размещение.

1

Тема 8. Сочетание элементов

1

Тема 9. Решение задач на сочетания

1

Тема 10. Решение примеров с факториалами.

1

Тема 11. Решение задач по комбинаторике.

1

Тема 12. Принцип Дирихле

1

Тема 13. Решение задач на принцип Дирихле

1

6

Цифровые и алгебраические задачи

Тема 1. Решение цифровых занимательных задач

1

Тема 2. Числовые закономерности

1

Тема 3. Решение задач на определение закономерностей

1

Тема 4. Решение задач на определение закономерностей

1

Тема 5. Решение задач с помощью таблиц

1

Тема 6. Решение задач с помощью таблиц

1

Тема 7. Арифметика Магницкого

1

Тема 8. Решение задач из арифметики Магницкого

1

Тема 9. Решение олимпиадных задач

1

7

Мир процентов и среднего арифметического

Тема 1 Проценты и доли

1

Тема 2 Решение задач на проценты

1

Тема 3 Решение задач на проценты

1

Тема 4 Среднее арифметическое

1

Тема 5 Решение задач на среднее арифметическое

1

Тема 6 Решение олимпиадных задач на проценты и среднее арифметическое

1

8

Проекты

Тема 1.Проектные работы. Цель, форма защиты.

1

Тема 2 Защита проектов по теме «Старинные меры длины»

1

Тема 3 Защита проектов по теме «Старинные меры площади»

1

Тема 4 Защита проектов по теме «Системы счисления. Мифы, сказки, легенды.»

1

Тема 5 Защита проектов по теме «Системы счисления. Мифы, сказки, легенды.»


9

Решение задач по всему курсу

Тема 1. Решение задач

1

Тема 2. Решение олимпиадных задач

1

Тема 3. Итоговое занятие

1

ИТОГО:

78

Краткое содержание разделов

I. Занимательная арифметика

Тема 1.Запись цифр и чисел у других народов

Как люди научились считать. Старинные системы записи чисел. Цифры у разных народов. Римская нумерация.

Тема 2.Числа - великаны и числа- малютки

Открытие нуля. Мы живём в мире больших чисел. Числа-великаны. Названия больших чисел. Числа - малютки. Решение задач с большими и малыми числами.

Тема3. Упражнения на быстрый счёт

Некоторые приёмы быстрого счёта.

Умножение двухзначных чисел на 11,22,33, . . . , 99.

Умножение на число, оканчивающееся на 5.

Умножение и деление на 25,75,50,125.

Умножение и деление на 111,1111 и т.д.

Умножение двузначных чисел, у которых цифры десятков одинаковые, а сумма цифр единиц составляет 10. Умножение двузначных чисел, у которых сумма цифр равна 10, а цифры единиц одинаковые.

Умножение чисел, близких к 100.

Умножение на число, близкое к 1000.

Умножение на 101,1001 и т.д.

II. Занимательные задачи

Тема 1 . Магические квадраты.

Отгадывание и составление магических квадратов.

Тема 2.Математические фокусы.

Математические фокусы с «угадыванием чисел». Примеры математических фокусов.

Тема 3.Математические ребусы.

Решение заданий на восстановление записей вычислений.

Тема 4. Софизмы.

Понятие софизма. Примеры софизмов.

Тема 5.Задачи с числами

Запись числа с помощью знаков действий, скобок и определённым количеством одинаковых цифр.

Тема 6.Задачи - шутки

Решение шуточных задач в форме загадок.

III. Логические задачи

Тема 1.Задачи, решаемые с конца.

Решение сюжетных, текстовых задач методом «с конца».

Тема 2.Круги Эйлера.

Решение задач с использованием кругов Эйлера.

Тема 3.Простейшие графы

Понятие графа. Решение простейших задач на графы.

Тема 4.Задачи на переливания.

Решение текстовых задач на переливание.

Тема 5. Взвешивания.

Решение задач на определение фальшивых монет или предметов разного веса с помощью нескольких взвешиваний на чашечных весах без гирь.

Тема 6. Задачи на движение.

Решение текстовых задач на движение: на сближение, на удаление, движение в одном направлении, в противоположных направлениях, движение по реке.

Тема 7.Старинные задачи

Решение занимательных старинных задач и задач-сказок.

IV. Геометрические задачи

Тема 1.Задачи на разрезания.

Геометрия вокруг нас. Геометрия на клетчатой бумаге. Игра «Пентамино».

Тема 2. Задачи со спичками.

Решение занимательных задач со спичками.

Тема 3.Геометрические головоломки.

«Танграм».

V. Комбинаторика

Решение комбинаторных задач на перестановки, размещения и сочетания.

VI. Цифровые задачи. Алгебраические задачи.

Решение цифровых занимательных задач. Решение задач с помощью таблиц

VII. Мир процентов и среднего арифметического.

Решение задач на применение процентов и среднего арифметического.

VIII. Проекты

Тема 1.Выбор тем и выполнение проектных работ. Примерные темы проектов:

· Системы счисления. Мифы, сказки, легенды.

· Софизмы и парадоксы.

· Математические фокусы.

· Математика и искусство.

· Математика и музыка.

· Лабиринты.

· Палиндромы.

· Четыре действия математики.

· Древние меры длины.

· Возникновение чисел.

· Счёты.

· Старинные русские меры.

· Магические квадраты.

· Свои темы проектов.

Предполагаемые результаты обучения

В результате занятий в кружке учащиеся должны

Знать:

- старинные системы записи чисел, записи цифр и чисел у других народов;

- названия больших чисел;

- свойства чисел натурального ряда, арифметические действия над натуральными числами и нулём и их свойства, понятие квадрата и куба числа;

- приёмы быстрого счёта;

- методы решения логических задач;

- свойства простейших геометрических фигур на плоскости;

- понятие графа;

- понятие софизма.

Уметь:

- читать и записывать римские числа;

- читать и записывать большие числа;

- пользоваться приёмами быстрого счёта;

- решать текстовые задачи на движение, на взвешивание, на переливание;

- использовать различные приёмы при решении логических задач;

- решать геометрические задачи на разрезание, задачи со спичками, геометрические головоломки, простейшие задачи на графы;

- решать математические ребусы, софизмы, показывать математические фокусы;

- решать задачи по комбинаторике;

- выполнять проектные работы.

Для реализации программы кружка необходимо:

Материально-техническое обеспечение

Методическое и дидактическое обеспечение

Учебный кабинет, учебные столы, стулья, компьютеры, принтер, сканер, проектор, классная доска, мел.

-Подборка информационной и справочной литературы;

-Обучающие и справочные электронные издания;

- Доступ в Интернет

Литература

1. Гусев В.А., Орлов А.И., Розенталь А.Л. Внеклассная работа с учениками 5-6 классов. - М.: Просвещение,2005 .

2. Журналы «Математика в школе», 1980-2008.

3.А.С.Чесноков, С.И. Шварцбурд, В.Д.Головина, И.И. Крючкова, Л. А. Литвачук. Внеклассная работа по математике в 4-5 классах. М. , «Просвещение»,1974.

4. Фарков А.В. Математические кружки в школе. 5-8 классы- М. Айрис-пресс, 2006

5.Фарков А.В. Математические олимпиады в школе. 5-11 классы. М.: Айрис-пресс, 2002.

6.Фарков А.В. Внеклассная работа по математике.5-11 классы М.: Айрис-пресс, 2008

7. Ю.В.Щербакова. Занимательная математика на уроках и внеклассных мероприятиях. 5-8 классы. М.: Глобус.2008.

8.П.М. Камаев. Устный счёт. М.: Чистые пруды, 2007.(Библиотека « Первого сентября», серия « Математика», №3 (15)/2007)

9.Н.П. Кострикина. Задачи повышенной трудности в курсе математики 4-5 классов. Книга для учителя.- М.: Просвещение, 1986







2.2 Содержание занятий математического кружка в 5-ом классе
Занятие №1. «Цифры у разных народов».
Рассказать об арабской и римской нумерации. В ходе беседы решить задачи:
1. Записать арабскими цифрами: ХХV, CХIV, XCII, MMDLXXI.
2. Записать римскими цифрами: 37, 92, 2164, 3068, 4527, 183693.
3. В данных неверных равенствах переложить по одной «спичке», чтобы все равенства стали верными:
а)VI- IV=IX; б) VI-IV=XI; в) VI+IV=XII; г) X+X=I; д) X-IX=VI; е) VIII+IV=XVII; ж) IV-I+V=II; з) X=VII-III.
4. Фокус: (отгадывание задуманного числа)
Задумайте любое число, умножьте его на 2, прибавьте 1, полученный результат увеличьте в5 раз, вычтите 4, умножьте на 2. Что у вас получилось? (Если от названного числа отнять 2, а затем полученное число разделить на 20, то получим задуманное число.)
5.На одной из старых улиц Москвы стоят два дома, на фасаде которых обозначена дата их постройки: а)MDCCCCV б) MDCCCLXXXXIX
В каком году построен каждый дом?
Домашнее задание:
v Запишите арабскими цифрами числа: XXXIV; XXIX; CDXXI; CMIII.
v Запишите римскими цифрами числа: 49; 574; 1147; 1974; 5003.
v Этот греческий храм построен из 11 спичек. Требуется переложить 4 спички так, чтобы получилось 15 квадратов.
Занятие №2 -3. Числа великаны. + презентация

Для сокращения записи больших чисел давно используется система величин, в которой каждая из последующих в тысячу раз больше предыдущей:

1000 единиц - просто тысяча

1000 тысяч - 1 миллион

1000 миллионов - 1 биллион( или миллиард)

1000 биллионов - 1 триллион

1000 триллионов - 1 квадриллион

1000 квадриллионов- 1 квинтиллион

1000 квинтиллионов - 1секстиллион

1000 секстиллионов - 1 септиллион

1000 септиллионов - 1октиллион

1000 октиллионов - 1 нониллион

1000 нониллионов - 1 дециллион

1000 дециллионов - ундециллион.


Миллиард

Слово "миллиард" употребляется у нас в смысле тысячи миллионов и при денежных вычислениях и в точных науках. Но, например, в Германии и в Америке под миллиардом иногда имеют ввиду не тысячу, а всего сто миллионов. Этим, между прочим, можно объяснить то, что слово "миллиардер" было в ходу за океаном еще тогда, когда ни один из тамошних богачей не имел состояния в тысячу миллионов. Огромное состояние Рокфеллера незадолго до войны исчислялось "всего" 900 миллионов долларов, а остальных "миллиардеров"-меньшими числами. Только во время войны появились в Америке миллиардеры в нашем смысле слова (их иногда называют на родине "биллионерами").

Чтобы составить себе представление об огромности миллиарда, представьте себе, что в книжке в 200 страниц не более 200.000 букв. В пяти таких книжках окажется один миллион букв. А миллиард букв будет заключать в себе стопка из 5.000 экземпляров такой книжки. Стопка, которая, будучи аккуратно сложена, составила бы столб высотой с Исаакиевский собор. Миллиард секунд часы отобьют более чем в 30 лет (точнее в 31,7 лет). А миллиард минут составляет более 19 столетий; человечество всего 29 апреля 1902 года в 10 часов 40 минут начало считать второй миллиард минут от первого дня нашего летосчисления.

Биллион и Триллион

Ощутить огромность этих числовых исполинов трудно даже человеку, опытному в обращении с миллионами. Великан - миллион такой же карлик рядом со сверхвеликаном биллионом, как единица рядом с миллионом. Об этом взаимоотношении мы забываем и не делаем в своем воображении большой разницы между миллионом, биллионом и триллионом. Волос, увеличенный по толщине в биллион раз, был бы раз в 8 шире земного шара, а муха при таком увеличении была бы в 70 раз толще Солнца!

Взаимоотношение между миллионом, биллионом и триллионом можно с некоторою наглядностью представить следующим образом. В Санкт - Петербурге еще недавно было миллион жителей. Представьте себе длинный прямой ряд городов таких как Санкт - Петербург, целый миллион их: в этой цепи столиц, тянущихся на семь миллионов километров (в 20 раз дальше Луны) будет насчитываться биллион жителей... Теперь вообразите, что перед вами не один такой ряд городов, а целый миллион рядов, т.е. квадрат, каждая сторона которого состоит из миллиона Санкт - Петербургов и, который внутри сплошь уставлен такими городами: в этом квадрате будет триллион жителей.

Одним триллионом кирпичей можно было бы, размещая их плотным слоем по твердой поверхности земного шара, покрыть все материки равномерным сплошным пластом высотою с четырехэтажный дом (16 м).

Если бы все видимые в сильнейшие телескопы звезды обоих небесных полушарий, т. е. не менее 500 миллионов звезд были обитаемы и населены каждая, как наша Земля, то на всех этих звездах, вместе взятых, насчитывался бы только один триллион людей.

Молекула по ширине меньше точки типографского шрифта примерно в миллион раз. Вообразите триллион таких молекул, нанизанных вплотную на одну нитку. Какой длины была бы эта нить? Ею можно было бы семь раз обмотать земной шар по экватору. Световой год - путь, проходимый лучом света в 1 год (свет пробегает в секунду 300000 км); он равен, примерно, 9.5 биллионам км.

Числовые великаны вокруг и внутри нас

Часто можно встретиться с числовыми великанами. Они присутствуют всюду вокруг и даже внутри нас самих - надо лишь уметь рассмотреть их. Небо над головой, песок под ногами, воздух вокруг нас, кровь в нашем теле - все скрывает в себе невидимых великанов из мира чисел.

Числовые исполины небесных пространств для большинства людей не являются неожиданными.

Хорошо известно, что зайдет ли речь о числе звезд вселенной, об их расстояниях от нас и между собою, об их размерах, весе, возрасте - во всех случаях мы неизменно встречаемся с числами, подавляющими воображение своей огромностью. Недаром выражение «астрономическое число» сделалось крылатым. Многие, однако, не знают, что даже и те небесные тела, которые астрономы часто называют «маленькими», оказываются настоящими великанами, если применить к ним привычную земную мерку. Существуют в нашей солнечной системе планеты, которые, ввиду их незначительных размеров, получили у астрономов наименование «малых». Среди них имеются и такие, поперечник которых равен нескольким километрам. В глазах астронома, привыкшего к исполинским масштабам, они так малы, что, говоря о них, он пренебрежительно называет их «крошечными». Но они представляют собой «крошечные» тела только рядом с другими небесными светилами, еще более огромными: на обычную же человеческую мерку они далеко не миниатюрны. Возьмем такую «крошечную» планету с диаметром 3 км. По правилам геометрии легко рассчитать, что поверхность такого тела заключает 28 кв. км, или 28 000 000 кв. м. На 1 кв. м может поместиться стоя человек 7. Значит, на 28 миллионах кв. м найдется место для 196 миллионов человек.

Песок под нашими ногами также вводит нас в мир числовых исполинов. Недаром сложилось издавна выражение: «бесчисленны, как песок морской». Древние недооценивали многочисленность песка, считая ее одинаковой с многочисленностью звезд. В старину не было телескопов, а простым глазом мы видим на небе всего около 3500 звезд (в одном полушарии). Песок на морском берегу в миллионы раз многочисленнее, чем звезды, доступные невооруженному зрению.

Величайший числовой гигант скрывается в том воздухе, которым мы дышим. Каждый кубический сантиметр воздуха, каждый наперсток заключает в себе 27 квинтиллионов (т. е. 27 с 18 нулями) мельчайших частиц, называемых «молекулами».

Невозможно даже представить себе, как велико это число. Если бы на свете было столько людей, для них буквально недостало бы места на нашей планете. В самом деле: поверхность земного шара, считая все его материки и океаны,- равна 500 миллионам кв. км. Раздробив в квадратные метры, получим 500 000 000 000 000 кв.м.

Поделим 27 квинтиллионов на это число, и мы получим 54 000. Это означает, что на каждый квадратный метр земной поверхности приходилось бы более 50 тысяч человек!

Числовые великаны скрываются и внутри человеческого тела. Покажем это на примере нашей крови. Если каплю ее рассмотреть под микроскопом, то окажется, что в ней плавает огромное множество чрезвычайно мелких телец красного цвета, которые и придают крови ее окраску. Каждое такое «красное кровяное тельце» имеет форму крошечной круглой подушечки, посредине вдавленной. Все они у человека примерно одинаковых размеров и имеют в поперечнике около 0,007 мм, а толщину - 0,002 мм. Зато число их огромно. В крошечной капельке крови, объемом 1 куб. мм, их заключается

5 миллионов. Сколько же их всего в нашем теле? В теле человека примерно в 14 раз меньше литров крови, чем килограммов в его весе. Если вы весите 40 кг, то крови в вашем теле около 3 литров, пли

3 000 000 куб. мм. Так как каждый куб. мм заключает 5 миллионов красных телец, то общее число их в вашей крови:

5 000 000 x 3 000 000=15 000 000 000 000.

15 триллионов кровяных телец!

Какую длину займет эта армия кружочков, если выложить ее в ряд один к другому? Нетрудно рассчитать, что длина такого ряда была бы 105 000 км. Более чем на сто тысяч километров растя¬нулась бы нить из красных телец вашей крови. Ею можно было бы обмотать земной шар по экватору;

100 000 : 40 000=2,5 раза, а нитью из кровяных шариков взрослого человека три раза.

Объясним, какое значение для нашего организма имеет такое измельчение кровяных телец. Назначение этих телец разносить кислород по всему телу. Они захватывают кислород, когда кровь проходит через легкие, и вновь выделяют его, когда кровяной поток заносит их в ткани нашего тела, в его самые удаленные от легких уголки. Сильное измельчение этих телец способствует выполнению ими этого назначения, потому что чем они мельче, при огромной численности, тем больше их поверхность, а кровяное тельце может поглощать и выделять кислород только со своей поверхности. Расчет показывает, что общая поверхность их во много раз превосходит поверхность человеческого тела и равна 1200 кв. м. Такую площадь имеет большой огород в 40 м длины и 30 м ширины. Теперь вы понимаете, до какой степени важно для жизни организма то, что кровяные тельца сильно раздроблены и так многочисленны: они могут захватывать и выделять кислород на поверхности, которая в тысячу раз больше поверхности нашего тела.

Сколько пищи поглощает человек за свою жизнь

Числовым великаном следует назвать и тот внушительный итог, который получился бы, если бы вы подсчитали, сколько всякого рода пищи поглощает человек за 70 лет средней жизни. Целый железнодорожный поезд понадобился бы для перевозки тех тонн воды, хлеба, мяса, дичи, рыбы, картофеля и других овощей, тысяч яиц, тысяч литров молока и т. д., которые человек успевает поглотить в течение своей жизни. Наглядным примером служит случай, описанный Джонатаном Свифтом в книге «Приключение Гулливера в стране Лиллипутов». При виде его не веришь, что человек может справиться с таким исполином, буквально проглатывая правда, не разом груз длинного товарного поезда.

Быстрое размножение.

Спелая маковая головка полна крошечных зернышек; из каждого может вырасти целое растение. Сколько же получится маков, если зернышки до единого прорастут? Чтобы узнать это, надо сосчитать зернышки в целой головке. Скучное занятие, но результат так интересен, что стоит запастись терпением и довести счет до конца. Оказывается, одна головка мака содержит (круглым числом) 3000 зернышек.

Что отсюда следует? То, что будь вокруг нашего макового растения достаточная площадь подходящей земли, каждое упавшее зернышко дало бы росток, и будущим летом на этом месте выросло бы уже 3000 маков. Целое маковое поле от одной головки!

Посмотрим, что будет дальше. Каждое из 3000 растений принесет не менее одной головки (чаще же несколько), содержащей 3000 зерен. Проросши, семена каждой головки дадут 3000 новых растений, и, следовательно, на второй год у нас будет уже не менее 3000 х 3000 = 9 000 000 растений.

Легко рассчитать, что на третий год число потомков нашего единственного мака будет уже достигать 9 000 000 х 3000 = 27000 000 000.

На пятом году макам станет тесно на земном шаре, потому что число растений сделается равным

81 000 000 000 000 х 3000 = 243 000 000 000 000 000.

Поверхность же всей суши, то есть всех материков и островов земного шара, составляет только

135 миллионов кв. км - 135 000 000 000 000 кв. м. - примерно в 200 раз менее, чем выросло бы экземпляров мака.

Видим, что, если бы все зернышки мака проростами, потомство одного растения могло бы уже в пять лет покрыть сплошь всю сушу земного шара густой зарослью по 2000 растений на каждом квадратном метре. Вот такой числовой великан скрывается в крошечном маковом зернышке! Сделав подобный же расчет не для мака, а для какого-нибудь другого растения, приносящего меньше семян, мы пришли бы к такому же результату, но только потомство его покрыло бы всю Землю не в пять лет, а в немного больший срок.

Возьмем хотя бы одуванчик, приносящий ежегодно около 100 семянок. Если бы все они прорастали, мы имели бы:

В " 1-й " год 1 растение

Во " 2-й " 100 растений

В " 3-й " 10 000

" 4-й " 1 000 000

" 5-й " 100 000 000

" 6-й " 10 000 000 000

" 7-й " 1 000 000 000 000

" 8-й " 100 000 000 000 000

" 9-й " 10 000 000 000 000 000

Это в 70 раз больше, чем имеется квадратных метров на всей суше.

Следовательно, на девятом году материки земного шара были бы покрыты одуванчиками, по 70 на каждом квадратном метре.

Почему же в действительности не наблюдаем мы такого чудовищно быстрого размножения? Потому что огромное большинство семян погибает, не давая ростков: они или не попадают на подходящую почву и вовсе не прорастают, или, начав прорастать, заглушаются другими растениями, или же, наконец, просто истребляются животными. Если бы этого массового уничтожения семян и ростков не было, каждое растение в короткое время покрыло бы сплошь всю нашу планету.

Это верно не только для растений, но и для животных. Не будь смерти, потомство одной пары любого животного рано или поздно заполнило бы всю Землю. Полчища саранчи, сплошь покрывающие огромные пространства, могут дать некоторое представление о том, что было бы, если бы смерть не препятствовала размножению живых существ. В каких-нибудь два-три десятка лет материки покрылись бы непроходимыми лесами и степями, где кишели бы миллионы животных, борющихся между собой за место. Океан наполнился бы рыбой до того густо, что сходство стало бы невозможно. А воздух сделался бы едва прозрачным от множества птиц и насекомых… в заключение приведем несколько подлинных случаев необыкновенно быстрого размножения животных, поставленных в благоприятные условия…

В Америке первоначально не было воробьев. Эта столь обычная у нас птица была ввезена в Соединенные Штаты намеренно с той целью, чтобы она уничтожала там вредных насекомых. Воробей, как известно, в изобилии поедает прожорливых гусениц и других насекомых, вредящим садам и огородам. Новая обстановка полюбилась воробьям; в Америке не оказалось хищников, истребляющих этих птиц, и воробей стал быстро размножаться. Количество вредных насекомых начало заметно уменьшаться, но вскоре воробьи так размножились, что за недостатком живой пищи принялись за растительную и стали опустошать посевы. Пришлось приступить к борьбе с воробьями; борьба эта обошлась американцам так дорого, что на будущее время издан был закон, запрещающий ввоз в Америку каких бы то ни было животных.

Гугол

Американский математик Кастнер изобрел «самое большое число» и назвал его «гугол». Это единица со ста нулями! То есть, 10 в 100. Хотя естественный ряд чисел и бесконечен, все же в известной мере гугол - это граница исчисляемого мира.

Дадим простор своему воображению и попытаемся проверить это утверждение. Вычислим площадь Земли в квадратных миллиметрах - можно надеяться, что получится головокружительная величина. Ничего подобного. Площадь земного шара равна квадратных миллиметров. Если же подсчитаем объем Земли в кубических миллиметрах, то получим чуть большее число - 10 в 30. Но и это слишком мало по сравнению с гуголом. Если предположить, что в одном кубическом миллиметре вместится десять песчинок, и подсчитать их количество в объеме Земли, то получится всего 1031. Иными словами, Земля слишком мала для какого бы то ни было вычисления в масштабах гугола.

Возьмем просторы космоса и попытаемся выразить расстояние между звездами в ангстремах - один ангстрем равен одной десятимиллионной части миллиметра. Обычно межзвездные расстояния измеряют в световых годах - это расстояние, которое солнечный луч проходит за год, - приблизительно 9,5 триллиона километров. И если выразить световой год в ангстремах, то получим 10 в 26 ангстрема. И расстояние до самых удаленных галактик не превышает ангстрем. Предположим, что Вселенная имеет ограниченные размеры (что не доказано) и сопоставим этот самый крупный физический объект, известный людям, с ядром атома - одним из самых малых объектов, изученных физиками. Соотношение между ними составит 10 в 40. Это также не гугол.

А теперь подсчитаем возраст Вселенной. Самое короткое время, которое мы используем в этом вычислении, составляет тот миг, который необходим световому лучу, чтобы пересечь диаметр атомного ядра. Получается, что возраст Вселенной в этих единицах составляет также 10 в 40. Пересчитаем все атомные частицы, существующие в известной нам Вселенной: протоны, электроны, нейтроны, а также нейтрино и фотоны. Даже в одной пылинке содержится несколько миллиардов элементарных частиц. А во Вселенной их 10 в 88 - то есть миллионная миллионной части гугола!

Энергия, излучаемая всеми звездами во Вселенной, должна быть исключительно велика. Но даже выраженная в микроваттах, она не достигает 10 в 40.

Гугол- недостижим, даже если подсчитать, сколько энергии содержится во всем веществе Вселенной.

Конечно, зная такие огромные числа. В этом случае запись числа занимает много места и мало наглядна. Неудобно было бы с ними работать . Поэтому решено было изменить написание таких чисел. При записи больших чисел часто используют степень числа 10.

Таким образом,

Тысяча - 1000 = 10 в 3

Миллион - 1000000 = 10 в 6

Биллион - 1000000000=10 в 9

Триллион -1000000000000 = 10 в 12

Квадриллион - 1000000000000000=10 в 15

Занятие №4 - 5. Приёмы быстрого счёта + презентация

Умножение на 5.

Вместо умножения числа а на 5

можно разделить его на 2 и умножить на 10:

36 х 5 = (36 : 2) х 10 = 18 х 10 = 180

124 х 5 = (124 : 2) х 10 = 62 х 10 = 620

Умножение на 25.

Чтобы число умножить на 25, можно его разделить на 4 и умножить на 100:

32 х 25 = 32 :4 х 100 = 800

Умножение на 11.

а)Если сумма цифр двузначного числа не превышает 10, надо:

1)цифры этого числа раздвинуть;

2)поставить между ними сумму этих цифр.

72 х 11 = 7(7 + 2)2 = 792

б) Если сумма цифр равна 10 или больше 10, надо:

1)мысленно раздвинуть цифры этого числа;

2)поставить между ними сумму этих цифр;

3) к первой цифре прибавить единицу, а вторую и третью оставить без изменения.

Умножение на 111, 1111, 11111 (для двузначных чисел, сумма цифр которых меньше 10) .

1) Мысленно цифры этого числа раздвинуть на 2, 3 и т.д. шагов.

2) Сложить цифры

3) Записать полученную сумму между раздвинутыми цифрами

2, 3 и т.д. раз

24 х 111 = 2(2 + 4)(2 + 4)4 = 2664

36 х 1111 = 3(3 + 6)(3 + 6)(3 + 6 )6 = 39996

«Задачи на делимость».
1. Мужичок привез продавать фуки, глюки и друки. Пройдясь по рынку, он решил увеличить им цену, добавив еще по одному нулю, но не в конце, а в середине чисел. В результате цена за один фук увеличилась в 6 раз, за глюк - в 7 раз, а за друк - в 9 раз. Сколько они стали стоить, если первоначальная цена каждого из них была меньше 100 рублей?
2. В корзине лежит меньше 100 яблок. Их можно разделить поровну между двумя, тремя или пятью детьми, но нельзя разделить поровну между четырьмя детьми. Сколько яблок в корзине?
3. Если из задуманного трехзначного числа вычесть 7, то полученная разность разделится на 7, если вычесть 8, то полученная разность разделится на 8, если вычесть 9, то полученная разность разделится на 9. Какое наименьшее из возможных чисел задумано?
4. Коля и Петя купили одинаковые беговые лыжи. Сколько стоит одна пара лыж, если Петя уплатил стоимость лыж трехрублевыми ассигнациями, Коля - пятирублевыми, а всего они дали в кассу меньше 10 ассигнаций?
5. Докажите, что сумма четырех последовательных нечетных чисел делится на 8.
Домашнее задание:
Ø Ковбой Джо зашел в бар и попросил у бармена бутылку виски за 3 доллара, трубку за 6 долларов, 3 пачки табака и 9 коробок непромокаемых спичек, цену которых он не знал. Бармен потребовал 11 долларов 80 центов (в одном долларе 100 центов), на что Джо вытащил револьвер. Бармен сосчитал снова и исправил ошибку. Как Джо догадался, что бармен пытался его обсчитать?
Ø Найти наименьшее число, которое делится на 41, а при делении на 39 дает в остатке 24.
Ø Найдите натуральные числа, дающие при делении на 2, 4, 5 и 6 остаток 1 и, кроме того, делящиеся на 7.
«Признаки делимости на 3 и на 9».
1. За альбом стоимостью 12 руб., книгу стоимостью 24 руб., 6 коробок карандашей и 9 линеек кассир выбил чек на 202 руб. 85 коп. И хотя покупатель не обратил внимание на стоимость карандашей и линеек, сразу определил, что кассир ошибся. Какое он имел на это основание?
2. Андрей нашел произведение всех чисел от 1 до 11 включительно и записал результат на доске. Во время перемены кто-то случайно вытер три цифры, и в записи осталось число 399*68**. Помогите восстановить цифры, не прибегая к повторному вычислению.
3. Найдите цифры сотен и единиц числа 72*3*, если число делится без остатка на 45.
4. Докажите или опровергните утверждение: «Разность между трехзначным числом и суммой его цифр всегда делится на 9».
5. Игра: Задумайте многозначное число. Найдите сумму цифр этого числа, отнимите ее от задуманного числа, затем в полученном числе зачеркните одну цифру и сообщите все остальные. Я немедленно назову вам зачеркнутую цифру.
Домашнее задание:
Ø Произвольно взято 2 натуральных числа и составлены сумма, разность и произведение их. Докажите, что среди этих 3 чисел по крайней мере одно, кратное 3.
Ø Докажите, что из любых 11 чисел всегда можно выбрать два таких числа, разность которых кратна 10.
Ø Разложите число 75 на два слагаемых так, чтобы большее из них было бы в 3 раза больше их разности.
«Признаки делимости на 7, на 11, на 13».
1. Если к любому двузначному числу приписать справа число, записанное теми же цифрами, но в обратном порядке, то получим четырехзначное число, делящееся на 11 без остатка. Докажите это.
2. Если сумма первой и второй цифр трехзначного числа, у которого одинаковые цифры сотен и единиц, делится на 7, то и число делится на 7. Докажите.
3. Разобьем некоторое четырехзначное число справа налево на грани по две цифры в каждой и сложим эти грани. Докажите, что если полученная сумма делится на 11 без остатка, то и испытуемое число кратно 11.
4. Докажите следующий признак делимости на 11: если сумма цифр через одну равна сумме остальных цифр через одну или разность этих сумм делится на 11, то и данное число делится на 11.
5. Докажите обобщенный признак делимости на 7, на 11, на 13: Разобьем число на грани по три разряда в каждой, считая справа налево. Если разность сумм граней данного числа, взятых через одну, делится на 7 (или на 11, или на 13), то и данное число делится соответственно на 7 (или на 11, или на 13).
Домашнее задание:
Ø Докажите, что слово ХАХАХА делится на 7, если в нем буквами Х и А обозначены любые цифры. (Одинаковые буквы обозначают одинаковые цифры, разные буквы - разные цифры).
Ø Из трех цифр, среди которых нет нуля, образовали все возможные трехзначные числа с различными цифрами. При этом оказалось, что сумма двух самых больших из этих чисел равна 1444. Каковы взятые цифры?
Ø Докажите, что если в трехзначном числе средняя цифра равна сумме крайних, то число делится на 11.

V Полезно помнить

1) 37·3=111

37·6=37·3·2

37·12=37·3·4

37·15=37·3·5

2) 7·11·13=1001

77·39=77·13·3

77·26=77·13·2

3) 91·11=1001

91·22

Умножение на 9 и на 11.

Чтобы устно умножить число на 9, приписывают к нему нуль и отнимают множимое.

Например.

  1. 62·9=620-62=600-42=558

  2. 73·9=730-73=700-43=657

Чтобы устно умножить число на 11 приписывают нуль и прибавляют множимое.

  1. 8Юный математик дополнительные занятия в 5 классе7·11=870+87=957

  2. 34·11=340+34=374

  3. Расшифруйте запись умножения.

Юный математик дополнительные занятия в 5 классеЮный математик дополнительные занятия в 5 классе

б)

х ГГГГ

ГГГ

АААА

+АААА

АААА

АБВГДА

в)

х **,*

Юный математик дополнительные занятия в 5 классе2,*7

+ **,**

**,835

Занятие 6. Магические квадраты. + презентация

Одно из самых загадочных произведений изобразительного искусства хранится в Кунстхалле города Карлсруэ. Речь о гравюре Альбрехта Дюрера «Меланхолия I» (1514).

Значимая деталь, изображенная на гравюре «Меланхолия I» - составленный впервые в европейском искусстве магический квадрат 4 Х 4. Сумма чисел в любой строке или столбце равна 34. Два средних числа в нижнем ряду указывают дату создания картины 1514 год.

Размерность квадрата 4*4. Он заполнен числами от 1 до 4*4(16) таким образом, что сумма чисел на любой горизонтали, вертикали и диагонали равна 34. Эта сумма также встречается во всех угловых квадратах 2×2, в центральном квадрате (10+11+6+7), в квадрате из угловых клеток (16+13+4+1), в квадратах, построенных «ходом коня» (2+8+9+15 и 3+5+12+14), в прямоугольниках, образованных парами средних клеток на противоположных сторонах (3+2+15+14 и 5+8+9+12).

Магические квадраты - это таблицы чисел, в которых суммы чисел в каждой строке, в каждом столбце и в каждой из двух диагоналей квадрата все равны между собой.

Магические квадраты были известны еще арабам, к которым вероятно, они перешли от индусов; затем они сделались достоянием математиков восточной части Римской империи и, наконец, появились в Западной Европе, где методами получения магических квадратов заинтересовались многие ученые. В средние века люди верили в магическую силу этих квадратов. Они использовались для изготовления талисманов, оберегающих от различных болезней.

Из всякого магического квадрата путем различных перестановок составляющих его чисел можно получить множество новых магических квадратов, обладающих теми же свойствами.

Известно, что магических квадратов 2х2 не существует (может быть, кто-нибудь это докажет?).

Магический квадрат 3х3 только один.

Магических квадратов 4х4, как на картине Дюрера, составлено уже 800, а количество магических квадратов 5х5 близко к четверти миллиона!

Магический квадрат - древнекитайского происхождения. Согласно легенде, во времена правления императора Ю (ок. 2200 до н.э.) из вод Хуанхэ (Желтой реки) всплыла священная черепаха, на панцире которой были начертаны таинственные иероглифы, и эти знаки известны под названием ло-шу. В 11 в. о магических квадратах узнали в Индии, а затем в Японии, где в 16 в. магическим квадратам была посвящена обширная литература. Европейцев с магическими квадратами познакомил в 15 в. византийский писатель Э.Мосхопулос. Первым квадратом, придуманным европейцем, считается квадрат А.Дюрера, изображенный на его знаменитой гравюре Меланхолия I.

Каждый элемент магического квадрата называется клеткой. Квадрат, сторона которого состоит из n клеток, содержит n² клеток и называется квадратом n-го порядка. В 16 в. Корнелий Генрих Агриппа построил квадраты 3-го, 4-го, 5-го, 6-го, 7-го, 8-го и 9-го порядков, которые были связаны с астрологией 7 планет. В 19 и 20 вв. интерес к магическим квадратам вспыхнул с новой силой. Их стали исследовать с помощью методов высшей алгебры и операционного исчисления.

Бытовало поверье, что выгравированный на серебре магический квадрат защищает от чумы. Магическим квадратам приписывали различные мистические свойства. Даже сегодня среди атрибутов европейских прорицателей можно увидеть магические квадраты.

Рассмотрим удобный способ заполнения магического квадрата 3-го порядка. Наш квадрат разделен на 9 равных клеток. Необходимо расставить в этих клетках числа 1, 2, 3, 4, 5, 6, 7, 8, 9 так, чтобы сумма чисел в каждой строке и в каждом столбике равнялась 15.

Юный математик дополнительные занятия в 5 классеЮный математик дополнительные занятия в 5 классе Юный математик дополнительные занятия в 5 классе Юный математик дополнительные занятия в 5 классе Юный математик дополнительные занятия в 5 классе
Рис. 1. Рис. 2. Рис. 3. Рис. 4. Рис. 5.


1. Добавим «крылышки» в средний столбец и в среднюю строку.
2. Выделим по диагоналям клетки, которые мы заполним числами.
3. Запишем в выделенные клетки числа от 1 до 9.
4. Перенесем числа из «крылышек» во внутреннюю часть квадрата, как показано на рисунках 3, 4, 5.
Анимационный вариант решения в презентации.


Используя рассмотренный алгоритм, можно решать занимательные задачи с магическими квадратами.

1. В клетках квадрата переставьте числа так, чтобы по любой вертикали, горизонтали и диагонали их суммы были равны между собой (рис. 6). Решение в презентации.
2. Даны числа: 5, 10, 15, 20, 25, 30, 35, 40, 45. Впишите их в клетки девятиклеточного квадрата так, чтобы получилось в сумме одно и то же число по любой вертикали, горизонтали и диагонали. Решение в презентации.
3. Разместите в свободных клетках квадрата еще числа 3, 4, 5, 6, 7, 8, 9 так, чтобы по любой вертикали, горизонтали и диагонали получилось в сумме одно и то же число (рис. 7).Решение в презентации.

Занятие 7-8. Математические фокусы.+ презентация

Ни одно человеческое исследование
не может назваться истинной
наукой, если оно не прошло через
математические доказательства.
Леонардо да Винчи

Учитель: сегодня мы проводим первое занятие математического кружка в этом учебном году. Тему занятия мы с вами узнаем чуть позже. А сейчас поработаем устно.

2. Актуализация опорных знаний.

  1. Выполните цепочку простейших вычислений (слайд 2).

  2. Найдите сумму цифр данных чисел (слайд3).

  3. Разложите на разрядные слагаемые числа (слайд3):

  4. Ответьте на вопросы (слайд 4):
    Каким числом является сумма двух четных, двух нечетных, четного и начетного чисел?
    Каким числом является произведение двух четных, двух нечетных, четного и начетного чисел?

3. Изучение нового материала (слайд 5).

Учитель: итак, основную часть нашего занятия мы посвятим арифметическим фокусам. Арифметические фокусы - это эксперименты, основанные на свойствах чисел и действий, математических законах. И понять суть того или иного фокуса - это значит понять пусть небольшую, но математическую закономерность. Математических фокусов очень много, они появились вместе с возникновением математики, как науки. Их можно найти в различной литературе, а можно придумать и самим. Сегодня на нашем занятии мы не только познакомимся с некоторыми из них, но и попробуем разгадать их секреты.

Фокус 1. «Отгадывание суммы очков на открытых гранях» (слайд 6).

Учитель: мне нужен помощник, сложите три игральных кубика столбиком. Введем в компьютер число очков на верхней грани столбика., и компьютер угадает сумму очков на гранях, по которым кубики соприкасаются, и на самой нижней грани (слайд 7).

Секрет фокуса (слайд8). В самом деле, если бы складывались очки, соответствующие всем горизонтальным граням трех кубиков, то есть очки, соответствующие трем парам взаимно противоположных граней кубиков, то такая сумма составляла бы ровно 21 (3 ·7 = 21). Но в сумме, обусловленной зада­чей, не участвует число очков, соответствующих верхней грани. Вы­читая это число из 21, мы получим искомую сумму.

Фокус 2. «Белая пешка» (слайд 9).

Учитель: готовясь к игре в шахматы, партнер тайно от вас зажимает в один кулак белую пешку, а в другой - черную. Вам хочется выбрать белую, и вы, при помощи несложного фокуса беретесь угадать, в каком она кулаке: правом или левом. Вы говорите партнеру: (слайд 10) "Оценим черную пешку числом 1, а белую числом 2. Числовое значение пешки, зажатой в правой руке, умножь на какое хочешь четное число, а числовое значение другой пешки умножь на любое нечетное число. Сложи результаты и объяви последнюю цифру суммы. Теперь я скажу безошибочно, в какой руке белая пешка". А у нас это угадает компьютер (слайд 11).

Секрет фокуса (слайд 12). Если объявленное число четное, то белая пешка в левой руке, а если нечетное, то в правой. Произведение четного числа на нечетное - четно, нечетного на нечетное - нечетно, сумма двух четных чисел - четна, а сумма четного и нечетного - нечетна.

Фокус 3. «Сколько палочек в кулаке?» (слайд 13)

Учитель: для показа этого фокуса возьмем коробочку с 20 палочками. Мне нужен опять один помощник. Вытяните из коробка несколько палочек (не больше 10) и положите в карман. Пересчитайте оставшиеся в коробочке палочки. Допустим, их 14. Это число нужно «выписать палочками» на столе следующим образом: единица изображается одной палочкой, положенной слева, а четверка - четырьмя палочками, положенными несколько правее. Эти пять палочек берутся из числа оставшихся в коробочке. После этого палочки, изображавшие число 14, тоже кладутся в карман. Достаньте из коробка ещё несколько палочек и зажмите их в кулаке. Сейчас я высыплю палочки из коробка на стол, введу одно число в компьютер и компьютер угадает число палочек, зажатых в кулаке (слайд 14).

Секрет фокуса (слайд 15).Количество палочек, спрятанных в карман за два шага всегда равно 11, а оставшееся в коробке - 9. Вычитаем из 9 число палочек, рассыпанных на столе, и получаем ответ.

Фокус 4. «Угадывание возраста и дня рождения» (слайд 16).

Учитель: сейчас мы узнаем совершенно точно, когда каждый из вас родился. Для этого я попрошу сделать ряд вычислений, которые вы должны выполнить аккуратно и правильно (слайд 17). Слушайте внимательно и выполняйте. Порядковый номер месяца вашего рождения умножайте на 100. К полученному произведению прибавьте число месяца. Теперь полученную сумму умножьте на 2 и к новому произведению прибавьте 8. Новую сумму надо умножить на 5 и к полученному произведению прибавить 4. Умножьте опять полученную сумму на 10 и опять прибавьте 4. Затем прибавьте полное число ваших лет. Назовите окончательный результат. Введите полученное число в компьютер. Компьютер угадает дату вашего рождения и ваш возраст(слайд 18).

Секрет фокус (слайд 19). Допустим, это будут числа 101676 и 50964. В уме отнимаем от написанных чисел всегда 444, и остаток разбиваем справа налево на группы по две цифры в каждой (слайд 20):

Юный математик дополнительные занятия в 5 классе

Последние две цифры показывают полное число лет; вторая группа - число месяца, а первая группа - порядковый номер месяца. Объявляйте первому человеку: "Вам 32 года, вы родились 12 октября". Второму человеку говорите: "Вам полных 20 лет, вы родились 5 мая". Если обозначить порядковый номер месяца буквой а, число месяца - в, а число полных лет - с, то все производимые выше вычисления будут выражены следующей формулой:

(((100а+в)·2+8)·5+4)·10+4+с=10000а+100в+444+с. Если отнять число 444, то получается: 10000а+100в+с.

Фокус 5. «Математические забавы из «Арифметики» Л.Магницкого» (слайд 21)

Учитель: первое упоминание о математических фокусах встречаются в книге русского математика Леонтия Филипповича Магницкого с длинным названием "Арифметика, сиречь наука числительная, с разных диалектов на славянский язык переведенная и во едино собрана и на две книги разделена…", опубликованной в 1703 году и содержащей начала математических знаний того времени.

Одна глава книги была названа автором "Об утешных некиих действах, через арифметику употребляемых". Эта глава содержала математические игры и фокусы. Сам Магницкий пишет, что поместил эту главу в книгу для "утехи и особенно для изощрения ума учащихся". Вот один из фокусов…

Я попрошу выйти к доске несколько учениц. Сейчас мы узнаем, кто из них взял перстень, на какой палец надел его и на какой именно сустав пальца. Сначала договоримся о номерах участниц, пальцев и суставов (слайд 22).

Сейчас я попрошу вас выполнить следующие действия: номер, взявшего перстень умножить на 2; к произведению прибавить 5; сумму умножить на 5; к полученному числу прибавить номер пальца, результат умножить на 10; к произведению прибавить номер сустава; введем полученный результат в компьютер, который угадает у кого перстень, на каком пальце и на каком суставе одет(слайд 23).

Секрет фокуса (слайд 24). От полученного в результате арифметических действий числа нужно отнять 250: в остатке первая цифра слева - это номер взявшего кольцо, вторая - номер пальца, третья - номер сустава. Пусть, например, перстень взял 6-й ученик и надел его на 3-й сустав безымянного пальца (то есть 4-го пальца), тогда: ((6·2+5) ·5+4) ·10+3=893

  • 893-250=643

  • 6·2·5·10=600

  • 4·10=40

  • 5·5·10=250.

Фокус 6. «Математическая забава М. Ю. Лермонтова» (слайд 25)

Учитель: все вы знакомы с творчеством великого русского поэта М.Ю. Лермонтова, но не каждому известно, что он был большим любителем и математики, особенно его привлекали математические фокусы, которых он знал великое множество, причем некоторые из них он придумывал сам.

Вот отрывок из воспоминаний однополчанина поэта Е. И. Мейделя о забавном случае, связанном с пребыванием Михаила Юрьевича в крепости (в Анапе) «… Зимой офицеры анапского гарнизона, проходя службу в захолустном местечке, собирались по вечерам у кого-либо из друзей и развлекались от скуки как могли. Однажды, находясь в такой компании, Лермонтов предложил: "Задумайте какую угодно цифру, и я с помощью простых арифметических действий, которые вы будете проводить со мною, определю эту цифру". В итоге Лермонтов всегда безошибочно называл ее. Батальонный был изумлен: "Фу ты... Да вы уж не колдун ли?!" Поэт улыбнулся: "Колдун - не колдун, а математике учился", и раскрыл секрет фокуса…»

Вот один из фокусов М.Ю. Лермонтова: задумать любое число, прибавить к нему 25, прибавить еще 125, отнять 36, вычесть задуманное число, остаток умножить на 5, полученное число разделить на 2. Посмотрите на экран компьютера. Получится 285 (слайд).

Секрет фокуса (слайд 26).(а + 25 + 125 - 36 - а) · 5 : 2 = 114 · 5 : 2 = 285.

Как видно, в процессе выполнения действий задуманное число а исключается, и собеседник выполняет остальные действия только над теми числами, которые дает сам отгадчик. Вместо чисел 25, 125, 36, 5 и 2 можно брать, конечно, и другие числа, но тогда и ответ будет иной.

Фокус 7. «Математический фокус Дэвида Копперфильда» (слайд 27)

Учитель: фокусы знаменитого американского иллюзиониста восхищают и поражают зрителей не только сложностью и оригинальностью, но прежде всего грандиозностью замысла и мастерством его воплощения, использованием сложнейших оптических эффектов, специальных устройств и приспособлений. Примечательно, что Дэвид Копперфильд включил в свои программы также серию математических фокусов, которые редко показывают на эстраде из-за того, что они не очень зрелищны. Тем не менее Копперфильду удалось найти эффектную подачу одного такого фокуса (слайд 28).

Фокусник размещает на экране пятнадцать предметов, например кружков, и выкладывает их в виде шестерки: в колечке - 12, а в хвостике - 3. Можно кружки заменить предметами. Задумайте любое число больше трех и отсчитайте его сверху вниз, начиная с первого кружка, по хвостику и далее по колечку против часовой стрелки. Затем снова посчитайте предметы до задуманного числа, начиная с того, на котором вы остановились, но на этот раз по часовой стрелке и только вокруг колечка. Удивительно то, что в результате вы все указываете на один и тот же кружок, который угадал и компьютер.

Фокусы такого типа называются фокусами с предопределенным выбором. Они основаны на том, что, независимо от варианта схемы (количества предметов на хвостике или предметов на колечке), действий фокусника и зрителей, результат предсказуем и будет одним и тем же для всех участников, несмотря на то, что каждый из них задумал свое число.

Секрет фокуса. Итак, независимо от того, какое первоначальное число задумал зритель, счет заканчивается всегда на одном и том же предмете. Чтобы его найти, нужно хвостик шестерки наложить на колечко по часовой стрелке, начиная с предмета, следующего (тоже по часовой стрелке) за тем, к которому подходит хвостик. Кончик хвостика ляжет на задуманный предмет на колечке. Все остальные манипуляции фокусника - лишь отвлекающий маневр для того, чтобы замаскировать этот факт (слайд 28).

Легко догадаться, для чего фокусник ставит ограничение на задуманное число (в нашем случае больше трех): только выполнение этого условия позволит зрителям при счете предметов попасть на кольцо - основную фигуру для манипуляции. Узнав секрет фокуса, его можно изменить по собственному усмотрению.

Фокус 8. «Удивительные часы» (слайд 29)

Учитель: некоторая вариация описанного фокуса Дэвида Копперфильда - угадывание задуманного числа на циферблате часов.

Задумайте какой-нибудь час (от 1 до 12). Задуманный вами час запомните. Теперь я буду указкой постукивать по часам. Каждый раз, когда постучу, прибавляйте к задуманному вами числу по одному. Когда вы досчитаете до двадцати, остановите меня. В этот момент моя указка укажет на часах задуманное вами время.

Секрет фокуса. Вначале нужно ударять указкой по циферблату по любым делениям до семи ударов. Восьмым ударом показывается число 12, а потом с каждым ударом перемещаемся влево (11, 10, 9 и т.д.) Когда вы скажете: "Довольно", - указка будет стоять на том часе, который вы задумали. Расчет очень простой. Всего будет ударов (20-х). Когда будет сделано восемь ударов, указка покажет число 12. С этого момента мы делаем еще столько ударов, сколько не достает вам до двадцати, так как, двигаясь влево, будут показываться числа, последовательно уменьшенные на единицу.

Следует заметить, что совсем не обязательно просить зрителя прекращать счет именно на двадцати. Фокусник может предложить ему самому выбрать число для окончания счета, нужно лишь, чтобы оно было больше двенадцати. Но есть одно условие: зритель должен предупредить фокусника, на каком числе он собирается остановиться. Тогда, как вы уже догадываетесь, необходимо отнять от этого числа 12, чтобы получить число прикосновений к циферблату, которые можно сделать наугад, прежде чем коснуться числа 12 и начать двигаться последовательно против часовой стрелки.

4. Подведение итогов занятия.

5. Постановка домашнего задания (слайд 30).

Всем учащимся раздается индивидуальное творческое задание:

1. Разобрать фокус с карточки; показать фокус и объяснить его секрет на следующем занятии.

2. Придумать свой фокус.

Фокусы для домашнего задания:

«Лесной» фокус

Узнаете деревья по нарисованным веткам? Правильно: клен, дуб, ива. Теперь замените каждую букву цифрой, но так, чтобы ни одна цифра не повторялась. Сложите образовавшиеся числа и объявите в любом порядке все цифры результата, кроме любой одной. Найдите сумму объявленных цифр и введите ее в компьютер. Компьютер покажет утаенную цифру.

Секрет фокуса. Сумма десяти цифр равна 45 независимо от их расположения в трех слагаемых, следовательно, делится на 9. Сумма чисел, образованных из этих цифр, является числом, кратным 9, следовательно, и сумма его цифр должна делиться на 9. Поэтому, чтобы выявить утаенную цифру, надо сложить объявленные цифры; тогда число, дополняющее эту сумму цифр до ближайшего числа, кратного 9, определит утаенную цифру. (Например, получилось 13, тогда 18-13=5).

«Таинственные квадраты в календаре»

Выделите из календаря на любой месяц квадрат 4Х4, содержащий 16 чисел. Выберите наибольшее из них и введите в компьютер. Компьютер угадает сумму всех шестнадцати чисел выбранного квадрата.

Секрет фокуса. Если р - наибольшее число в указанном квадрате, то всякий такой квадрат имеет вид:

р-24

р-17

р-10

р-3

р-23

р-16

р-9

р-2

р-22

р-15

р-8

р-1

р-21

р-14

р-7

р

и сумма всех чисел квадрата равна 16р-192=16(р-12).

(Например, пусть наибольшее число - 24, тогда 16(24-12)=192).

«Мгновенное умножение трехзначного числа на 999»

Можно мгновенно умножить любое трехзначное число на 999. Например, 573·999=572427.

Секрет фокуса. В результате умножения получается шестизначное произведение: первые три цифры его есть умножаемое число, только уменьшенное на единицу, а остальные три цифры (кроме последней) - «дополнения» первых до 9. Стоит лишь взглянуть на следующую строку, чтобы понять происхождение этой особенности: 573·999= 573·(1000-1)= 573000-573 =572427.

«Число Шахерезады»

Напишите на бумажке (не показывая) трехзначное число, а затем припишите еще раз то же самое число. Полученное шестизначное число разделите сами (или предложите любому другому) разделить, не показывая, без остатка на 7. Результат деления еще раз разделите сами (или передав другому ученику) без остатка на 11, а затем на 13. После троекратного деления должно получиться загаданное число.

Секрет фокуса. Вспомним, что приписать к трехзначному числу его само - значит, умножить его на 1001 - число Шехерезады. Но 1001=7·11·13. а в результате деления последовательно на эти три числа оно должно снова дать полученное число.

«Число из любимой цифры»

Скажите, у кого какая любимая цифра (например, 5). Выполните умножение числа 15873 на 35 (любимая цифра, умноженная на 7) или числа 12345679 на 45 (любимая цифра, умноженная на 9). Получится произведение, записанное только любимой цифрой.

Секрет фокуса.

1) 15873·7=111111, 11111·5=555555, значит, 15873·35=555555.

2) 12345679·9=111111111,111111111·5=555555555,

значит, 12345679·45=555555555.

«Быстрое суммирование»

Можно удивить своих товарищей искусством суммирования чисел. Сделать это можно так. Напиши на классной доске какое-нибудь многозначное число, например, 450678. Можешь написать любое число, пусть только число единиц вэтом числе будет не меньше 2. Предложи далее кому-нибудь из товарищей подписать под этим числом, как делается при сложении, любое число, имеющее столько же знаков. Вслед за этим сам подпиши третье слагаемое, цифры которого дополняли бы соответствующие цифры второго слагаемого до 9. Пусть затем кто-либо из товарищей подпишет любое четвертое слагаемое (с тем же числом знаков). Пятое слагаемое подпиши также сам, как и третье. Сумму получившихся пяти чисел ты можешь написать моментально. Начни сединиц. Их должно быть на 2 меньше, чем в первом числе. Дальше последовательно перепиши все цифры первого числа и впереди поставь 2. Вот и все. На доске получится, например, такая запись:

  • 450678

  • 329157

  • 670842

  • 257934

  • 742065

  • 2450676

То же самое можно проделать, взяв не пять, а семь слагаемых. Только в этом случае число единиц в сумме будет на 3 меньше числа единиц первого слагаемого и впереди придется писать не 2, а 3.

«Угадайте задуманное число»

В своей книге "Арифметика" Леонтий Филиппович Магницкий привел следующий способ отгадывания задуманного двузначного числа: "Если кто задумает двузначное число, то ты скажи ему, чтобы он увеличил число десятков задуманного числа в 2 раза, к произведению прибавил бы 5 единиц, полученную сумму увеличил в 5 раз и к новому произведению прибавил сумму 10 единиц и числа единиц задуманного числа, а результат произведенных действий сообщил бы тебе. Если ты из указанного тебе результата вычтешь 35, то узнаешь задуманное число".

Секрет фокуса. ((2а+5)·5+10+в)=10а+в+35.

«Угадайте сумму цифр задуманного числа»

Предложите каждому задумать какое-нибудь трехзначное число, запись которого не содержит одинаковых цифр. Пусть затем, беря цифры задуманного числа по две, каждый составит всевозможные двузначные числа (таких чисел будет 6) и вычислит сумму всех этих чисел. Спросите у любого участника: какая сумма получилась? Разделите ее на 22, и вы найдете сумму цифр задуманного числа. Секрет фокуса. Пусть, например, задумано число 145. Сумма всех двузначных чисел для этого числа будет равна 14+15+45+41+51+54 = 220. Если вы разделите эту сумму на 22, то действительно получите 10 - сумму цифр задуманного числа.

Занятие 9-10. Математические ребусы.


МАТЕМАТИЧЕСКИЕ ЗАГАДКИ

Прах Диофанта

Прах Диофанта гробница покоит;

Дивись ей - и камень

Мудрым искусством его

Скажет усопшего век.

Волей богов шестую часть жизни он прожил ребенком

И половину шестой встретил с пушком на щеках.

Только минула седьмая, с подругою он обручился,

С нею, пять лет проведя, сына дождался мудрец.

Только полжизни отцовской возлюбленный сын его прожил,

Отнят он был у отца ранней могилой своей.

Дважды два года родители оплакивали тяжкое горе.

Тут и увидел мудрец предел жизни печальной своей.

(64 года)

Любитель порядка

Настольная лампа,

Зеленый диван,

Сидит на диване

Матюшин Иван.

Он пишет...

Не будем, ребята, мешать,

А только тихонько

Заглянем в тетрадь.

В тетрадке написано

Все по порядку:

«В семь двадцать встаем,

Производим зарядку.

В восемь тридцать,

Умывшись холодной водой,

Застелем постель

И займемся едой.

Без четверти восемь

Дрова мы приносим.

Готовим по плану

Похлебку Полкану -

И в класс направляемся

В восемь ноль пять».

Вопрос: сколько времени уходит у Ивана от подъема до выхода в школу?

(45 мин)

Жуки и пауки

Вопрос: сколько ног у жука?

Сколько ног у паука?

У меня в одной коробке три жука,

А в другой имею я три паука.

В уголке шуршат бумагой два ежа,

А в двух клетках распевают два чижа.

Кто, ребята, сосчитать бы мне помог,

Сколько вместе все они имеют ног?

(54 ноги)

Два цыпленка стоят,

Два в скорлупках сидят.

Шесть яиц под крылом

У наседки лежат.

Посчитай поскорей,

Отвечай поточней:

Сколько будет цыплят

У наседки моей?

(10)

Как-то вечером к медведю

На порог пришли соседи:

Еж, барсук, енот, «косой»,

Волк с плутовкою-лисой.

А медведь никак не мог

Разделить на всех пирог.

От труда медведь вспотел -

Он считать ведь не умел!

Помоги ему скорей,

Посчитай-ка всех зверей.

(7)

Скоро 1 0 лет Сереже -

Диме нет еще шести.

Дима все никак не может

До Сережи дорасти.

А на сколько лет моложе

Мальчик Дима, чем Сережа?

(на 4 года)

Я сегодня рано встала,

Кукол всех своих считала:

Три матрешки - на окошке,

Две Маринки - на перинке,

Пупсик с Катей, Буратино

И Петрушка в колпачке

На зеленом сундучке.

Я считала, я трудилась,

Но потом со счета сбилась.

Помогите мне опять

Кукол всех пересчитать!

(9 кукол)

Кто стучится в дверь ко мне

С толстой сумкой на ремне,

С цифрой 5 на медной бляшке,

В синей форменной фуражке.

Это он, это он,

Ленинградский почтальон.

В семь часов он начал дело,

В 1 0 сумка похудела,

А к 12 часам

Все разнес по адресам.

Сколько ж в день часов он сам

Все ходил по адресам?

(5 часов)

Посадила бабка в печь

Пирожки с капустой печь.

Для Наташи, Маши, Тани,

Коли, Оли, Гали, Вали

Пирожки уже готовы.

Да еще один пирог

Кот под лавку уволок.

Да в печи четыре штуки,

Пироги считают внуки.

Если можешь, помоги

Сосчитать им пироги.

(12 пирогов)

ПОДАРОК

Нам из Гомеля тетя

Ящик яблок прислала,

В этом ящике яблок

Было, в общем, немало.

Начал яблоки эти

Спозаранок считать я.

Помогали мне сестры,

Помогали мне братья.

И пока мы считали,

Восемь раз отдыхали,

Восемь раз мы сидели

И по яблоку съели.

И осталось их сколько?

Ох, осталось их столько,

Что, когда в этот ящик

Мы опять поглядели,

Там на дне его чистом

Только стружки белели.

Вот прошу угадать я

Всех ребят и девчонок:

Сколько было нас, братьев?

Сколько было сестренок? Поделили мы яблоки

Все без остатка,

А всего то их было 50 без десятка.

(3 брата, 2 сестрёнки)

Помощник

7 тарелок им умыты,

8 чашек не забыты,

Ложек - дюжина одна

Чистота кругом видна

Вы готовы дать ответы,

Сколько всей посуды этой

Перемыл он, сын-проказник?

Дело было в мамин праздник.

(27)

Над болотцем тихо, тихо...

В теплом воздухе парят

Сам Комар да Комариха,

С ними туча комарят!

Комариха с Комаром говорят:

- Сосчитай-ка, Комар, комарят.

- Как же счесть, Комариха, комарят?

Не поставишь комарят наших в ряд.

Насчитала Комариха сорок пар,

А продолжил счет сам Комар.

Комарят Комар до вечера считал,

Насчитал 13 тысяч и устал...

А теперь сосчитайте сами вы, друзья,

Велика ли комариная семья?

(13082)

Если Грушам дать по груше,

То одна в избытке груша,

Если дать по паре груш,

То не хватит пары груш.

Сколько Груш и сколько груш?

(3 девочки, 4 груши)

У деда Архипа - большая семья.

Детей всего восемь, и все сыновья.

У каждого по паре ребят, внучат Архипа.

Их сколько, внучат?

(14)

ПРОГРЕССИЯ

Задача очень непростая:

Как сделать, чтобы быстро

От единицы и до ста

Сложить в уме все числа?

Пять первых связок изучи -

Найдешь к решению ключи!

Давным-давно один мудрец сказал,

Что прежде надо

Связать начало и конец

У численного ряда.

(5050)

10 солдат строились в ряд,

10 солдат шли на парад.

9/ 10 было усатых,

Сколько там было безусых солдат?

10 солдат строились в ряд,

10 солдат шли на парад.

8/ 1 0 было носатых,

Сколько том было курносых солдат?

(1 безусый, 2 курносых)

22 совы скучали

На больших сухих суках,

22 совы мечтали

О 7 больших мышах.

О мышах довольно юрких,

В аккуратных серых шкурках.

Слюнки капали с усов

У огромных серых сов.

Сколько было сов?

(22 совы)

Вот несколько задачек

: 1. У стола отпилили один угол. Сколько углов у него теперь? А сколько углов будет, если отпилить два, три, четыре угла?

если стол бы четырехугольный (что не очевидно), то конечно стало пять. Или четыре. В зависимоти от линии, по которой резали. И в предположение, что отпиливается прямым резом, а не фигурным выпиливанием. Если отпилить еще угол - то смотря какой и опять же смотря по какой линии.

В общем, задача плохая, слишком много умолчаний в условии...

: 2. В тарелке лежали три морковки и четыре яболка. Сколько фруктов было в тарелке?

ну, что такое "фрукт" биологи до сих пор четкого определения не дали, а если по магазинному - так 4 яблока. Вот если яболко - не опечатка, тогда не знаю, фрукт ли это.

: 3. В люстре горело пять лампочек. Две из них погасли. Сколько лампочек осталось в люстре?

отличная задача. лампочек, ясен барабан, осталось пять (в задаче же ничего не сказано о том, что их вывернули).

: 4. У мамы дочка Даша, сын Саша, собака Дружок и кот Пушок. Сколько детей у мамы?

Биологических - два. Юридически тоже. А считает ли она свое зверье - детьми (пусть приемными) - вопрос отдельный.

: 5. В коридоре стоят 8 башмаком. Сколько детей играет в комнате?

м-да... а сколько детей сидит в сортире? без занния этого задача не решается. Можно предположить, что не больше 4-х, если исключить возможность, что кто-то пришел босиком или поперся в комнату в обуви.

: 6. У нескольких столов 12 ножек. Сколько всего столов в комнате? очевидно - несколько.

: 7. У кошки Мурки родились щенята: один черненький и два беленьких. Сколько щенят у Мурки?

щенят-то трое, но Мурка какая-то мутантная.

: 8. Прилетели два чижа, два стрижа и два ужа. Сколько стало птиц всего Возле дома моего?

ну если поскольку способность летать еще не делает ужа птицей (может его просто сильно и метко бросили), то птиц 4-ре. Но вот сколько из них село возле дома - неочевидно.

: 9. Сели на воду три воробья. Один улетел. Сколько осталось?

один остался. который улетел. остальные утонули.

: 10. Катится по столу колесо разноцветное: один угол у него красный, другой зеленый, третий желтый. Когда колесо докатится до края стола, какой цвет будет виден?

угол, я надеюсь, у стола. а колесо разноцветное ответ: откуда я знаю?

: 11. На полке стояли детские книжки. Подбежала собачка, взяла одну книжку, потом еще одну, потом еще две. Сколько книжек она прочитает?

ну кто же ее знает. не больше 4-х. Может она не все читать будет.

: 12. Мама уронила поднос, на котором стояли 2 чашки с цветочками, 2 в Горошек и 2 с ягодками. Сколько теперь стало чашек? зависит от мягкости пола (или на что она их уронила) и прочности чашек

: 13. На дубе три ветки, на каждой три яблока. Сколько всего яблок?

сколько всего или сколько на дубе? на дубе - девять. А всего яблок - их мно-о-ого...

: 14. Сколько цыплят вывел петух, если он снес 5 яиц? куда вывел? на прогулку?

много наверное. если они его настолько достали.

: 15. Один банан падает с елки каждые 5 минут. Сколько их упадет за один час?

ну это очевидно - 12. мало ли кто на елке с мешком бананов сидит.

: 16. По двору гуляли петух и курица. У петуха 2 ноги, а у курицы 4. Сколько ног гуляло по двору?

смотря как ноги у курицы растут и смотря что считать гуляющей ногой. Может они у нее кверху торчат, а гуляет она брюхом по земле, пресмыкаясь таки гад ползающий... Тогда ногу гуляющей считаем или нет?

: 17. На столе стояло 5 стаканов ягод. Миша съел один и поставил его на стол.

: Сколько стаканов стоит на столе?

так съел или поставил?

если съел - то 4. если не стал есть и поставил, то пять.

: 18. На Машином платье были вышиты три вишни и два яблока. Съели одну вишню и два яблока. Сколько фруктов осталось?

На платье - осталось вышито 5 изображений фруктов (если вишня - фрукт, тут тоже есть путаница). А фруктов осталось на три меньше, чем было.

Это что, курс "дзен для первоклассников" ?

Что-то в этом есть, задам своему первокласснику, пусть знает, как звучит хлопок одной ладонью.

ВОПРОСНИК:

1. Какие часы два раза в сутки показывают верное время? (которые стоят)

2. Что случится с красным платком, если его опустить на дно моря на 5 минут? (намокнет)

3. Как далеко в лес может забежать заяц? (до середины, дальше уже из леса)

4. Можно ли в решете принести воду? (можно, когда она замерзнет)

5. Летело 3 страуса, одного сбили. Сколько осталось лететь? (Страусы не летают)

6. Когда мы смотрим на цифру 2, а говорим 10? (когда смотрим на часы, минутная стрелка)

7. В каком числе столько же цифр, сколько букв в его названии? (100 - сто)

8. Какие три числа, если их сложить или перемножить дают один и тот же результат? (1, 2, 3)

9. Назовите пять дней не называя чисел и дней недели. (позавчера, вчера, сегодня, завтра, послезавтра)

10. Месяца имеют 30-31 день, а какой месяц имеет 28 дней? (каждый)

11. Как нужно трижды записать цифру 3, чтобы в итоге получилось 4? (3:3+3)

12. Напишите 100 пятью единицами, а затем пятью пятерками.

(111-11), (5+5+5+5)•5, (5•5•5-5•5)

13. Сумма каких двух натуральных чисел равна их произведению? (2+2=2·2)

14. Человек зашел в комнату и увидел там 6 собак и 2 кошки с котятами. Сколько ног в комнате?

(две)

15. В комнате 4 угла, в каждом углу по кошке. Напротив каждой кошки по 3 кошки. Сколько кошек в комнате? (четыре)

16. Человек шел в Ленинград, ему навстречу отряд ребят. У каждого по лукошку, а в лукошке по кошке, у кошки по пять котят. Сколько человек шло в Ленинград? (один)

17.Представь, что ты машинист и ведешь поезд. В нем 16 вагонов: 2 вагона с дровами, 3 - с нефтью, остальные с углем. Сколько лет машинисту? (столько сколько тебе)

18. Трое играли в шашки. Всего сыграли три партии. Сколько партий сыграл каждый? (две)

19. Летела стая гусей. Один гусь впереди и два позади. Один позади и два впереди. Один между двумя и три в ряд. Сколько их было? (трое)

20. Две дочери, две матери, да бабушка с внучкой. Сколько всех? (трое)

21. Как можно одним мешком пшеницы, смолов её, наполнить два таких же мешка?

(надо один из пустых мешков вложить в другой такой же,а затем в него насыпать пшеницу)

22. Что это может быть: две головы, две руки, шесть ног, а идут или бегут только четыре?

(всадник на лошади)

23. Один человек купил трех коз и заплатил 3 рубля. Спрашивается: «По чему пошла каждая коза?» (по земле)

24. Двое пошли - три гвоздя нашли. Следом четверо пойдут - много ли гвоздей найдут?

(скорее всего ни одного)

25. Два землекопа выкапывают 2 м канавы за 2 часа. Сколько землекопов за 5 часов выкопают 5 м канавы? (два землекопа)

26. Одно яйцо варят 4 минуты, сколько минут нужно варить 5 яиц? (4 минуты)

27. В семье 5 сыновей и у каждого есть сестра. Сколько детей в этой семье? (шесть)

28. Два отца и два сына поймали трех зайцев, а досталось всем по одному зайцу. Спрашивается: «Как это могло случиться?» (дед, сын, внук)

29. Написать цифрами число, состоящее из 11 тысяч, 11 сотен и 11 единиц.

(11000 + 1100 + 11 = 12 111)

30. Чтобы найти пиратский клад надо пройти от старого дуба 12 шагов на север и 5 шагов на юг, затем 4 на север и 11 на юг. Где зарыт клад? (у старого дуба)

31. Две ноги на трех ногах, а четвертая в зубах.

Вдруг четыре прибежали и с одною побежали.

Подскочили две ноги, закричали на весь дом

Да тремя по четырем, но четыре завизжали

И с одною убежали. (Человек, собака, стул и куриная ножка)

Занятие 11. Софизмы.

Математический парадокс можно определить как истину, настолько противоречащую нашему опыту, интуиции и здравому смыслу, что в нее трудно поверить даже после того, как мы шаг за шагом проследим все ее доказательство. Математическим софизмом принято называть не менее удивительные утверждения, в доказательствах которых в отличие от доказательства парадоксов кроются незаметные, а подчас и довольно тонкие ошибки. В любой области математики - от простой арифметики до современной теоретико-множественной топологии - есть свои псевдодоказательства, свои софизмы. В лучших из них рассуждения с тщательно замаскированной ошибкой позволяют приходить к самым невероятным заключениям. Ошибкам в геометрических доказательствах Евклид посвятил целую книгу, но до наших дней она не дошла, и нам остается лишь гадать о том, какую невосполнимую утрату понесла из-за этого элементарная математика.

Семь математических софизмов, о которых пойдет речь в этой главе, выбраны из разных областей математики, каждый из них по-своему интересен. Объяснять, в чем состоит ошибочность рассуждения в каждом софизме, мы не будем, чтобы не лишать читателя удовольствия самостоятельно найти ее.

Наш первый софизм чрезвычайно элементарен. Мы предпошлем ему занимательный парадокс, на примере которого великий немецкий математик Давид Гильберт любил объяснять необычные свойства наименьшего из трансфинитных чисел «алеф-нуль». Как-то раз хозяину одной великолепной гостиницы с бесконечным, но счетным числом номеров, ни один из которых не был свободен, нужно было принять нового гостя. Хозяин вышел из положения очень просто: каждого из своих постояльцев он переселил в комнату, номер которой был на единицу больше номера прежней комнаты, в результате чего обитатель n-й комнаты переехал в (n + 1)-ю и освободил для нового гостя самую первую комнату. Как может поступить хозяин, если прибудет бесконечное множество новых гостей? Ничуть не смущаясь, хозяин переселяет всех своих прежних постояльцев в комнаты с вдвое большими номерами (гость из комнаты 1 переезжает в комнату 2, гость из комнаты 2 - в комнату 4, гость из комнаты 3 - в комнату 6, гость из комнаты 4 - в комнату 8 и т. д.) и размещает вновь прибывших в освободившихся комнатах с нечетными номерами.

Но так ли необходимо хозяину иметь счетное число комнат для того, чтобы разместить новых гостей? В приведенных ниже стишах, взятых из одного английского журнала, выходившего в прошлом веке, рассказывается о хитром хозяине гостиницы, сумевшем разместить в девяти номерах десять гостей так, что каждому из них досталось по отдельной комнате.

Их было десять чудаков,
Тех спутников усталых,
Что в дверь решили постучать
Таверны «Славный малый».

- Пусти, хозяин, ночевать,
Не будешь ты в убытке,
Нам только ночку переспать,
Промокли мы до нитки.

Хозяин тем гостям был рад,
Да вот беда некстати:
Лишь девять комнат у него
И девять лишь кроватей.

- Восьми гостям я предложу
Постели честь по чести,
А двум придется ночь проспать
В одной кровати вместе.

Лишь он сказал, и сразу крик,
От гнева красны лица:
Никто из всех десятерых
Не хочет потесниться.

Как охладить страстей тех пыл,
Умерить те волненья?
Но старый плут хозяин был
И разрешил сомненья.

Двух первых путников пока,
Чтоб не судили строго,
Просил пройти он в номер «А»
И подождать немного.

Спал третий в «Б», четвертый в «В»,
В «Г» спал всю ночь наш пятый,
В «Д», «Е», «Ж», «3» нашли ночлег
С шестого по девятый.

Потом, вернувшись снова в «А»,
Где ждали его двое,
Он ключ от «И» вручить был рад
Десятому герою.

Хоть много лет с тех пор прошло,
Неясно никому,
Как смог хозяин разместить
Гостей по одному.

Иль арифметика стара,
Иль чудо перед нами,
Понять, что, как и почему,
Вы постарайтесь сами.

Примером более тонкого математического софизма служит следующее «алгебраическое» доказательство того, что любое число а равно меньшему числу b.

Начнем с равенства

а = b + c.


Умножив обе его части на a - b, получим

а² - аb = аb + аc - b² - be.


Перенесем ас в левую часть:

а² - аb - аc = аb - b² - be

и разложим на множители:

а(а - b - c) = b(а - b - c).


Разделив обе части равенства на а - b - c, найдем

а = b,


что и требовалось доказать.

Много неприятностей подстерегает того, кто неосторожно обращается с мнимой единицей i (квадратным корнем из -1). Об этом свидетельствует хотя бы следующее удивительное «доказательство» равенства 1 = -1:

Рис. 82. Треугольник Керри.Юный математик дополнительные занятия в 5 классеЮный математик дополнительные занятия в 5 классе

В планиметрии большая часть ошибочных доказательств связана с использованием неправильных чертежей. Рассмотрим, например, удивительное «доказательство» того, что площадь лицевой стороны многоугольника, вырезанного из бумаги, отличается от площади оборотной стороны того же многоугольника. Это «доказательство» придумано врачом-психиатром Л. Восбургом Лионсом, в нем используется один любопытный принцип, открытый П. Керри.

Прежде всего начертим на листке бумаги в клетку треугольник, площадь которого равна 60 клеткам (рис. 82), и разрежем его вдоль прямых, показанных на верхнем рисунке. Перевернув части треугольника на другую сторону и составив из них треугольник, изображенный на рис. 82 в середине, мы обнаружим, что в центре нового треугольника появилась дырка площадью в 2 клетки. Иначе говоря, суммарная площадь частей исходного треугольника при переворачивании уменьшилась до 58 клеток! Перевернув еще раз (лицевой стороной вверх) лишь три части исходного треугольника, мы сможем составить из всех шести частей фигуру, изображенную на рис. 82 внизу. Ее площадь равна 59 клеткам. Что-то здесь не так, это ясно, но что именно?

Теория вероятностей изобилует правдоподобными, но логически не безупречными рассуждениями. Предположим, что вы встретились со своим другом Джоном и что каждый из вас носит тот галстук, который ваша жена подарила ему на Рождество. Вы начинаете спорить о том, чей галстук дороже, и в конце концов решаете пойти в магазин, где были куплены галстуки, и узнать, сколько стоит каждый из них. Тот, кто выиграет (чей галстук окажется дороже), по условию пари должен отдать свой галстук проигравшему, чтобы смягчить горечь поражения.

Вы рассуждаете так: «Шансы выиграть и проиграть у меня одинаковые. Выиграв, я обеднею на сумму, равную стоимости моего галстука. Проиграв, я получу более дорогой галстук. Следовательно, заключив пари, я окажусь в более выгодном положении, чем мой приятель».

Разумеется, ничто не мешает Джону рассуждать точно так же. Могут ли обе стороны, заключившие пари, иметь преимущество друг перед другом?

Один из наиболее впечатляющих парадоксов топологии заключается в том, что тор (поверхность бублика), если его поверхность растягивать (не разрывая при этом), можно вывернуть наизнанку через любую сколь угодно малую дырочку. Никакой проблемы здесь нет. Но уж если тор действительно можно вывернуть наизнанку, то следует обратить внимание и еще на один, пожалуй, даже более замечательный факт.

Юный математик дополнительные занятия в 5 классе

Рис. 83. Если тор вывернуть наизнанку, то кажется, что кольца, нарисованные на его поверхности, расцепляются.

На наружной стороне тора проведем меридиан (рис. 83, вверху). На внутренней стороне того же тора проведем параллель. Обе эти окружности, очевидно, сцеплены между собой. Вывернем теперь тор наизнанку через дырочку в его поверхности. Как видно из нижнего рисунка, первая окружность перейдет с наружной поверхности тора внутрь, а вторая - наружу, и обе окружности окажутся расцепленными! Очевидно, что это нарушает фундаментальный топологический закон, который гласит: разделить две сцепленные замкнутые кривые можно, лишь разорвав одну из кривых и протащив через место разрыва вторую.
В нашем последнем софизме, заимствованном из элементарной теории чисел, речь пойдет о сравнительных достоинствах «интересных» чисел. Разумеется, числа могут представлять интерес с различных точек зрения. Так, для Джорджа Мура, когда он писал свою знаменитую оду тридцатилетней женщине, особый интерес представляло число 30 - Мур считал, что в этом возрасте замужние женщины особенно привлекательны. Для специалиста по теории чисел число 30 представляет, по-видимому, еще больший интерес, поскольку это наибольшее из чисел, обладающих тем свойством, что все меньшие числа, не имеющие с ними общих делителей, про­сты. Число 15 873 также небезынтересно: если его умножить сна­чала на любую цифру, то есть на любое из чисел от 1 до 9, а затем на 7, то результат будет состоять из повторений выбранной для первого умножения цифры. Еще более удивительными свойствами обладает число 142 857: умножая его на числа от 1 до 6, вы будете получать циклические перестановки одних и тех же шести цифр.
Возникает вопрос: существуют ли неинтересные числа? С помощью элементарных рассуждений нетрудно доказать, что неинтересных чисел нет. Если бы скучные числа существовали, то все числа можно было бы разбить на два класса: интересные числа и неинте­ресные, скучные числа. Во множестве неинтересных чисел нашлось бы одно число, которое было бы наименьшим из всех неинтересных чисел. Но наименьшее из всех неинтересных чисел - это уже число само по себе интересное. Поэтому мы должны были бы изъять его из множества неинтересных чисел и перевести в другое множество. В оставшемся множестве в свою очередь нашлось бы наименьшее число. Повторяя этот процесс достаточно долго, можно сделать ин­тересным любое неинтересное число.

Наибольшее беспокойство доставил софизм с вывернутым наизнанку тором. Тор действительно можно вывернуть наизнанку, но это изменяет его ориентацию. В результате обе окружности меняются местами и остаются в зацеплении. Если отрезать нижнюю часть чулка и сшить концы в трубку, получится превос­ходная модель тора. На ней нитками различных цветов можно про­стегать меридиан и параллель. Такой тор легко вывернуть через дырочку в поверхности, при этом прекрасно видно все, что проис­ходит с меридианом и параллелью.

Занятие 12. Задачи с числами.

Арифметическая викторина

Цель. Развивать сообразительность, фантазию.

  1. На какое число нужно разделить 2, чтобы получить 4?

  2. Когда делимое и частное равны между собой?

  3. Может ли сумма трех последовательных натуральных чисел быть простым числом? двух? четырех?

  4. Существует ли простое число, являющиеся четным?

  5. Как с помощью одного знака неравенства можно записать, что число а больше -2, но меньше 2.

  6. Сколько га в 1 м2?

  7. За книгу заплатили 60 коп. и еще Юный математик дополнительные занятия в 5 классестоимости ее. Сколько стоила эта книга?

  8. Половина от половины числа равна половине. Какое это число?

  9. наполненные довержу водой сосуд имеет массу 5 кг, а заполненный наполовину 3 кг 250 г. Сколько воды вмещает сосуд?

  10. Сколько будет трижды сорок и пять?

  11. 2. Веселые вопросы.

  12. Когда нельзя сокращать сократимую обыкновенную дробь?

  13. Три курицы за три дня снесут три яйца. Сколько снесут 6 куриц за 6 дней? 4 курицы за 9 дней?

  14. Юный математик дополнительные занятия в 5 классечисла равняется Юный математик дополнительные занятия в 5 классе его. Какое это число?

  15. Половина - треть числа. Какое это число?

Занятие 13. Задачи - шутки.

Задача 1

Будем условно считать, что если человек не будет семь суток есть или семь суток спать, то он умрет. Пусть человек неделю не ел и не спал. Что он должен сделать в первую очередь к концу седьмых суток: поесть или поспать, чтобы остаться в живых?
(Несмотря на шутливый характер, задача имеет строгое и единственное решение).

Задача 2

Снесли вместе 7 стожков сена и 11 стожков. Сколько стожков получилось?

Задача 3

Каждую из пяти шашек передвиньте на одну клетку так, чтобы в итоге в каждом ряду, столбце и по диагоналям находилась одна шашка.
Юный математик дополнительные занятия в 5 классе

Задача 4

Задумайте число и запишите его. Удвойте его и прибавьте 1. Затем умножьте на 5 и вычтите 5. Разделите на 10. Результат запишите рядом с задуманным числом. Что получилось?

Задача 5

ВЮный математик дополнительные занятия в 5 классеставьте в кружочки на рисунке числа от 1 до 7 так, чтобы на каждой прямой сумма чисел равнялась 15. (Решение задачи не единственно).

Задача 6

На одном доме четыре дымовые трубы, на соседнем три и на следующем две. Что получается в результате?

Задача 7

Как правильно сказать: "9 и 7 будет 15" или "9 плюс 7 равно 15"?

ЗЮный математик дополнительные занятия в 5 классеадача 8

Нарисуйте этот конверт, не отрывая карандаша от бумаги.


Задача 9

ЗЮный математик дополнительные занятия в 5 классеаполните пустые клеточки на рисунке числами 2, 4, 8, 12, 16, 18 так, чтобы сумма чисел, соединенных прямыми по всем направлениям, равнялась 30. (Решение задачи не единственно).

Задача 10

Задумайте число и запишите его, умножьте на 5, прибавьте 2, умножьте на 4 и затем прибавьте 3. Теперь умножьте на 5 и прибавьте еще 7. Запишите результат. Вычеркните две последние цифры. Какое число получилось?

Задача 11

У мальчика сестер столько же, сколько и братьев. Но у каждой сестры братьев в 2 раза больше, чем сестер. Сколько всего детей в семье? Сколько из них мальчиков и сколько девочек?

Задача 12

Числа 9, 16, 23, 30, 37, 44, 51, 58, 65 необходимо расположить в магическом квадрате так, чтобы сумма чисел по каждой вертикали, горизонтали и диагонали была одинакова.

Юный математик дополнительные занятия в 5 классе

Задача 13

Как из 45 (сумма, которая составляется из сложения чисел от 1 до 9) вычесть 45, чтобы в итоге получилось... 45?
Задача 14

Электропоезд едет с востока на запад. Набрав скорость, поезд делает 60 км/ч. В том же направлении - с востока на запад - дует ветер, но со скоростью 50 км/ч. В какую сторону относит дым поезда?
Задача 15

ИЮный математик дополнительные занятия в 5 классез 12 палочек сложили 5 квадратов. Уберите две палочки так, чтобы остались только два разных по величине квадрата.


Задача 16

Предположим, что земной шар охвачен по экватору обручем, который по длине превосходит экватор на 10 метров. Допустим, что обруч на всем протяжении равно удален от поверхности земли. Как велик промежуток между поверхностью и обручем? Смогла бы, скажем, проползти под обручем муха?
Задача 17

Петя говорит другу: "Я поймал много больших рыб, а маленьких вдвое меньше. Всего у меня было 16 рыб." Верно ли это?
Задача 18

Составьте примеры с ответом 100. При этом можно пользоваться математическими знаками +, -, ×, / :
а) пять раз цифрой 1 ;
б) четыре раза цифрой 9 ;
в) пять раз цифрой 5 .
Например, "пять раз цифрой 3" : 33×3+3/3 = 100.
Задача 19

В знойный летний день, когда воздух звенит от насекомых, на зеленой лужайке площадью в три с половиной га пасутся две лошади одной породы и масти, различающиеся между собой разве только тем, что у одной хвост подвязан, а у другой - нет. Лужайка имеет форму параллелограмма, и одна из лошадей щиплет траву, передвигаясь по его диагонали, а другая - по его сторонам. Какая из этих лошадей в течение часа съест больше травы, если аппетит у них одинаков, одинаков и травяной покров лужайки, на которой они пасутся?
Задача 20

ВЮный математик дополнительные занятия в 5 классеосемь чисел 1, 2, 3, 4, 6, 7, 8, 9 необходимо так расставить по квадратикам, чтобы каждая из четырех сумм (в наружном и внутреннем квадратах, а также по диагоналям) составляла 20.


Задача 21

Мельник пришел на мельницу. В каждом из четырех углов он увидел по 3 мешка, на каждом мешке сидело по 3 кошки, а каждая кошка имела при себе троих котят. Спрашивается, много ли ног было на мельнице?
Задача 22

КЮный математик дополнительные занятия в 5 классеак можно одним мешком пшеницы, смоловши ее, наполнить два мешка, которые столь же велики, как и мешок, в котором находится пшеница?
Задача 23

ПЮный математик дополнительные занятия в 5 классеереложите одну из палочек так, чтобы равенство было верным: а)

б)


Задача 24

Двое прошли - три гвоздя нашли,
Следом четверо пройдут - много ли гвоздей найдут?
Задача 25

Летели утки: одна впреди и две позади, одна позади и две впереди, одна между двумя и три в ряд. Сколько всего летело уток?
Задача 26

Два землекопа выкапывают 2 м канавы за 2 ч. Сколько землекопов за 5 ч выкопают 5 м канавы?
Задача 27

Два отца и два сына поймали 3 зайцев, а досталось каждому по 1 зайцу. Спрашивается, как это могло случиться?
Задача 28

ВЮный математик дополнительные занятия в 5 классеставьте числа 1, 1, 2, 2, 3, 3, 4, 4, 5, 5, 6, 6, 7, 7, 8, 8 в клетки магического квадрата так, чтобы сумма в каждом ряду и столбце равнялась 18.

Задача 29

Написать цифрами число, состоящее из одиннадцати тысяч, одиннадцати сотен и одиннадцати единиц.
Задача 30

Что это такое: две ноги сидели на трех, а когда пришли четыре и утащили одну, то две ноги схватили три, бросили их в четыре, чтобы четыре оставили одну?
Задача 31

Что это может быть: две головы, две руки и шесть ног, а при ходьбе только четыре?
Задача 32
Как найти задуманное четное число?

Предложите кому-нибудь задумать четное число, затем это число утроить, полученное произведение разделить на 2, а частное опять утроить. После объявления результата арифметических действий вы называете задуманное число. Как это сделать?

Задача 1 На корабле «Пиратское счастье» несколько кошек, несколько матросов, кок и одноногий капитан. У всех них, вместе взятых, 15 голов и 41 нога. Сколько кошек было на корабле? (6 кошек)

Задача 2 Рассматривая свою коллекцию наклеек, девочка думала: « Если бы к моим наклейкам прибавить половину их да еще десяток, то у меня была бы целая сотня!» Сколько наклеек у неё было?

Ответ: Эту задачу надо решать с конца. От ста наклеек сначала отнимем десяток, получится 90. Это количество получится, если к наклейкам прибавить их половину. Всего половин в любом числе 2, а девочка хочет еще 1, значит, 3 половины количества наклеек равны 90, а одна половина это 30 наклеек. 90 - 30 = 60. У девочки было 60 наклеек. Вы можете легко проверить правильность решения.

Задача 3 У Вани пять двоек в дневнике. Он задумался и расставил такие математические знаки между двойками, что они превратились в пятерку. Как он это сделал? Ответ: (2*2*2+2):5=5

Задача 4 Два мальчика купили 8 пряников. На эту покупку первый потратил 5 рублей, а второй 3 рубля. По пути они встретили товарища и втроем съели все пряники, причем их порции были одинаковыми. При прощании встретившийся товарищ дал мальчикам 8 рублей. Как они должны разделить между собою полученные деньги?

Ответ: Присоединившийся товарищ дал 8 рублей за съеденные им 8/3 пряника, или 1/3 пряника за 1 рубль. Первый мальчик уделил товарищу 15/3 -8/3=7/3 пряника, а второй - 9/3-8/3=1/3 пряника. Следовательно, первый должен получить 7 рублей, а второй - 1 рубль.

Задача 5 Алик, Боря и Костя соревнуются, кто из них быстрее 50 раз подряд попадет в яблочко. У двоих из них счет идет уже на двузначные числа. Алик уже набрал половину того количества очков, которые набрал бы Боря, если бы Боря набрал половину количества очков, которые набрал Костя. Костя уже набрал половину того количества очков, которые набрал бы Алик, если бы Алик набрал половину от того количества очков, которые набрал бы Боря. Так кто же выигрывает?

Ответ: Пока что Алик набрал 9 очков, Боря - 16 очков, Костя - 12 очков. То есть выигрывает Боря.

Задача 6 В одном из своих самых сложных номеров школьный духовой оркестр заканчивает выступление с разным количеством музыкантов в каждом ряду. В первом ряду их вдвое меньше, чем в последнем, а во втором втрое меньше, чем в третьем. Количество музыкантов в первом и в последнем рядах вдвое больше, Чем в третьем ряду. Сколько музыкантов в каждом ряду?

Ответ: В первом ряду 4 музыканта, во втором - 3, в третьем - 6 и в последним ряду 8 музыкантов.

Задача 8 В булочную зашли три женщины. Одна из них купила поло вину того хлеба, что был на полке, и еще половину буханки. Вторая и третья женщины купили то же самое. Когда они ушли, булочная закрылась, потому что хлеб кончился. При этом продавцу не пришлось резать пополам ни одной буханки. Сколько буханок хлеба было в булочной перед приходом покупательниц?

Ответ: В булочной было 7 буханок хлеба. 4 купила первая женщина, 2 - вторая и 1 - третья.
Занятие 14-15. Старинные задачи.

Продавец продает шапку. Стоит 10 р. Подходит покупатель, меряет и согласен взять, но у него есть только 25 р. Продавец отсылает мальчика с этими 25 р. к соседке разменять. Мальчик прибегает и отдает 10+10+5. Продавец отдает шапку и сдачу в 15 руб. Через какое то время приходит соседка и и говорит, что 25 р. фальшивые, требует отдать ей деньги. Ну что делать. Продавец лезет в кассу и возвращает ей деньги.
ВОПРОС: на сколько обманули продавца?
Ответ: Рассуждаем: доходы продавца: 25р от мальчика расходы: шапка (10р) + сдача (15р) + соседка(25р) итого 25-50=-25, т.е. убыток 25р
Можно рассуждать и по другому: соседка осталась при своих деньгах (25р отдала на размен, потом 25р забрала у торговца), т.е. ее можно не учитывать. Покупатель ушел с 15р сдачи и шапкой за 10р, т.е. убыток торговца составил как раз 25р (15р сдачи + 10р шапка).
2. Это старинная народная задача. Крестьянка пришла на базар продавать яйца. Первая покупательница купила у нее половину всех яиц и еще пол-яйца. Вторая покупательница приобрела половину оставшихся яиц и еще пол-яйца. Третья купила всего одно яйцо. После этого у крестьянки не осталось ничего. Сколько яиц она принесла на базар?
Ответ: Задачу решают с конца. После того как вторая покупательница приобрела половину оставшихся яиц и еще пол-яйца, у крестьянки осталось только одно яйцо. Значит, полтора яйца составляют вторую половину того, что осталось после первой продажи. Ясно, что полный остаток составляет три яйца. Прибавив пол-яйца, получим половину того, что имелось у крестьянки первоначально. Итак, число яиц, принесенных ею на базар, семь.
3. Эта задача из книги "Арифметика" Леонтия Магницкого.
Чтобы порадовать внуков, дед купил для них орехи. Но прежде чем разрешить внукам полакомиться, дед попросил внуков поделить орехи на две части, чтобы меньшая часть, увеличенная в четыре раза, была бы равна большей части, уменьшенной в три раза. Что за части?
Ответ: 1 и 12 орехов. Также правильным ответом будет любая пара целых чисел с соотношением 12 к 1.
4. Еще одна задача из книги "Арифметика" Леонтия Магницкого.
Отец решил отдать сына в учебу и спросил учителя: "Скажи, сколько учеников у тебя в классе?" Учитель ответил: "Если придет еще учеников столько же, сколько имею, и полстолько, и четвертая часть, и твой сын, тогда будет у меня сто учеников". Сколько же учеников было в классе?
Ответ: 36 учеников.
5. В рассказе А. П. Чехова "Репетитор" гимназист Егор Зиберов не сумел решить арифметическую задачу, а отец репетируемого ученика, отставной губернский секретарь Удодов, пощелкав на счетах, получил правильный ответ. Решите и Вы эту задачу арифметически. Интересно, умеют ли решать подобные задачи современные репетиторы. Вот она.
Купец купил 138 аршин черного и синего сукна за 540 руб. Спрашивается, сколько аршин купил он и того и другого, если синее стоило 5 руб. за аршин, а черное - 3 руб.?
Ответ: Если бы купец приобрел сукно одного типа, например синее, то он заплатил бы 138*5 = 690 руб. Образовавшаяся разность в 150 руб. получена за счет того, что черное сукно повышено в цене на 2 руб. Значит, черного сукна было 150:2 = 75 аршин, а синего было 138-75 = 63 аршина.

Занятие 16. Задачи решаемые с конца

1. Магия чисел. Я задумал число, прибавил к нему 5, потом разделил сумму на 3, умножил на 4, отнял 6, разделил на 7 и получил число 2. Какое число я задумал.

2. Яблоки. Трое мальчиков имеют по некоторому количеству яблок. Первый мальчик дает другим столько яблок, сколько каждый из них имеет. Затем второй мальчик дает двум другим столько яблок, сколько каждый из них теперь имеет; в свою очередь и третий дает каждому из двух других столько, сколько есть у каждого в этот момент. После этого у каждого из мальчиков оказывается по 8 яблок. Сколько яблок было у каждого мальчика вначале?

3. Черт и бездельник. Однажды черт предложил бездельнику заработать. "Как только ты перейдешь через этот мост, - сказал он, - твои деньги удвоятся. Можешь переходить по нему сколько хочешь раз, но после каждого перехода отдавай мне за это 24 рубля". Бездельник согласился и … после третьего перехода остался без денег. Сколько денег у него было сначала?

4. Туристы. Группа туристов отправилась в поход. В первый день они прошли 1/3 пути, в второй - 1/3 остатка, в третий - 1/3 нового остатка. В результате им осталось пройти 32 км. Сколько километров был маршрут туристов?

5. Гуси. Над озерами летели гуси. На каждом озере садилась половина гусей и еще полгуся, остальные летели дальше. Все сели на семи озерах. Сколько было гусей?

6. Крестьянин и царь. Крестьянин пришел к царю и попросил: "Царь, позволь мне взять одно яблоко из твоего сада". Царь ему разрешил. Пошел крестьянин к саду и видит: весь сад огорожен тройным забором. Каждый забор имеет только одни ворота, и около каждых ворот стоит страж. Подошел крестьянин к первому стражу и сказал: "Царь разрешил мне взять одно яблоко из сада". "Возьми, но при выходе должен будешь отдать мне половину яблок, что возьмешь, и еще одно", - поставил условие страж. Это же повторили ему второй и третий, которые охраняли другие ворота. Сколько яблок должен взять крестьянин, чтобы после того, как отдаст положенные части трем стражам, у него осталось одно яблоко?

Решение задач.

1. Магия чисел. Я задумал число, прибавил к нему 5, потом разделил сумму на 3, умножил на 4, отнял 6, разделил на 7 и получил число 2. Какое число я задумал.

Решение.
Решаем задачу с конца:
1) 2 ∙ 7 = 14 - число до деления на 7.
2) (14 + 6) : 4 = 5 - число до умножения на 4.
3) 5 ∙ 3 = 15 - число до деления на 3.
4) 15 - 5 = 10 - искомое число.
Ответ: задумано число 10.

2. Яблоки. Трое мальчиков имеют по некоторому количеству яблок. Первый мальчик дает другим столько яблок, сколько каждый из них имеет. Затем второй мальчик дает двум другим столько яблок, сколько каждый из них теперь имеет; а третий дает каждому из двух других столько, сколько есть у каждого в этот момент. После этого у каждого из мальчиков оказывается по 8 яблок. Сколько яблок было у каждого мальчика вначале?

Решение.

Решаем задачу с конца с помощью таблицы.

Номер мальчика

1

2

3

Число яблок в конце

8

8

8

Число яблок до передачи их третьим мальчиком

8 : 2 = 4

8 : 2 = 4

8 + 8 + 4 = 16

Число яблок до передачи их вторым мальчиком

4 : 2 = 2

4 + 2 + 8 = 14

16 : 2 = 8

Число яблок первоначально

2 + 4 + 7 = 13

14 : 2 = 7

8 : 2 = 4

Таким образом, первоначально яблок у первого, второго и третьего мальчиков было соответственно 13, 7 и 4. Ответ: 13 яблок, 7 яблок, 4 яблока.

3. Черт и бездельник. Однажды черт предложил бездельнику заработать. "Как только ты перейдешь через этот мост, - сказал он, - твои деньги удвоятся. Можешь переходить по нему сколько хочешь раз, но после каждого перехода отдавай мне за это 24 рубля". Бездельник согласился и … после третьего перехода остался без денег. Сколько денег у него было сначала?

Решение.

Так как после третьего перехода у бездельника денег не осталось, то после перехода моста в третий раз у него было 24 рубля, а до перехода третьего моста - 12 рублей. Тогда после перехода второго моста у бездельника было 12 + 24 = 36 (рублей), а до перехода второго моста - 36 : 2 = 18 (рублей). Рассуждая аналогично, получим, что после перехода первого моста у бездельника стало 18 + 24 = 42 (рубля), а перед переходом первого моста - 42 : 2 = 21 (рубль). Таким образом, у бездельника сначала был 21 рубль.

Ответ: 21 рубль.

4. Туристы. Группа туристов отправилась в поход. В первый день они прошли 1/3 пути, в второй - 1/3 остатка, в третий - 1/3 нового остатка. В результате им осталось пройти 32 км. Сколько километров был маршрут туристов?

Решение.

Так как осталось 32 км, а в третий день туристы прошли остаток, то 32 км будут составлять последнего 2/3 остатка, тогда сам последний остаток будет равен 32 : 2/3 = 48 (км). Эти 48 км будут составлять 2/3 длины маршрута, оставшегося пройти после первого дня. Тогда весь маршрут, который осталось пройти, будет равен 48 : 2/3 = 72 (км). Эти 72 км составляют вновь 2/3, но уже всего маршрута туристов, а значит, весь маршрут будет равен 72 : 2/3 = 108 (км).

Ответ: 108 км

5. Гуси. Над озерами летели гуси. На каждом озере садилась половина гусей и еще полгуся, остальные летели дальше. Все сели на семи озерах. Сколько было гусей?

Решение.

Так как на последнем озере сели оставшиеся гуси и больше не осталось, то там сел 1 гусь. Если бы село 2, то 1 гусь остался бы еще. Тогда к шестому озеру подлетало 1 + 12∙2 = 3 гуся. А к пятому 3 + 12∙2 = 7, к четвертому 7 + 12∙2 = 15, к третьему - 15 + 12∙2 = 31, ко второму 31 +12∙2 = 63, тогда к первому подлетело 63 + 12∙2 = 127 гусей.

Ответ: 127 гусей

6. Крестьянин и царь. Крестьянин пришел к царю и попросил: "Царь, позволь мне взять одно яблоко из твоего сада". Царь ему разрешил. Пошел крестьянин к саду и видит: весь сад огорожен тройным забором. Каждый забор имеет только одни ворота, и около каждых ворот стоит страж. Подошел крестьянин к первому стражу и сказал: "Царь разрешил мне взять одно яблоко из сада". "Возьми, но при выходе должен будешь отдать мне половину яблок, что возьмешь, и еще одно", - поставил условие страж. Это же повторили ему второй и третий, которые охраняли другие ворота. Сколько яблок должен взять крестьянин, чтобы после того, как отдаст положенные части трем стражам, у него осталось одно яблоко?

Решение.

Перед последними воротами у крестьянина должно остаться (1 + 1) ∙ 2 = 4 яблока, перед вторыми - (4 + 1) ∙ 2 = 10, и перед первыми - (10 + 1) ∙ 2 = 22 яблока.

Ответ: 22 яблока.

7. Лилия на озере. На озере расцвела лилия. Каждый день число цветков удваивалось и на 20-й день все озеро покрылось цветами. За сколько дней покрылась цветами половина озера?

Решение.

Начнем с конца. Так как каждый день число цветков удваивается, а на 20-й день все озеро покрылось цветами, то половина его была покрыта цветами за один день до того, т.е. на 19-й день.

Ответ: за 19 дней.

Дополнительные задачи и задачи для самостоятельного решения.

1. Это старинная задача. Крестьянка пришла на базар продавать яйца. Первая покупательница купила у нее половину всех яиц и еще половину яйца. Вторая покупательница приобрела половину оставшихся яиц и еще половину яйца. Третья купила всего одно яйцо. После этого у крестьянки не осталось ничего. Сколько яиц она принесла на базар?

2. Задача из книги "Арифметика" Леонтия Магницкого. Отец решил отдать сына в учебу и спросил учителя: "Скажи, сколько учеников у тебя в классе?" Учитель ответил:

"Если придет еще учеников столько же, сколько имею, и полстолько, и четвертая часть, и твой сын, тогда будет у меня сто учеников". Сколько же учеников было в классе?

3. Мать купила яблоки. Два из них взяла себе, а остальные разделила между тремя своими сыновьями. Первому она дала половину всех яблок и половину яблока, второму - половина остатка и еще половину яблока, третьему - половину нового остатка и оставшуюся половину яблока. Сколько яблок купила мать, и сколько яблок получил каждый из сыновей?

4. Поставили самовар, а потом 7 раз садились пить чай и каждый раз выпивали половину имеющейся в нем воды. Оказалось, что после этого остался всего стакан воды. Сколько воды было в самоваре перед чаепитием?

5. Я задумал число, отнял 57, разделил на 2 и получил 27. Какое число я задумал?

6. На праздник купили торт. Но ели его очень интересно - к торту подходил человек и съедал половину того, что осталось. Всего торт ели 5 человек, а пришедшему последним (пятым) Стасу, отдали все, что осталось - полкило торта. Сколько весил торт в начале?

7. Некто прогулял 1/4 урока. На следующий день он прогулял половину урока. Каждый день количество прогулянных уроков увеличивалось в два раза. На десятый день он впервые прогулял все уроки. На какой день он прогулял четверть уроков, если их количество в каждый день одинаково.

8. Хулиган Леша с занятия украл много спичек. По дороге другие ребята увидели его и каждый забрал у него несколько. Вова забрал треть, Вася - треть оставшихся, Гриша - еще треть оставшихся, Толя - тоже треть оставшихся. В итоге Леша сжег 16 спичек, и у него после этого спичек не осталось. Сколько у него их было?

9. Три мальчика делили 120 фантиков. Сначала Петя дал Ване и Толе столько фантиков, сколько у них было. Затем Ваня дал Толе и Пете столько фантиков, сколько у них стало. И, наконец, Толя дал Пете и Ване столько, сколько у них к этому моменту имелось. В результате всем досталось поровну. Сколько фантиков было у каждого вначале?

Занятие 17-18. «Круги Эйлера»
Из 100 туристов, отправляющихся в заграничное путешествие, немецким языком владеют 30 человек, английским - 28, французским - 42. Английским и немецким одновременно владеют 8 человек, английским и французским - 10, немецким и французским - 5, всеми тремя языками - 3. Сколько туристов не владеют ни одним языком?

Выразим условие этой задачи графически. Обозначим кругом тех кто знает английский, другим кругом - тех, кто знает французский, и третим кругом - тех, кто знают немецкий. Тогда, например, те, кто владеет и английским и немецким, "попадут" в общую часть первого и третьего круга.

Всеми тремя языками владеют три туриста, значит, в общей части кругов вписываем число 3. Английским и французским языками владеют 10 человек, а 3 из них владеют еще и немецким. Следовательно, только английским и французским владеют 10-3=7 человек.

Аналогично получаем, что только английским и немецким владеют 8-3=5 человек, а немецким и французским 5-3=2 туриста. Вносим эти данные в соответствующие части.

Юный математик дополнительные занятия в 5 классе

Определим теперь, сколько человек владеют только одним из перечисленных языков. Немецкий знают 30 человек, но 5+3+2=10 из них владеют и другими языками, следовательно, только немецкий знают 20 человек. Аналогично получаем, что одним английским владеют 13 человек, а одним французским - 30 человек.

Юный математик дополнительные занятия в 5 классе

По условию задачи всего 100 туристов. 20+13+30+5+7+2+3=80 туристов знают хотя бы один язык, следовательно, 20 человек не владеют ни одним из данных языков.

Ответ: только английским владеет 13 человек, только французским - 30, только немецким - 20 человек. 20 человек не знают ни одного из этих языков.

Задачи на круги Эйлера

Задача 16. Про учеников школы, которые участвовали в физико-математическом конкурсе, известно, что 7 из них справились с задачами и по математике и по физике, 11 из них справились с задачами по математике, 9 из них справились с задачами по физике. Сколько учеников принимали участие в конкурсе?

Задача 17. В одной семье было много детей. 7 из них любили капусту, 6 - морковь, 5 - горох, 4 - капусту и морковь, 3 - капусту и горох, 2 - морковь и горох, один - и капусту, и морковь, и горох. Сколько детей было в семье?

Задача 18. На полу комнаты площадью 24 м2 лежат три ковра. Площадь одного из них - 10 м2, другого - 8 м2, третьего - 6 м2. Каждые два ковра перекрываются по площади 3 м2, а площадь участка пола, покрытого всеми тремя коврами, составляет 1 м2. Найдите площадь участка пола: а) покрытого первым и вторым коврами, но не покрытого третьим ковром; б) покрытого только одним первым ковром; в) не покрытого коврами.

Задача 19. На спортивные соревнования в Летней математической школе ходили 220 школьников. При этом некоторые из них участвовали в чемпионатах, а остальные были зрителями. В легкоатлетической эстафете приняли участие 30 человек, в соревнованиях по волейболу - 26, пионерболу - 32, футболу - 31, шахматам - 28 и теннису - 36 человек. 53 школьника приняли участие более чем в одном соревновании; из них 24 школьника участвовали 3 или более раз, 9 школьников - не менее 4 раз и 3 школьника - даже 5 раз (в последнюю тройку входит и один чудак, который выступал во всех шести соревнованиях). Сколько школьников были зрителями?

Разнобой

Задача 20. Дано 6 гирь: две зеленых, две красных, две синих. В каждой паре одна гиря тяжелая, а другая легкая, причём все тяжелые гири весят одинаково и все легкие тоже. Можно ли за 2 взвешивания на чашечных весах найти все тяжелые гири?

Задача 21. На плоскости расположено 11 шестерёнок, соединенных в кольцо. Могут ли все шестерёнки вращаться одновременно?

В пионерском лагере 70 ребят. Из них 27 занимаются в драмкружке, 32 поют в хоре, 22 увлекаются спортом. В драмкружке 10 ребят из хора, в хоре 6 спортсменов, в драмкружке 8 спортсменов; 3 спортсмена посещают и драмкружок и хор. Сколько ребят не поют, не увлекаются спортом, не занимаются в драмкружке? Сколько ребят заняты только спортом?

РЮный математик дополнительные занятия в 5 классеешение

Изобразим множества следующим образом:
70 - (6 + 8 + 10 + 3 + 13 + 6 + 5) = 19 - ребят не поют, не увлекаются спортом, не занимаются в драмкружке. Только спортом заняты 5 человек.

Обитаемый остров" и "Стиляги"

Некоторые ребята из нашего класса любят ходить в кино. Известно, что 15 ребят смотрели фильм «Обитаемый остров», 11 человек - фильм «Стиляги», из них 6 смотрели и «Обитаемый остров», и «Стиляги». Сколько человек смотрели только фильм «Стиляги»?
Решение

Любимые мультфильмы

Среди школьников шестого класса проводилось анкетирование по любимым мультфильмам. Самыми популярными оказались три мультфильма: «Белоснежка и семь гномов», «Губка Боб Квадратные Штаны», «Волк и теленок». Всего в классе 38 человек. «Белоснежку и семь гномов» выбрали 21 ученик, среди которых трое назвали еще «Волк и теленок», шестеро - «Губка Боб Квадратные Штаны», а один написал все три мультфильма. Мультфильм «Волк и теленок» назвали 13 ребят, среди которых пятеро выбрали сразу два мультфильма. Сколько человек выбрали мультфильм «Губка Боб Квадратные Штаны»?
Решение

«Мир музыки»

В магазин «Мир музыки» пришло 35 покупателей. Из них 20 человек купили новый диск певицы Максим, 11 - диск Земфиры, 10 человек не купили ни одного диска. Сколько человек купили диски и Максим, и Земфиры?
Решение

Гарри Поттер, Рон и Гермиона

На полке стояло 26 волшебных книг по заклинаниям, все они были прочитаны. Из них 4 прочитал и Гарри Поттер, и Рон. Гермиона прочитала 7 книг, которых не читали ни Гарри Поттер, ни Рон, и две книги, которые читал Гарри Поттер. Всего Гарри Поттер прочитал 11 книг. Сколько книг прочитал только Рон?

Занятие №19-20 Тема: Знакомство с графами.

Цель: Научиться использовать графы при решении задач.

Что такое теория графов?

Графовые задачи обладают рядом достоинств, позволяющих использовать их для развития соображения и улучшения логического мышления детей. Теория графов в настоящее время является интенсивно развивающимся разделом дискретной математики. Это объясняется тем, что в виде графовых моделей описываются многие объекты и ситуации: коммуникационные сети, схемы электрических и электронных приборов, химические молекулы и т.д.

План занятий:

1. Ознакомить с графами.

  1. Занимательные и провоцирующие задачи на смекалку.

  2. Соответствия, отношения и их описание графами.

  3. Определение графа и подграфа.

  4. Полные графы.

  5. Лемма о рукопожатиях.

  6. Связные графы и компоненты.

  7. Двудольные графы.

Ход занятий:

  1. Рисунки. Например: Пятиконечная звезда, конверт и т.д.

  2. Провоцирующие задачи обладают высоким развивающим потенциалом. Они способствуют воспитанию одного из важнейших качеств мышления, критичности, приучают к анализу воспринимаемой информации, ее разносторонней оценке, повышают интерес школьников к занятиям математикой. Представляют интерес и задачи, в которых нужно сделать простой, но неожиданный ход, выйти за рамки стандартного мышления.

  3. При решении задач с помощью графов элементы множеств обозначаются точками, установленное соответствие - сплошной линией, отсутствие соответствия - пунктирной. Полученные схемы называются графами.

  4. Графом называется множество точек, изображенных на плоскости (листе бумаги, доске), некоторые пары из которых соединены линиями. Точки называются вершинами графа, линии ребрами. Степенью вершины называется число ребер, выходящих из вершины.

Задача1: Красный, синий, желтый, и зеленый карандаши лежат в четырех коробках по одному. Цвет карандаша отличается от цвета коробки. Известно, что зеленый карандаш лежит в синей коробке, а красный не лежит в желтой. Какой коробке лежит каждый карандаш?

Решение: Обозначим точками карандаши и коробки. Сплошная линия будет обозначать, что карандаш лежит в соответствующей коробке, а пунктирная, что не лежит. Тогда с учетом задачи имеем граф:

карандаши коробки карандаши коробки

к  - - - - - - - к к  - - - - -- - -  к

Юный математик дополнительные занятия в 5 классес  - - - - - - - с с - - - - - - -- -  с

з  - - - - - - - з з - - - - - - ---  з

ж  - - - - - - - ж ж - - - - - - - -  ж

Задача2:

Задача 3: Известно, что в компании из 20 человек каждый знаком не менее чем с 10. докажите, что можно выбрать из компании 4 человек и рассадить их за круглым столом так, что каждый будет сидеть рядом со своим знакомым.

Задача 4: В праздник Оля, Катя и Наташа решили поздравить друг друга. Оля подарила Кате и Наташе по 2 подарка и один общий. Катя подарила Оле и Наташе по одному подарку и один общий. Наташа подарила Оле и Кате по 3 подарка и один общий. Сколько подарков было всего, сколько подарили каждой девочке?

Задача 5: В одной компании при встрече каждый пожал руку каждому. Всего было сделано 15 рукопожатий. Сколько человек в компании?

Число ребер m в полном графе с n вершинами равно m = .

Определение: полным графом называется такой граф, в котором каждая пара вершин соединена ребром.

Лемма: В любом графе сумма степеней вершин равна удвоенному числу ребер.

Следствие из леммы: В любом графе число вершин нечетной степени четное.

Занятие 21-22. Задачи на переливание.

1. Один человек имеет в бочонке 12 пинт вина (пинта - старинная французская мера объема, 1 пинта≈0,568 л) и хочет подарить половину вина, но у него нет сосуда в 6 пинт, однако имеются два пустых сосуда объемом 8 пинт и 5 пинт. Как с их помощью отлить ровно 6 пинт вина?

2. Имеются два пустых бидона - трехлитровый и пятилитровый. Как, пользуясь этими бидонами, набрать из реки ровно 1 л воды?

3. Как налить ровно 4 л воды, пользуясь двумя пустыми ведрами объемом 5 л и 7 л, водопроводным краном для наливания воды и раковиной для ее выливания?

4. Как и бочки с квасом налить ровно 3 л кваса, пользуясь пустыми девятилитровым ведром и пятилитровым бидоном?

Задача. Как с помощью двух пустых бидонов емкостью 17 л и 5 л отлить из молочной цистерны ровно 13 л молока?

Задача. Как, пользуясь двумя пустыми ведрами объемом 12 л и 7 л, а также водопроводным краном и раковиной, налить ровно 1 л воды?

Задача. В бидоне не менее 10 л молока. Как отлить из него ровно 6 л молока с помощью пустых девятилитрового ведра и пятилитрового бидона?

Задача. В школу привезли коробку мела, которой было 144 куска. В течение 20 дней первый класс использовал по 1 куску в 2 дня, второй класс - по 1 куску в день, а третий и четвертый классы - по 2 куска в день каждый. Сколько кусков мела осталось?

  1. Для разведения картофельного пюре быстрого приготовления "Зеленый великан" требуется 1 л воды. Как, имея два сосуда емкостью 5 и 9 литров, налить 1 литр воды из водопроводного крана?

  2. Для марш-броска по пустыне путешественнику необходимо иметь 4 литра воды. Больше он взять не может. На базе, где имеется источник воды, выдают только 5-литровые фляги, а также имеются 3-литровые банки. Как с помощью одной фляги и одной банки набрать 4 литра во флягу?

  3. В походе приготовили ведро компота. Как, имея банки, вмещающие 500г и 900г воды, отливать компот порциями по 300 г?

  4. Нефтяники пробурили скважину нефти. Необходимо доставить в лабораторию на экспертизу 6 литров нефти. В распоряжении имеется 9-литровый и 4-литровый сосуды. Как с помощью этих сосудов набрать 6 литров?

  5. Как с помощью двух бидонов емкостью 17 литров и 5 литров отлить из молочной цистерны 13 литров молока?

  6. К продавцу, стоящему у бочки с квасом, подходят два веселых приятеля и просят налить им по литру кваса каждому. Продавец замечает, что у него есть лишь две емкости в 3 л и 5 л, и поэтому он не может выполнить их просьбу. Приятели продолжают настаивать и дают продавцу 100 рублей с одним условием, что они получат свои порции одновременно. После некоторого размышления продавец сумел это сделать. Каким образом?

  7. Взгляни на берег - там ты увидишь две банки. В одну из них помещается ровно два литра воды, а в другую - три. Как налить в двухлитровую банку точно один литр? Укажи два способа.

  8. Располагая двухлитровым и пятилитровыми банками, сделай так, чтобы в одном из них оказался ровно литр воды.

  9. Возьми две стеклянные банки. В одну из них, наполненную до краёв, помещается один литр воды, а в другую - два. Как сделать так, чтобы в двухлитровой банке оказался точно один литр? Сделай это различными способами.

  10. Задача - шутка. Перед тобой двухлитровый и трёхлитровый банки, а также девятилитровая тяжелая бочка. Как бы ты не старался с помощью банок налить в нее ровно один литр воды, у тебя ничего не получится. Как думаешь, почему? Дай хотя бы один верный ответ.

  11. Поставили самовар, а потом 7 раз садились пить чай и каждый раз выпивали половину имеющейся в нем воды. Оказалось, что после этого остался всего стакан воды. Сколько воды было в самоваре перед чаепитием?

  12. Поставили самовар, а потом 7 раз садились пить чай и каждый раз выпивали половину имеющейся в нем воды и еще полстакана, после чего воды не осталось. Сколько воды было в самоваре перед чаепитием?

  13. Имеются две одинаковые чашки, одна с чаем, а другая - пустая. Из первой переливают половину имеющегося в ней чая во вторую, затем из второй переливают треть имеющегося в ней чая в первую, затем из первой переливают четверть имеющегося в ней чая во вторую и т.д. Сколько чая окажется в каждой из чашек после 100 переливаний?

  14. В два достаточно больших бидона как-то разлили 3 л воды. Из первого переливают половину имеющейся в нем воды во второй, затем из второго переливают половину имеющейся в нем воды в первый, затем из первого переливают половину имеющейся в нем воды во второй и т.д. Докажите, что независимо от того, сколько воды было сначала в каждом из сосудов, после 100 переливаний в них будет 2 л и 1 л с точностью до миллилитра.

  15. Тому Сойеру нужно покрасить забор. Он имеет 12 л краски и хочет отлить из этого количества половину, но у него нет сосуда вместимостью в 6 л. У него 2 сосуда: один - вместимостью в 8 л, а другой - вместимостью в 5 л. Каким образом налить 6 л краски в сосуд на 8 л? Какое наименьшее число переливаний необходимо при этом сделать?

  16. Две группы альпинистов готовятся к восхождению. Для приготовления еды они используют примусы, которые заправляют бензином. В альплагере имеется 10-литровая канистра бензина. Имеются еще пустые сосуды в 7 и 2 литров. Как разлить бензин в два сосуда по 5 литров в каждом?

  17. Как разделить поровну между двумя семьями 12 литров хлебного кваса, находящегося в двенадцатилитровом сосуде, воспользовавшись для этого двумя пустыми сосудами: 8-литровым и 3-литровым?

  18. Летом Винни Пух сделал запас меда на зиму и решил разделить его пополам, чтобы съесть половину до Нового Года, а другую половину - после Нового года. Весь мед находится в ведре, которое вмещает 6 литров, у него есть 2 пустые банки - 5-литровая и 1-литровая. Может ли он разделить мед так, как задумал?

  19. Белоснежка ждет в гости гномов. Зима выдалась морозной и снежной, и Белоснежка не знает наверняка, сколько гномов решатся отправиться в далекое путешествие в гости, однако знает, что их будет не более 12. В ее хозяйстве есть кастрюлька на 12 чашек, она наполнена водой, и две пустых - на 9 чашек и на 5. Можно ли приготовить кофе для любого количества гостей, если угощать каждого одной чашкой напитка?

  20. Нефтяники пробурили скважину нефти. Необходимо доставить в лабораторию на экспертизу 6 литров нефти. В распоряжении имеется 9-литровый и 4-литровый сосуды. Как с помощью этих сосудов набрать 6 литров?

  21. Бидон ёмкостью 10 л наполнен молоком. Требуется перелить из этого бидона 5 л в семилитровый бидон, используя при этом ещё один бидон, вмещающий 3 л. Как это сделать?

  22. Можно ли отмерить 8 л воды, находясь у реки и имея два ведра: одно вместимостью 15 л, другое вместимостью 16 л?

  23. Есть три бидона емкостью 14, 9 и 5 литров. В большом бидоне 14 л молока, остальные пусты. Как с помощью этих бидонов разделить молоко пополам?

  24. Имея два полных десятилитровых бидона молока и пустые четырехлитровую и пятилитровую кастрюли, отмерьте по два литра молока в каждую кастрюлю.

  25. Имеется три сосуда без делений объемами 6 л, 7 л, 8 л, кран с водой, раковина и 6л сиропа в самом маленьком сосуде. Можно ли с помощью переливаний получить 12 л смеси воды с сиропом, так чтобы в каждом сосуде воды и сиропа было поровну?

  26. Двое должны разделить поровну 8 вёдер кваса, находящегося в большом бочонке. Но у них есть ещё только два пустых бочонка, в один из которых входит 5 вёдер, а в другой - 3 ведра. Спрашивается, как они могут разделить этот квас, пользуясь только этими тремя бочонками? Решите задачу двумя способами.

  27. Как, имея пятилитровое ведро и девятилитровую банку, набрать из реки ровно три литра воды?

Занятие 23-24. Задачи на взвешивание.

Алгоритмы поиска и задачи на взвешивания

Появление подобных задач, например задачи определения фальшивой монеты, в контексте рассмотрения алгоритмов поиска не случайно. Во-первых, поиск и в этом случае осуществляется путем операций сравнения, правда, уже не только одиночных элементов, но и групп элементов между собой. Во-вторых, как будет показано ниже, в отличие от олимпиад по математике для младших школьников, задачи на взвешивания вполне можно решать конструктивно.

В качестве примера рассмотрим задачу, предлагавшуюся на теоретическом туре одной из региональных олимпиад по информатике. Пусть у нас имеется 12 монет, одна из которых фальшивая, по весу отличающаяся от остальных монет, причем неизвестно, легче она или тяжелее. Требуется за три взвешивания определить номер фальшивой монеты (попробуйте решить эту задачу самостоятельно и вы убедитесь, что это совсем не просто, а порой вообще кажется невозможным). Введем следующие обозначения. Знаком "+" будем обозначать монеты, которые во время текущего взвешивания следует положить на весы, причем, если монета на весах уже была, то на ту же самую чашу, на которой эта монета находилась во время своего предыдущего взвешивания. Знаком "-" будем обозначать монеты, которые следует переложить на противоположную чашу весов, по отношению к той, на которой они находились (каждая в отдельности), заметим, что если монета на весах еще не была, то знак "-" к ней применен быть не может. Наконец, знаком "0" - монеты, которые в очередном взвешивании не участвуют. Тогда, существует 14 различных способов пометки монет для трех взвешиваний:

1 2 3 4 5 6 7 8 9 10 11 12 13 14

+ + + + + + + + + 0 0 0 0 0 - первое взвешивание

+ + + - - - 0 0 0 + + + 0 0 - второе взвешивание

+ - 0 + - 0 + - 0 + - 0 + 0 - третье взвешивание

Из полученной таблицы вычеркнем 2 столбца так, чтобы в каждой из трех строк количество ненулевых элементов оказалось четным (ведь мы не можем во время одного взвешивания положить на две чаши весов нечетное число монет). Это могут быть, например, столбцы 4 и 14. Теперь будем взвешивать 12 монет так, как это записано в оставшихся 12 столбцах. То есть, в первом взвешивании будут участвовать 8 произвольных монет, во втором - три монеты следует с весов убрать, две - переложить на противоположные по отношению к первому взвешиванию чаши весов и три монеты положить на весы впервые (на свободные места так, чтобы на каждой из чаш вновь оказалось по 4 монеты). Согласно схеме проведем и третье взвешивание, опять располагая на каждой чаше весов по 4 монеты. Результат каждого взвешивания в отдельности никак не анализируется, а просто записывается. При этом равновесие на весах всегда кодируется нулем, впервые возникшее неравновесное состояние - знаком плюс, если при следующем взвешивании весы отклонятся от равновесия в ту же самую сторону, то результат такого взвешивания также кодируется плюсом, а если в другую сторону - то минусом. Например, записи "=<<" и "=>>" кодируются как "0++", а записи "<=>" и ">=<" - как "+0-". Так как мы не знаем, легче или тяжелее остальных монет окажется фальшивая, то нам важно как изменялось состояние весов от взвешивания к взвешиванию, а не то какая именно чаша оказывалась тяжелее, а какая легче. Поэтому два, на первый взгляд, различных результата трех взвешиваний в этом случае кодируются одинаково. После подобной записи результатов взвешиваний фальшивая монета уже фактически определена. Ею оказывается та, которой соответствует такой же столбец в таблице, как и закодированный нами результат трех взвешиваний. Для первого из примеров это монета, которая участвовала во взвешиваниях по схеме, указанной в 10-м столбце таблицы, а для второго - в 8-м. В самом деле, состояние весов в нашей задаче меняется в зависимости от того, где оказывается фальшивая монета во время каждого из взвешиваний. Поэтому монета, "поведение" которой согласуется с записанным результатом взвешиваний, такой результат и определяет.

Анализ таблицы показывает, что эту задачу можно решить не только для 12, но и для 13 монет. Для этого следует исключить из рассмотрения любой не содержащий нулей столбец, например, все тот же четвертый. В остальном все действия остаются неизменными. Для произвольного числа монет N>2 количество взвешиваний при определяется по формуле log3(2*N + 1) (за одно взвешивание задача не разрешима ни для какого количества монет!!!), но подход к решению задачи при этом не изменится.

Попробуйте теперь решить задачу, которая предлагалась в 1998 году на уже упоминавшемся выше полуфинале чемпионата мира по программированию среди студенческих команд вузов. В ней также требовалось определить номер фальшивой монеты, вес которой отличался от остальных. Но все взвешивания уже были проведены, а их результаты записаны. Число взвешиваний являлось входным параметром в задаче (оно могло быть и избыточным по сравнению с описанным выше оптимальным алгоритмом определения номера фальшивой монеты). При этом в каждом из взвешиваний могло участвовать любое четное количество имеющихся монет (сколько и какие именно - известно). Результаты записывались с помощью символов "<", ">" и "=".

Еще одна задача на взвешивания рассмотрена в [8]. В общем случае в ней требуется найти набор из минимального количества гирь такой, что с его помощью можно взвесить любой груз, весящий целое число килограмм, в диапазоне от 1 кг до N кг. При необходимости гири можно располагать на обеих чашах весов. Так, для N=40 это гири 1, 3, 9 и 27 кг.

1. На столе лежит десять пронумерованных шляп. В каждой шляпе лежит по десять золотых монет. В одной из шляп находятся фальшивые монеты. Настоящая весит 10 граммов, а поддельная только 9. В помощь даны весы со шкалой в граммах. Как определить в какой из шляп находятся фальшивые монеты, используя весы только для одного взвешивания? Весы могут взвешивать не более 750 грамм. Ответ

2. Имеется 13 монет, из них ровно одна фальшивая, причем неизвестно, легче она настоящих или тяжелее. Требуется найти эту монету за три взвешивания. Весы - стандартные для задач этого типа: две чашечки без гирь.Ответ

3. У барона Мюнхгаузена есть 8 внешне одинаковых гирек весом 1 г, 2 г, 3 г, ..., 8 г. Он помнит, какая из гирек сколько весит, но граф Склероз ему не верит. Сможет ли барон провести одно взвешивание на чашечных весах, в результате которого будет однозначно установлен вес хотя бы одной из гирь?

Ответ

4. В аптеку поступило сильнодействующее лекарство - 8 упаковок по 150 таблеток. Следом пришло сообщение, что в этой партии есть несколько упаковок с бракованными таблетками - их вес на 1 мг больше нормальной дозы. Как за одно взвешивание выявить все упаковки с бракованными таблетками? Упаковки можно вскрывать.Ответ

5. Среди 101 одинаковых по виду монет одна фальшивая, отличающаяся по весу. Как с помощью чашечных весов без гирь за два взвешивания определить, легче или тяжелее фальшивая монета? Hаходить фальшивую монету не требуется.Ответ

6. Как развесить 20 фунтов чая в 10 коробок по 2 фунта в каждой за девять развесов имея только гири на 5 и на 9 фунтов? Используются обычные весы с двумя чашами - как у статуи Правосудия :)Ответ

7. Эта история случилась давным-давно, еще во времена крестовых походов. Один из рыцарей был захвачен мусульманами в плен и предстал перед их предводителем - султаном Саладином, который объявил, что освободит пленника и его коня, если получит выкуп в 100 тысяч золотых монет. "О, великий Саладин, - обратился тогда к султану рыцарь, у которого за душой не было ни гроша, - ты лишаешь последней надежды. У меня на родине мудрому и находчивому пленнику дается шанс выйти на свободу. Если он решит заданную головоломку, его отпускают на все четыре стороны, если нет - сумма выкупа удваивается!"
"Да будет так, - ответил Саладин, и сам обожавший головоломки. - Слушай же. Тебе дадут двенадцать золотых монет и простые весы с двумя чашками, но без гирь. Одна из монет фальшивая, однако неизвестно, легче она или тяжелее настоящих. Ты должен найти ее всего за три взвешивания. Hе справишься с задачей до утра - пеняй на себя!" А вы смогли бы выкрутиться?Ответ

8. Имеется 8 с виду одинаковых монет. Одна из них фальшивая и известно, что она легче настоящей. Как с помощью всего лишь двух взвешиваний найти фальшивую монету? В Вашем распоряжении только лабораторные весы, которые показывают только больше-меньше.Ответ

9. Имеется 100 серебряных монет разных размеров и 101 золотая монета также разных размеров. Если у одной монеты размер больше, чем у другой, то она и больше весит, но это верно только для монет, сделанных из одного и того же металла. Все монеты можно легко упорядочить по размерам на глаз. Отличить золота от серебра можно тоже :-). Как за 8 взвешиваний определить, какая монета из всех 201 штук занимает по весу ровно 101-е место? Все 201 монеты также различны по весу. Весы с двумя чашками, как обычно.Ответ

10. Еще известная задача такого уровня: (Возможно это легенда, но очень уж красивая)
Во времена Второй Мировой Войны, английские ученые подбросили немецким ученым, чтобы они не решали военные проблемы, а решали головоломки, следующую логическую задачу.
Кладоискатели нашли клад и записку в которой было написано: В этих 20 мешках с золотыми монетами есть один мешок с фальшивыми монетами. Известно, что фальшивая монета в два раза тяжелее настоящей.
Задача:
Как при помощи одного взвешивания определить в каком мешке находятся фальшивые монеты?
Примечание.
Взвешиванием называется тот момент, когда весы, типа коромысла, станут горизонтально, показывая, что на правой стороне весов и на левой стороне одинаковый вес.
И еще: англичане приделали приписку к задаче, что они потратили 10 тысяч человеко-часов для решения этой задачи.Ответ

11. Три человека купили сосуд, полностью заполненный 24 унциями бальзама. Позже они приобрели три пустых сосуда объемом 5, 11 и 13 унций. Как они могли бы поделить бальзам на равные части используя эти четыре сосуда? Постарайтсь решить задачу за наименьшее количество переливаний.

Ответ

12. Имеются трёхлитровая банка сока и две пустые банки: одна - литровая, другая - двухлитровая. Как разлить сок так, чтобы во всех трёх банках было по одному литру?Ответ

13. В одном порту моряк пришел в лавку с пустым бочонком на пять галлонов и попросил лавочника налить туда четыре галлона отборного ямайского рома. К несчастью, единственным сосудом для измерения был старый оловянный кувшин на три галлона. Как лавочник сумел точно отмерить четыре галлона с помощью этих двух емкостей?Ответ

14. Имеюся 6 гирь весом 1, 2, 3, 4, 5 и 6 г. На них нанесена соответствующая маркировка. Однако есть основания считать, что при маркировке гирь допущена одна ошибка. Как при помощи двух взвешиваний на чашечных весах, на которых можно сравнить веса любых групп гирь, определить, верна ли имеющаяся на гирях маркировка?Ответ

15. Имеется 9 одинаковых монет, одна из которых фальшивая и по этой причине легче остальных. Мы располагаем двумя весами без гирь, позволяющими сравнивать по весу любые группы монет. Однако одни из имеющихся весов являются грубыми, на них нельзя отличить фальшивую монету от настоящей. Их точность не позволяет уловить разницу в весе. Зато другие весы точные. Но какие весы грубые, а какие точные - неизвестно. Как в этой ситуации с помощью трех взвешиваний определить фальшивую монету?

Занятие 25-26. Задачи на движение.

Решение задач на движение двух тел в противоположных направлениях.

Задача 1. Одновременно из одного пункта в противоположных направлениях вышли два пешехода. Один из них шёл со скоростью 6 км/ч, а другой 4 км/ч. Какое расстояние будет между ними через 3 часа?

4 км/ч 6 км/ч

3 ч 3 ч

Решение

Скорость

Время

Расстояние

1 пешеход

6 км/ч

3 ч.

? ?

2 пешеход

4 км/ч

3 ч.

?

1 способ: (6+4)*3=30 (км)

2 способ: 6*3+4*3=30 (км)

Ответ: 30 км.

Задача 2. Одновременно из одного пункта в противоположных направлениях вышли два пешехода. Один из них шёл со скоростью 6 км/ч, а другой 4 км/ч. Через сколько времени пешеходы удалятся друг от друга на 30 км? 30 км

4 км/ч 6 км/ч

Решение

Скорость

Время

Расстояние

1 пешеход

6 км/ч

x ч

6х 30 км

2 пешеход

4 км//ч

х ч

1 способ: 6х+4х=30

х=3 Пешеходы удалятся друг от друга на 30 км через 3 часа.

2 способ: (6+4)*х=30

х=3 Ответ: 3 часа.

Задача 3. Одновременно из одного пункта в противоположных направлениях вышли два пешехода. Один из них шёл со скоростью 6 км/ч. Через 3 часа пешеходы удалились друг от друга на 30 км. Определите скорость другого пешехода.

30 км

х км/ч 6 км/ч

3 ч 3 ч

Решение

(До заполнения таблицы выясняем, что обозначаем через х: то, о чём спрашивается в вопросе задачи). Заполнив 2 столбика, опять проговариваем фразу: «Третий столбик заполняем, глядя на первые два. Третий столбик нам даёт уравнение.»

Скорость

Время

Расстояние

1 пешеход

6 км/ч

3 ч

6*3 км 30 км

2 пешеход

х км/ч

3 ч

3х км

1 способ: 6*3+3х=30

х=4

2 способ: (без помощи уравнения)

(30-18):3=4

Ответ: 4 км/ч.

Решить самостоятельно задачу 4. По данным таблицы составьте задачи на движение двух тел в противоположных направлениях при одновременном начале движения из одного пункта. Найдите неизвестные величины.

Задача

1

2

3

4

Движущиеся тела

Велосипедисты

Лыжники

Катера

Поезда

Скорость первого тела

Скорость второго тела

Общее время движения

Путь, пройденный первым телом

Путь, пройденный вторым телом

Тела удалились на

20 км/ч

23 км/ч

2 ч

?

?

?

12 км/ч

9 км/ч

х ч

12х км

9х км

12х+9х=63

х км/ч

14 км/ч

5 ч

?

?

120 км

50 км/ч

х км/ч

6 ч

?

?

540 км

В следующих заданиях составить уравнение и решать задачу.

Задача 5. Из одного и того же пункта одновременно в противоположных направлениях вышли два пешехода. Через 2 часа расстояние между ними стало 16 км. Найдите скорость второго пешехода, если скорость первого была 5 км/ч. (ответ: 10+2х=16; 3 км/ч)

Задача 6. Из одного и того же пункта одновременно в противоположных направлениях вышли два пешехода. Через 3 часа расстояние между ними стало 27 км. Найдите скорость второго пешехода, если скорость первого была 4 км/ч. (ответ: 12+3х=27; 5км/ч)

Задача 7. Из одного и того же пункта в противоположных направлениях выехали одновременно две автомашины. Скорость одной из них 55 км/ч, скорость другой - 65км/ч. Через сколько часов расстояние между ними будет 600 км? (ответ: 55х+65х=600; 5ч.)

Решение задач на встречное движение двух тел.

Задача 1. Одновременно из двух пунктов навстречу друг другу вышли два пешехода. Через 3 часа они встретились. Какое расстояние до встречи прошёл каждый пешеход и какое расстояние было между пунктами, если один пешеход шёл со скоростью 6 км/ч, а другой - со скоростью 4 км/ч?

? км

4 км/ч 6 км/ч

3 ч 3ч

Решение

Скорость

Время

Расстояние

1 пешеход

4 км/ч

3 ч

?

2 пешеход

6 км/ч

3 ч

?

Ответ: 12 км; 18 км; 30 км.

Задача 2. Из двух пунктов, находящихся на расстоянии 30 км, одновременно навстречу друг другу вышли два пешехода. Один из них проходит в час 6 км, а другой 4 км. Через сколько часов пешеходы встретятся и какое расстояние пройдёт каждый из них до встречи.

30 км

? км 7 км

4 км/ч 6 км/ч

х ч х ч

Решение:

Скорость

Время

Расстояние

1 пешеход

4 км/ч

х ч

4х км 30

2 пешеход

6 км/ч

х ч

6х км км

Ответ: 3 ч; 12 км;18 км

Задача3. Из двух пунктов, расстояние между которыми 30 км, одновременно навстречу друг другу вышли два пешехода. Через 3 ч пешеходы встретились. Скорость одного пешехода 4 км/ч. Найдите скорость другого.

Решение. 30 км

4 км/ч х км/ч

3 ч 3ч

Скорость

Время

Расстояние

1 пешеход

4 км/ч

3 ч

4*3 км 30 км

2 пешеход

х км/ч

3 ч

3х км

4*3+3х=30 3х=30-12 х=6 (км/ч) Ответ: 6 км/ч

Решать самостоятельно задачу 4: по данным таблицы составьте задачи на встречное движение двух тел при одновременном начале движения из двух пунктов. Найдите неизвестные величины.

Задача

1

2

3

4

Движущиеся тела

Бегуны

Лодки

Катера

Поезда

Скорость первого тела

Скорость второго тела

Время движения до встречи

Расстояние между пунктами

Путь, пройденный первым телом

Путь, пройденный вторым телом

Уравнение

7 м/с

8 м/с

х с

120 м

7х м

8х м

7х+8х=120

12 км/ч

9 км/ч

х ч

84 км

?

?

?

15 км/ч

х км/ч

5 ч

160 км

15*5 км

х*5

?

Х км/ч

47 км/ч

4 ч

360 км

?

?

?

В следующих заданиях составить уравнения и решить задачу.

Задача 5. Из двух городов навстречу друг другу вышли одновременно два поезда. Скорость одного из них 70 км/ч, скорость другого - 80 км/ч. Через сколько часов они встретятся, если расстояние между городами 900 км? (ответ: 70х+80х=900; 5ч.)

Задача 6. Из двух городов, расстояние между которыми 162 км, одновременно навстречу друг другу выехали два велосипедиста. Скорость одного на 3 км/ч больше скорости другого. Встреча произошла через 6ч после их выезда. С какой скоростью ехал каждый велосипедист? (ответ: 6х+6(х+3)=162;12км/ч)

Задача 7. Из городов А и В, расстояние между которыми 240 км, одновременно навстречу друг другу выехали два поезда. Встретились они через 2,4 часа. Скорость одного поезда больше скорости другого на 10 км/ч. Найдите скорость каждого поезда.

(ответ: 2,4(х+10)+2,4х=420; 82,5 км/ч; 92,5 км/ч)

Решение задач на движение двух тел в одном направлении.

Задача 1. Одновременно из одного пункта в одном направлении вышли два пешехода. Первый пешеход идёт со скоростью 6 км/ч, а другой - со скоростью 4 км/ч. Какое расстояние будет между пешеходами через 5 часов?

Решение: ? км

4 км/ч 5 ч. 6 км/ч

Скорость

Время

Расстояние

1 пешеход

6 км/ч

5 ч

? 6*5 км 30

2 пешеход

4 км/ч

5 ч

? 4*5 км км

6*5=30 (км) - прошёл первый пешеход 4*5=20 (км) - прошёл второй пешеход

30-20=10 (км) - расстояние между пешеходами через 5 часов. Ответ: 10 км.

Задача 2. Одновременно из одного пункта в одном направлении вышли два пешехода. Первый пешеход идёт со скоростью 6 км/ч, а второй - со скоростью 4км/ч. Через сколько часов второй пешеход отстанет от первого на 10 км?

Решение: х ч 4 км/ч

10 км

х ч 6 км/ч

Скорость

Время

Расстояние

1 пешеход

6 км/ч

х ч

6х км, на 10>,чем

2 пешеход

4 км/ч

х ч

4х км

Составив таблицу, выясняем, что это задача на сравнение и уравнение составляем, проговорим фразу: «из большего отнимаем меньшее, получаем разницу».

6х-4х=10 2х=10 х=5 Ответ: второй пешеход отстанет от первого на 10 км через 5 часов.

Задача 3. Одновременно из одного пункта в одном направлении вышли два пешехода. Скорость первого пешехода 6 км/ч. Через 5 ч второй пешеход отстал от первого на 10 км. С какой скоростью шёл второй пешеход? Решение: 5 ч 10 км

х км/ ч

5 ч 6км/ч

Скорость

Время

Расстояние

1 пешеход

6 км/ч

5 ч

6*5 км, на 10>,чем

2 пешеход

х км/ч

5 ч

5х км

Задача на сравнение: 5*6-5х=10 …………… х=4

Ответ: второй пешеход шёл со скоростью 4 км/ч

Задача 4. Одновременно из двух пунктов вышли два пешехода. Первый пешеход, идущий со скоростью 6 км/ч, через 5 ч догнал второго, идущего со скоростью 4 км/ч. Какое расстояние между пешеходами было первоначально? Решение: 5 ч.

6 км/ч 4 км/ч

х км

Скорость

Время

Расстояние

1 пешеход

6 км/ч

5 ч

? 6*5 км

2 пешеход

4 км/ч

5 ч

? 4*5 км

6*5=30 (км) - прошёл первый пешеход 4*5=20 (км) - прошёл второй пешеход

30-20-=10 (км) - первоначальное расстояние между пешеходами. Ответ: 10 км.

Задача 5. Первый пешеход, идущий со скоростью 6 км/ч, догоняет второго, идущего со скоростью 4 км/ч. Через сколько часов первый пешеход догонит второго, если первоначально расстояние между ними было 10 км и они вышли одновременно?

Решение: 6 км/ч 4 км/ч

10 км х ч

Скорость

Время

Расстояние

1 пешеход

6 км/ч

х ч

6х, на 10км>, чем

2 пешеход

4 км/ч

х ч

6х-4х=10 2х=10 х=5 Ответ: первый пешеход догонит второго через 5ч.

В следующих заданиях составить уравнение и решить задачу.

Задача 6. Из двух пунктов в одном направлении выехали два велосипедиста. Скорость одного из них 11 км/ч, а скорость другого - 13 км/ч. Через сколько часов первый велосипедист догонит второго, если расстояние между пунктам 12 км?(Ответ: 13х-11х=12; 6 км/ч)

Задача 7. Из Саратова в Москву вышел пассажирский поезд со скоростью 55 км/ч, а через 2 часа вслед за ним отправился скорый поезд со скоростью 66 км/ч. На каком расстоянии от Москвы второй поезд догонит первый, если расстояние от Саратова до Москвы 855 км?(Ответ: 66х=55(х+2); 195км)

Задача 8. Со станции вышел поезд, скорость которого 48 км/ч, а через 1,25 ч за ним вышел второй поезд, скорость которого 56 км/ч. На каком расстоянии от станции отправления второй поезд догонит первый?(Ответ: 48(х+1,25)=56х; 420 км)

Задача 9. Из одного пункта в одном направлении одновременно выехали автомобилист и мотоциклист. Скорость автомобиля 63 км/ч, скорость мотоцикла 48 км/ч. Через сколько часов расстояние между ними будет равно 75 км?(Ответ: 63х-48х=75: 5 ч)

Задачи на движение по водоёму.

Ученик с 5 класса должен знать:

Скорость по течению равна сумме собственной скорости и скорости течения реки.

Скорость против течения равна разности собственной скорости и скорости течения реки.

Скорость по озеру равна собственной скорости.

Собственная скорость равна половине суммы скорости по течению и скорости против течения.

Краткая запись всех задач оформляется, как, обычно, в таблицу. В начале изучения таких задач выясняем, что, когда плывём по течению, течение нам помогает плыть, поэтому мы к своей скорости прибавляем скорость течения, против когда плывём против течения, течение нам мешает плыть, поэтому мы из своей скорости вычитаем скорость течения. У основной массы класса такие задачи не вызывают затруднений, поэтому, подробное решение и оформление таких задач не будем. Как обычно, два столбика заполняем по условию задачи, третий по первым двум. И этот столбик нам даёт уравнение. Дальше смотрим, к какому типу относится задача: на сравнение или на сложение величин, если это необходимо.

Задача 1. Катер прошёл 20 км по течению реки и такой же путь обратно, затратив на весь путь 1 ч 45 мин. Скорость течения реки равна 2 км/ч. Найдите время катера в пути.

Пусть х км/ч - собственная скорость катера. Какое из уравнений соответствует условию задачи.

20/(х+2)=1,45 20/(х-2)-20/(х+2)=1,45 20/(х-2)+20/(х+2)=7/4 20/(2-х)+20(2+х)=7/4

Решение:

Скорость

Время

Путь

По течению

х+2 км/ч

20/(х+2) ч 1 ч

45 мин

20 км

Против течения

х-2 км/ч

20/(х-2) ч

20 км

Эта задача на сложение величин. Переводим минуты в часы, 1 ч 45 мин.=7/4 ч., получаем уравнение:

20/(х+2)+20(х-2)=7/4. Ответ: 3

Задача 2. Катер прошёл 3 км по течению реки на 30 минут быстрее, чем 8 км против течения реки. Собственная скорость катера 15 км/ч.

Пусть х км/ч - скорость течения реки. Какое из уравнений соответствует условию задачи?

Решение:

Скорость

Время

Путь

По течению

х+15 км/ч

3/(15+х) ч, на 30 мин.< чем

3 км

Против течения

15-х км/ч

8/(15-х) ч

8 км

Эта задача на сравнение, из большего отнимаем меньшее, получаем разницу, так как 30 мин это 0,5 ч , то получаем:8/(15-х)-3/(15+х)=0,5 Ответ: 2

В следующих заданиях составить уравнение.

Задача 3. Катер прошёл 30 км по течению реки и 13 км против течения, затратив на весь путь 1 ч 30 мин. Какова собственная скорость катера, если скорость течения реки равна 2 км/ч?

(Ответ: 30/(х+2)+13/(х-2)=1,5)

Задача 4. Туристы проплыли на байдарке против течения реки 6 км и вернулись обратно. На все путешествие они затратили 4 ч 30 мин. Какова собственная скорость байдарки, если скорость течения реки 1 км/ч? (Ответ:6(х+1)+6(х-1)=4,5)

Задача 5. Моторная лодка шла 0,4 ч по озеру и 0,3 ч по течению реки, скорость течения которой 2 км/ч. Всего моторная лодка прошла 9 км. Найдите её собственную скорость.

(Ответ: 0,4(х+2)+0,4(х-2)=9)

Задача 6. Катер прошёл 0,6 ч против течения реки, скорость течения которой 2,5 км/ч, и 0,4 ч по озеру. Всего катер прошёл 17 км. Найдите собственную скорость катера.(Ответ: 0,6(х-2,5)+0,4х=17)

Занятие 27. Задачи на разрезание.

а) Разрежьте произвольный треугольник на несколько кусочков так, чтобы из них можно было сложить прямоугольник.
б) Разрежьте произвольный прямоугольник на несколько кусочков так, чтобы из них можно было сложить квадрат.
в) Разрежьте два произвольных квадрата на несколько кусочков так, чтобы из них можно было сложить один большой квадрат.

Юный математик дополнительные занятия в 5 классе Подсказка 1

б) Сначала составьте из произвольного прямоугольника такой прямоугольник, отношение большей стороны которого к меньшей не превышает четырех.

в) Используйте теорему Пифагора.

Юный математик дополнительные занятия в 5 классе Подсказка 2

а) Проведите высоту или среднюю линию.

б) Наложите прямоугольник на квадрат, который должен получиться, и проведите «диагональ».

в) Приложите квадраты друг к другу, на стороне большего квадрата отмерьте отрезок, равный длине меньшего квадрата, после чего соедините ее с «противоположными» вершинами каждого из квадратов (см. рис. 1).

Юный математик дополнительные занятия в 5 классе

Рис. 1

Юный математик дополнительные занятия в 5 классе Решение

Юный математик дополнительные занятия в 5 классе

Рис. 2


а) Пусть дан произвольный треугольник ABC. Проведём среднюю линию MN параллельно стороне AB, а в полученном треугольнике CMNопустим высоту CD. Кроме того, опустим на прямую MN перпендикулярыAK и BL. Тогда легко видеть, что ∆AKM = ∆CDM и ∆BLN = ∆CDN как прямоугольные треугольники, у которых равны соответствующие пара сторон и пара углов.

Отсюда вытекает метод разрезания данного треугольника и последующего перекладывания кусочков. Именно, проведём разрезы по отрезкамMN и CD. После этого переложим треугольники CDM и CDN на место треугольников AKM и BLN соответственно, как показано на рис. 2. Мы получили прямоугольник AKLB, как того и требовалось в задаче.

Отметим, что этот метод не сработает, если один из углов CAB или CBA - тупой. Так происходит из-за того, что в этом случае высота CD не лежит внутри треугольника CMN. Но это не слишком страшно: если проводить среднюю линию параллельно самой длинной стороне исходного треугольника, то в отсечённом треугольнике мы будем опускать высоту из тупого угла, а она обязательно будет лежать внутри треугольника.

б) Пусть дан прямоугольник ABCD, стороны которого AD и AB равны a и b соответственно, причём a > b. Тогда площадь того квадрата, который мы хотим получить в итоге, должна быть равной ab. Следовательно, длина стороны квадрата составляет √ab, что меньше, чем AD, но больше, чем AB.

Юный математик дополнительные занятия в 5 классе

Рис. 3


Построим квадрат APQR, равный искомому, таким образом, чтобы точка Bлежала на отрезке AP, а точка R - на отрезке AD. Пусть PD пересекает отрезки BC и QR в точках M и N соответственно. Тогда легко видеть, что треугольники PBM, PAD и NRD подобны, а кроме того, BP = (√ab - b) иRD = (a - √ab). Значит,

Юный математик дополнительные занятия в 5 классе

Юный математик дополнительные занятия в 5 классе

Следовательно, ∆PBM = ∆NRD по двум сторонам и углу между ними. Также отсюда несложно вывести равенства PQ = MC и NQ = CD, а значит, ∆PQN = ∆MCD тоже по двум сторонам и углу между ними.

Из всех приведённых рассуждений вытекает метод разрезания. Именно, сначала мы откладываем на сторонах AD и BC отрезки AR и CM, длины которых равны √ab (о том, как строить отрезки вида √ab, см. задачу «Правильные многоугольники» - врезку в разделе «Решение»). Далее, восстанавливаем перпендикуляр к отрезку AD в точке R. Теперь осталось только отрезать треугольники MCD иNRD и переложить их так, как показано на рис. 3.

Юный математик дополнительные занятия в 5 классе

Рис. 4


Отметим, что для того, чтобы этим методом можно было воспользоваться, требуется, чтобы точка M оказалась внутри отрезка BK (иначе не весь треугольник NRD содержится внутри прямоугольника ABCD). То есть необходимо, чтобы

Юный математик дополнительные занятия в 5 классе

Если это условие не выполняется, то сначала нужно сделать данный прямоугольник более широким и менее длинным. Для этого достаточно разрезать его пополам и переложить кусочки так, как показано на рис. 4. Ясно, что после проведения такой операции отношение большей стороны к меньшей уменьшится в четыре раза. А значит, проделывая её достаточно большое число раз, в конце концов мы получим прямоугольник, к которому применимо разрезание с рис. 3.

в) Рассмотрим два данных квадрата ABCD и DPQR, приложив их друг к другу так, чтобы они пересекались по стороне CDменьшего квадрата и имели общую вершину D. Будем считать, что PD = a и AB = b, причём, как мы уже отмечали, a > b. Тогда на стороне DR большего квадрата можно рассмотреть такую точку M, что MR = AB. По теореме Пифагора Юный математик дополнительные занятия в 5 классе.

Юный математик дополнительные занятия в 5 классе

Рис. 5


Пусть прямые, проходящие через точки B и Q параллельно прямымMQ и BM соответственно, пересекаются в точке N. Тогда четырёхугольникBMQN является параллелограммом, а так как у него все стороны равны, то это ромб. Но ∆BAM = ∆MRQ по трём сторонам, откуда следует (учитывая, что углы BAM и MRQ прямые), что Юный математик дополнительные занятия в 5 классе. Таким образом,BMQN - квадрат. А так как его площадь равна (a2 + b2), то это именно тот квадрат, который нам надо получить.

Для того чтобы перейти к разрезанию, осталось заметить, что ∆BAM = ∆MRQ = ∆BCN = ∆NPQ. После этого то, что нужно сделать, становится очевидным: необходимо отрезать треугольники BAM и MRQ и переложить их так, как изображено на рис. 5.

Юный математик дополнительные занятия в 5 классе Послесловие

Прорешав предложенные задачи, читатель, вполне возможно, задумается над таким вопросом: а когда вообще можно один данный многоугольник разрезать прямыми линиями на конечное число таких кусочков, из которых складывается другой данный многоугольник? Немножко поразмыслив, он поймёт, что как минимум необходимо, чтобы площади этих многоугольников были равны. Таким образом, исходный вопрос превращается в следующий: правда ли, что если два многоугольника имеют одинаковую площадь, то один из них можно разрезать на кусочки, из которых складывается второй (это свойство двух многоугольников называется равносоставленностью)? Оказывается, это действительно так, и об этом нам говорит теорема Бойяи-Гервина, доказанная в 30-х годах XIX века. Более точно, её формулировка заключается вот в чём.

Теорема Бойяи-Гервина. Два многоугольника равновелики тогда и только тогда, когда они равносоставлены.

Идея доказательства этого замечательного результата заключается в следующем. Во-первых, мы будем доказывать не само утверждение теоремы, а то, что каждый из двух данных равновеликих многоугольников можно разрезать на кусочки, из которых складывается квадрат той же площади. Для этого сначала мы разобьём каждый из многоугольников на треугольники (такое разбиение называется триангуляцией). А потом каждый треугольничек превратим в квадратик (например, при помощи метода, описанного в пунктах а) и б) настоящей задачи). Осталось сложить из большого количества маленьких квадратиков один большой - это мы умеем делать благодаря пункту в).

Аналогичный вопрос для многогранников составляет одну из знаменитых проблем Давида Гильберта (третью), представленных им в докладе на II Международном конгрессе математиков в Париже в 1900 году. Характерно, что ответ на него оказался отрицательным. Уже рассмотрение двух таких простейших многогранников, как куб и правильный тетраэдр, показывает, что ни один из них не получается разрезать на конечное число частей так, чтобы из них составлялся другой. И это не случайно - подобного разрезания просто не существует.

Решение третьей проблемы Гильберта было получено одним из его учеников - Максом Деном - уже в 1901 году. Ден обнаружил инвариантную величину, которая не изменялась при разрезании многогранников на кусочки и складывании из них новых фигур. Однако эта величина оказалась различной для некоторых многогранников (в частности, куба и правильного тетраэдра). Последнее обстоятельство явно указывает на тот факт, что эти многогранники равносоставленными не являются.

Занятие 28-29. Задачи со спичками.

Головоломки - задачи, для решения которых, как правило, требуется сообразительность, а не специальные знания высокого уровня. Разгадывание головоломок является одним из любимых занятий большинства ценителей интеллектуального досуга. Среди головоломок представленных в разделе Вы сможете найти занимательные головоломки на разрезание фигур, головоломки со спичками, зрительные головоломки на распознавание скрытых образов и много другое.

Юный математик дополнительные занятия в 5 классе

Юный математик дополнительные занятия в 5 классе

Четыре квадрата

Категория: Головоломки
со спичками
Разгадать »»

Юный математик дополнительные занятия в 5 классе

Ключ

Категория: Головоломки
со спичками
Разгадать »»

Юный математик дополнительные занятия в 5 классе

Два квадрата

Категория: Головоломки
со спичками
Разгадать »»

Юный математик дополнительные занятия в 5 классе

Оливка в бокале

Категория: Головоломки
со спичками
Разгадать »»

Юный математик дополнительные занятия в 5 классе

Домик из рюмок

Категория: Головоломки
со спичками
Разгадать »»

Юный математик дополнительные занятия в 5 классе

Три квадрата

Категория: Головоломки
со спичками
Разгадать »»

Юный математик дополнительные занятия в 5 классе

Пять квадратов

Категория: Головоломки
со спичками
Разгадать »»

Юный математик дополнительные занятия в 5 классе

Рыбка

Категория: Головоломки
со спичками
Разгадать »»

Юный математик дополнительные занятия в 5 классе

Ползущий жук

Категория: Головоломки
со спичками
Разгадать »»

Юный математик дополнительные занятия в 5 классе

Змейка

Категория: Головоломки
со спичками
Разгадать »»

Юный математик дополнительные занятия в 5 классе

Три квадрата

Категория: Головоломки
со спичками
Разгадать »»

Юный математик дополнительные занятия в 5 классе

Пять квадратов

Категория: Головоломки
со спичками
Разгадать »»

Занятие 30-31. Геометрические головоломки.

Цель. Развивать сообразительность, логическое мышление, пространственное воображение.

  1. Прямая MN лежит внутри угла АВС, который больше нулевого угла, но не больше полного. Какай это угол?

  2. Можно ли из проволоки, длина которой 20 см, согнуть такой треугольник, одна сторона которого была бы равна 8 см? 10 см? 12 см?

  3. Одна сторона равнобедренного треугольника равна 20 см, а другая равна Юный математик дополнительные занятия в 5 классе третье. Чему равен периметр этого треугольника?

  4. Все высоты данного треугольника пересекаются в одной из его вершин. Какой это треугольник?

  5. Имеются 13 равных квадратов. Как составить из них два квадрата?

  6. Листочек бумаги надо разрезать на 8 частей, ограниченных отрезками. Сколько разрезов нужно для этого сделать?

Представьте себе что перед вами шахматная доска 26*26. Выберите 2 клетки таким образом:

1) первая клетка автоматичеки зачеркивает все клетки этого цвета по вертикали и горизантали.
2) одну и туже клетку нельзя использовать два раза.
3) зачеркнутые клетки нельзя использовать.

  1. Сколько вариантов выбрать клетки?

Эрудит решил у себя в саду посадить 10 деревьев. А миссис Брэйн требует разместить деревья в саду так, чтобы получилось 5 рядов и в каждом ряду по 4 дерева.

  1. Как Эрудиту расположить деревья?

Занятие 32-35. Комбинаторные задачи.

Имеются три слова "ДРУЖБА", "ДЕЛО", "ЛЮБИТ" (нарезать листочки с этими словами - по 7 карточек на каждое слово). Сколькими способами из этих слов можно составить фразу?

Учащиеся предлагают варианты, эти варианты составляют на доске.

Ответ: 6 способов.

Учитель: Как вы думаете, какой вариант является верным с точки зрения русского языка? (Дружба любит дело). Как вы понимаете это высказывание?

Учитель: Здесь был приведен полный перебор всех возможных вариантов, или, как обычно говорят, всех возможных комбинаций. Поэтому это комбинаторная задача. Давайте подумаем, как можно записать, оформить решение этой задачи.

1 способ. Обозначим предложенные слова заглавными буквами:

ДРУЖБА - Д ЛЮБИТ - Л ДЕЛО - Е (возьмем вторую букву этого слова)

Тогда все названные вами способы можно просто перечислить: ДЛЕ, ДЕЛ, ЛДЕ, ЛЕД, ЕДЛ, ЕЛД.

2 способ.

Оказывается, решение можно оформить в виде модели, которую называют деревом возможных вариантов. Она, во-первых, наглядна, как всякая картинка, и, во-вторых, позволяет все учесть, ничего не пропустив,

Учащиеся под руководством учителя составляют схему:

Юный математик дополнительные занятия в 5 классе

Способ 3 (рассуждение)

На первом месте может стоять одно из трех слов: ДРУЖБА, ЛЮБИТ, ДЕЛО. Если первое слово выбрано, то на втором месте может стоять одно из двух оставшихся слов, а на третьем месте - только одно оставшееся слово. Значит, всего вариантов: Юный математик дополнительные занятия в 5 классе.

Заметим, что последний прием называется правилом умножения.

У каждого из этих трех способов есть свои преимущества и свои недостатки (обсудить) Выбор решения - за вами! Отметим все же, что правило умножения позволяет в один шаг решать самые разнообразные задачи.

Задача 2.

У Ани 3 подруги, и она каждой из них купила по шоколадке и хочет подарить их к празднику. Сколькими способами она может это сделать?

Решение: Решение выполняют на доске ученики (решение выполняется 3 способами)

Задача 3.

В компании друзей - 6 человек: Андрей, Борис, Витя, Гриша, Дима, Егор. В школьной столовой за столом 6 стульев. Друзья решили каждый день, завтракая, рассаживаться на эти 6 стульев по-разному. Сколько раз они смогут это сделать без повторений?

Решение:

Учитель: Какой способ мы выберем? (Учащиеся под руководством учителя должны придти к выводу, что это третий способ - правило умножения).

Решение оформляет на доске ученик.

Для удобства рассуждений будем считать, что друзья усаживаются за стол поочередно. Будем считать, что первой усаживается за стол Андрей. У него 6 вариантов выбора стула. Вторым усаживается Борис, и независимо выбирает стул из 5 оставшихся. Витя делает свой выбор третьим и на выбор у него будет 4 стула. У Гриши будет уже 3 варианта, у Димы - 2, у Егора - 1. По правилу умножения получаем:

Юный математик дополнительные занятия в 5 классе.

Ответ - 720 дней или почти 2 года.

Учитель: Как мы видим, условия задач разные, а решения, по сути дела, одинаковы. Удобно, поэтому ввести и одинаковые обозначения для этих ответов.

Определение: произведение всех натуральных чисел от 1 до п включительно называется п - факториал и обозначается символом п!

п! = Юный математик дополнительные занятия в 5 классе

Знак п! читается "Эн факториал", что в дословном переводе с английского языка обозначает "состоящий из п множителей". Отметим важную особенность этой величины - ее быстрый рост.

Задача 4.

Вычислите:

а) 1!; б) 2!; в) 3!; г) 4!; д) 5!; е)10!

Считают, что 0! =1 (записать)

Задача 5.

Учитель: ДРУЖБА - одно из важнейших богатств, которое может быть у человека. Недаром о дружбе слагаются стихи и песни, сочиняют пословицы и поговорки. Какие пословицы и поговорки о дружбе вы знаете?

Друзья познаются в беде.
Не имей сто рублей, а имей сто друзей.
Один в поле не воин.
Сам погибай, а товарища выручай.
Старый друг лучше новых двух.
Без друга в жизни туго.

- Молодцы! Для каждого человека очень важно, чтобы у него были хорошие, настоящие друзья. Давайте решим несколько примеров с применением нового понятия - факториал, и узнаем новую пословицу о дружбе.

7!+Юный математик дополнительные занятия в 5 классе

8! - (13 - 5)2

6! - 5!

Юный математик дополнительные занятия в 5 классе

Юный математик дополнительные занятия в 5 классе

п -?

Карточки с ответами выполняют с запасом (есть карточки с числами, не являющимися ответами).

5048

40256

600

24

7

10

6

Нет

друга -

ищи,

а нашел -

береги

сохраняй

врага

Таблица после заполнения:

7!+Юный математик дополнительные занятия в 5 классе

8! - (13 - 5)2

6! - 5!

Юный математик дополнительные занятия в 5 классе

Юный математик дополнительные занятия в 5 классе

п -?

5048

40256

600

24

7

Нет

друга -

ищи,

а нашел -

береги

Задание 6.

К Васе в гости пришли 4 друзей, и они собираются смотреть новый фильм. У Васи в комнате есть кресло и еще он принес 4 стула из кухни. Кресло он, несомненно, займет сам, а на стульях рассадит своих друзей. Вася подсчитал, что рассадить друзей он сможет 24 способами.

Учитель: Правильно ли рассчитал Вася? (Да, с точки зрения математики)

- Хорошо ли он поступил? (Обсуждается моральный аспект проблемы)

4. Физкультурная минутка.

Учитель: А теперь давайте немного отдохнем, а для этого проведем физкультурную минутку. Если я правильно прочитаю выражение, то вы встаете и поднимаете руки вверх, а если неправильно - садитесь, руки в бок.

Встали. Начинаем, будьте внимательны.

Выражение

Слова учителя

Верно / неверно

5! +3

Сумма 5! и 3

+

2 - 7!

Произведение 2 и 7!

-

4х : 2!

Частное 4х и 2!

+

5! + 7! + 3!

Сумма 5!, 7! и 3!

+

20! - 19!

Частное 20! и 19!

-

6. Самостоятельная работа.

Учитель: Ну, а теперь, когда мы хорошо отдохнули, давайте проверим, что мы научились делать сегодня на уроке. Для этого выполним самостоятельную работу.

Вариант 1

Вариант 2

1. В 5 классе в среду 5 уроков: математика, русский язык, литература, музыка и труд. Сколько вариантов расписания на день можно составить?

1. Шесть разных писем раскладывают в 6 разных конвертов. Сколько существует способов такого раскладывания?

2. Вычислите:

а) 6! - 2Юный математик дополнительные занятия в 5 классе; б) 4! + (2+3)2

2. Вычислите:

а) 32 + 5! б) (9-4)2 + 4!

3. Сколькими способами 5 мальчиков могут занять очередь к билетной кассе, если первым все равно будет Толя?

3. Сколькими способами Даша может съесть обед, состоящий из первого, второго, третьего и пирожного, если первым она наверняка съест пирожное?

Задача 1.
Учащимся раздаются цветные полоски (белый, синий, красный) и предлагается из них составить флаг РФ. Затем задаются вопросы исторического характера.
Что означает каждый цвет?
Значение цветов флага России: белый цвет означает мир, чистоту, непорочность, совершенство; синий - цвет веры и верности, постоянства; красный цвет символизирует энергию, силу, кровь, пролитую за Отечество.
Оказывается, есть государства, где флаги имеют такие же цвета.
Видим, что от перестановок цветных полосок, можно получить другой флаг. Как подсчитать, сколько таких флагов мы можем составить из трех цветных полосок?
Решение этой задачи можно записать тремя способами:
1. Таблица вариантов КБС КСБ БСК БКС СБК СКБ
2. Дерево вариантов
Всего получили 3*2=6 комбинаций Вывод: здесь мы получили ответ умножением. Математики сказали бы, что мы использовали известное в комбинаторике правило умножения.
Если существует n вариантов выбора первого элемента и для каждого из них есть m вариантов выбора второго элемента, то всего существует nm различных пар с выбранным первым и вторым элементами.
Такой способ подсчета возможен, если дерево вариантов правильное: из каждого узла одного уровня выходит одно и то же число ветвей. Конечно, совсем не каждую комбинаторную задачу можно решить способом умножения.
Задача 2. (для самостоятельного решения)
На уроке физкультуры Андрей, Максим, Костя, Саша, Петя и Сережа готовятся к прыжкам в высоту. Сколькими способами можно установить для них очередность прыжков?
Ответ: 6*5*4*3*2*1=720 способов
Задача 3 (Все ли задач в комбинаторике решаются умножением)
При встрече 8 приятелей обменялись рукопожатиями. Сколько всего было рукопожатий?
Дадим каждому из приятелей номер - от 1 до 8. тогда каждое рукопожатие можно закодировать двузначным числом. Например, двузначное число 47 - это рукопожатие между приятелями с номерами 4 и 7. ясно, что среди кодов рукопожатий у нас не появится, например, 33 - это означало бы, что один из друзей пожал бы руку сам себе. Кроме того, такие коды, как, например, числа 68 и 86, означают одно и тоже рукопожатие, а значит, учитывать надо только одно из них. Договоримся, что из чисел, кодирующих одно и тоже рукопожатие, мы всегда будем учитывать меньшее. Поэтому из чисел 68 и 86 надо брать 68. коды рукопожатий выписывать надо в порядке возрастания. Для подсчета их удобно расположить треугольником:
12, 13, 14, 15, 16, 17, 18,
23, 24, 25, 26, 27, 28
34, 35, 36, 37, 38
45, 46, 47, 48
56, 57, 58
67, 68
78
Число кодов равно: 7 +6+5+4+3+2+1=28
Таким образом, всего было 28 рукопожатий.
Для решения этой задачи использовали логику перебора.
Задача 4. На школьной олимпиаде по математике оказалось 6 победителей. Однако на районную олимпиаду можно отправить только двоих. Сколько существует вариантов выбора двух кандидатов?
Ответ: 15 вариантов.
Задача 5. Сколько четных четырехзначных чисел можно составить из цифр 0, 2, 3, 4, 5?
Решение:
- Какая цифра может стоять на первом месте в записи числа? (2,3,4,5)
- какие цифры будут стоять на втором и третьем местах в записи числа (Любая из пяти)
- А на последнем? (только четные: 2, 4, 0)
По правилу умножения 4*5*5*3=300
Задача 6.
В четверг в шестом классе пять уроков по разным предметам: русскому языку, истории, математике, географии и физкультуре. Сколько вариантов расписания на четверг можно составить для этого класса?
- Первым уроком какой урок может быть? (Любой из пяти. Значит 5 вариантов)
- Вторым уроком какой может быть? (Любой из оставшихся 4. значит 4 варианта выбора)
- Третьим уроком какой урок может быть? (Любой их оставшихся трех. № варианта выбора)
- Четвертым уроком какой может быть? (Любой из оставшихся 2. 2 варианта выбора)
- Пятым уроком какой может быть? (Только какой-то один из оставшихся уроков)
Решение: 5*4*3*2*1=120 (вариантов)
Итог урока.
Слово учителя: задачи, которые мы сегодня решали и будем решать в дальнейшем помогут вам творить, думать необычно, оригинально, смело, видеть то, мимо чего вы часто проходили не замечая, любить неизвестное, новое; преодолевать трудности и идти через невозможное вперед.
Математика повсюду -
Глазом только поведешь
И примеров сразу уйму
Ты вокруг себя найдешь…

Занятие 36-38. Тема: Занимательные цифровые задачи.

  1. Мороженщик продал 20 порций сливочного и 18 порций фруктового мороженого. Сколько человек он обслужил, если 8 человек купили по 2 порции (1 сливочного и 1 фруктового мороженого), а все остальные лишь по одной порции.

  2. В классе 18 лыжников, 8 борцов, 11 легкоатлетов. Как известно, каждый ученик занимается или одним, или тремя видами. Известно, что тремя видами спорта занимаются пятеро. Сколько всего в классе учеников?

  3. Трое смелых путешественников должны переправиться через речку. Их лодка может выдержать только 100кг груза. Вес путников известен, так как они недавно взвешивались: вес Алеши - 52 кг, Андрея - 42 кг, а дядя Алеши Михаил весит 98кг. Каким образом они могут переправиться на другой берег с наименьшим количеством переправ?

  4. В отчете об изучении иностранных языков студентами полицейской академии говорилось, что из ста человек 5 изучают английский, немецкий и французский языки, 10 - английский и немецкий, 8 - французский и английский, 20 - немецкий и французский, 30 - английский, 23 - немецкий, 50 - французский. Составителям отчета было указано на ошибки. Почему?

  5. В городе Якутске провели опрос у 800 граждан. Газету «Сахаада» читают 430 человек, «Кэскил» читают 220 человек. Газеты «Сахаада» и «Кэскил» читают 180 человек. Из 800, сколько человек не читают ни «Кэскил», ни «Сахааду»?

  6. В комнате сидели мужчины - все родственники. Двое имеют внуков, трое имеют сыновей, у троих есть деды. Четверо имеют отца. Сколько мужчин сидело в комнате?

  7. Алеша и Боря вместе весят 82 кг, Алеша и Вова весят 83 кг, Боря и Вова весят 85 кг. Сколько весят вместе Алеша, Боря и Вова?

Решение:

  1. 20 - сливочное

18 - фруктовое

8 человек по 2 порции, (1 - сливочное, 1 - фруктовое).

Сколько человек обслужил мороженщик?

Решение: 1. 20-8=12

2. 18-8=10

3. 12+10+8=30 Ответ: 30 человек.

  1. 18 лыжников.

8 борцов.

11 легкоатлетов.

1 или 3 вида.

3 вида - 5 учащихся.

Решение: 18 - 5 = 13, 8 -5 = 3, 11 - 5 = 6, 13 + 3 + 6 = 22, 22 + 5 = 27.

Ответ: В классе 27 учащихся.

  1. Алеша - 52 кг.

Андрей - 48 кг.

Михаил - 98 кг. Ответ: 5 переправ.

  1. 5 - английский, немецкий, французский.

10 - английский, немецкий.

8 - французский, английский.

20 - немецкий, французский.

30 - английский.

23 - немецкий.

50 - французский.

Всего - 100 студентов.

  1. «Сахаада» - 430.

«Кэскил» - 220.

«Сахаада», «Кэскил» - 180.

Сколько не читают ни «Сахаада», ни «Кэскил»?

Всего 800 граждан.

Решение: 430 -180 = 250, 220 -180 = 40, 180 + 250 + 40 = 480, 800 - 480 = 320.

Ответ: 320 граждан.

  1. А и В - имеют внуков.

А,В и С - имеют сыновей.

В,С и Д - имеют отца.

С,Д и Е - имеют деда. Ответ: В комнате сидели 5 родственников.

  1. Алеша и Боря - 82 кг.

Алеша и Вова - 83 кг.

Боря и Вова - 85 кг.

Сколько весят Алеша, Боря и Вова вместе?

Решение: А + Б = 82 А + Б + А + В + Б + В = 82 + 83 + 85.

А + В = 83 2А + 2Б + 2В = 250

Б + В = 85 А + Б + В = 250 : 2 = 125.

Ответ: Вместе весят 125 кг.

Занятие 39-41. Тема: Решение задач с помощью таблиц.

1. Четыре ученицы - Света, Зоя, Маша, Олеся - участвовали на лыжных соревнованиях и заняли 4 первых места. На вопрос, кто какое место занял, они дали три разных ответа:

а) Света заняла 1-е место, Маша - 2 -е.

б) Света - 2-е, Олеся - 3-е.

в) Зоя - 2-е, Олеся - 4-е.

2. У Гали, Иры, Кати и Даши было три пары туфель черного цвета и одна пара белого. Какого цвета туфли были у каждой девочки, если известно, что у Иры и Кати туфли были одинакового цвета, а у Кати и Гали - разного?

  1. Коля, Боря, Вова и Юра заняли первые четыре места. На вопрос: «Какие места они заняли?», трое из них ответили:

    1. Коля ни первое, ни четвертое;

    2. Боря второе;

    3. Вова не был последним;

Какое место занял каждый мальчик?

  1. В соревнованиях по шахматам участвовали три спортсмена: Игрок А выиграл партию у игрока Б и сыграли в ничью с игроком В. Если за выигрыш запишем 1 очко, за равенство очка и за проигрыш 0. Определите, кто из них какое место занял?

  2. На соревнованиях по шахматам А занял I место, Б - II -е, В - III - е, Г - IV - е, Д - V -е место.

А: не было ни с кем ничьи.

Б: ни кому не проиграл.

Г: ни кого не выиграл.

Как сыграли между собой В и Д ?

Решение задач с помощью таблицы.

  1. Ответ:

    1

    2

    3

    4

    1

    2

    3

    4

    Света

    +

    -

    Света

    +

    Олеся

    +

    -

    Олеся

    +

    Зоя

    +

    Зоя

    +

    Маша

    -

    Маша

    +

  2. Ответ:

    Черные

    Белые

    Черные

    Белые

    Галя

    +

    Галя

    +

    Ира

    +

    Ира

    +

    Катя

    +

    Катя

    +

    Даша

    Даша

    +


  3. Ответ:

    1

    2

    3

    4

    1

    2

    3

    4

    Коля

    -

    -

    Коля

    -

    -

    +

    -

    Боря

    +

    Боря

    -

    +

    -

    -

    Вова


    -

    Вова

    +

    -

    -

    -

    Юра


    Юра

    -

    -

    -

    +

  4. Ответ:

    Игрок

    1

    2

    3

    Сумма

    Место

    1.

    А

    1


    1

    I

    2.

    Б

    0



    II

    3.

    В



    1

    III

  5. Ответ: В выиграл Д.

Игроки

1

2

3

4

5

Сумма

Место

1.

А

0

1

1

1

3

I

2.

Б

1




2

II

3.

В

0



1

2

III

4.

Г

0




1

IV

5.

Д

0


0


1

V

Занятие №42-45. Проектные работы.

Оформление и выполнение проектов (разработка содержания).

Занятие №46-47. Решение задач

Задачи на «рассуждения» очень часто включаются в задания математических олимпиад разного уровня. Цель данного занятия разобрать основные типы задач, решаемые при помощи рассуждений с минимальным привлечением вычислений. Рассматриваются задачи, которые можно решать и при помощи элементарных алгебраических выкладок, но, учитывая, что учащиеся пятого класса не владеют алгебраическими приемами, предлагается решение задач только при помощи рассуждений.

Задача 1.

Десяти собакам и кошкам скормили 56 котлет. Каждой собаке досталось 6 котлет, а каждой кошке 5 котлет. Сколько было собак, а сколько кошек?.

Решение. Будем рассуждать следующим образом: Скормим каждому животному по 5 котлет. После этого у нас останется 6 котлет. По условию, каждой кошке досталось по 5 котлет, а значит, они уже получили причитающуюся им долю. Поэтому все оставшиеся котлеты надо скормить собакам, причем дать каждой по одной котлете. А значит, мы можем оставшиеся котлеты скормить шестерым псам. Это значит, что собак было 6, а поэтому кошек было 4, если всего животных было 10.

Задача 2.

В зоомагазине продают голубей и синиц. Голубь стоит в два раза дороже синицы. Школьники, зашедшие в магазин, купили для живого уголка 5 голубей и 3 синицы. Если бы они купили 3 голубя и 2 синицы, то потратили бы на 200 рублей меньше. Сколько стоит каждая птица?

Решение. Решим задачу как и предыдущую, используя только рассуждения. Так как цена одного голубя равна цене одной синицы, то 5 голубей стоят столько же сколько и 10 синиц. Значит, 5 голубей и три синицы стоят столько же, сколько и 13 синиц. С другой стороны, цена 3 голубей и 5 синиц равняется цене 11 синиц. Таким образом, разница между ценой 5 голубей и 3 синиц оказывается равной разнице между ценой 13 и11 синиц, а значит равна цене 2 синиц. Поскольку две синицы стоят 200 рублей, то одна стоит 100 рублей. Так как голубь в два раза дороже синицы, то он стоит 200 рублей.

Задача 3.

Масса 10 ящиков болтов и 7 ящиков гвоздей - 366 кг, а 5 ящиков шурупов и 3 ящика навесов - 262 кг. Определите массу одного ящика гвоздей, шурупов, болтов и навесов, если известно, что ящик с гвоздями в три раза легче ящика с навесами, а с болтами - на 4 кг тяжелее, чем с шурупами.

Решение. Зная, что ящик с гвоздями в три раза легче ящика с навесами, имеем, что 1 ящик с навесами весит столько же, сколько 3 ящика с гвоздями три ящика, а значит 5 ящиков с шурупами и 9 ящиков гвоздей весят 262 кг. Теперь, учитывая, что ящик с болтами тяжелее ящика с шурупами на 4 кг, видим, что 5 ящиков с болтами и 9 ящиков с гвоздями весят 282 кг. Учитывая первое условия задачи, получаем, что 11 ящиков с гвоздями весят198 кг, а значит 1 ящик - 18 кг. Теперь можно узнать массу ящика других материалов. Получается, что ящик навесов весит 54 кг, шурупов - 20 кг, болтов - 24 кг.

Из разбора решений видно, что задачи 2 и 3 решаются аналогичным образом, рассуждением и заменой одних объектов в условии задачи другими.

Рассмотрим теперь решение задачи на нахождение трех неизвестных по трем суммам этих неизвестных, взятых попарно. Задача легко решается при помощи алгебраической модели из трех линейных уравнений с тремя неизвестными. Но пятиклассники не владеют этим методом и, по моему мнению, им более понятны конкретные рассуждения по условию задачи.

Задача 4.

Английский и немецкий языки изучают 116 школьников, немецкий и испанский языки учат 46 школьников, а английский и испанский языки изучают 90 школьников. Сколько школьников изучают английский, немецкий и испанский языки отдельно, если известно, что каждый школьник изучает только один язык.

Решение. Сложим все заданные числа. В полученную сумму количество учащихся, изучающих какой-либо язык, войдут дважды, а значит, мы узнали удвоенное количество школьников, изучающих один из иностранных языков. Итак, 252 - это удвоенное количество учеников. Поэтому всего учеников, изучающих языки, будет 126. Вычитая из этого числа 116 школьников, изучающих английский и немецкий языки, получим, что испанский язык учат 10 школьников. Поводя аналогичные рассуждения, получим, что английский язык учат 80 школьников, а немецкий 36.

Эту же задачу можно решить другим способом.

Сложив первые два заданных числа, а именно 116 и 46, мы получим 162. По смыслу задачи, это будут все ученики, изучающие иностранный язык плюс те, кто учит немецкий. И если теперь мы от этого количества отнимем тех, кто учит английский и испанский, а по условию это 90 школьников, то получим 72 ученика, что в два раза больше изучающих немецкий язык. Значит, немецкий язык учат 36 школьников. Теперь из первого и второго условия легко найти, что английский язык учат 80, а испанский 10 учеников.

Рассмотрим еще одну задачу, решаемую при помощи рассуждений.

Задача 5.

В математической олимпиаде участвовали 100 школьников. Было предложено четыре задачи. Первую задачу решили 90 человек, вторую - 80, третью - 70 и четвертую -60. При этом никто не решил все задачи. Награду получили школьники, решившие и третью, и четвертую задачи. Сколько школьников было награждено?

Решение. Так, как первую или вторую задачу или первую и вторую задачу решили 90+80=170 человек, а всего в олимпиаде участвовали 100 человек, то как минимум обе задачи решили 70 человек. Рассуждая аналогично, получаем, что третью и четвертую. Задачу решили как минимум 30 человек. Но по условию, ни один из участников олимпиады не решил все задачи, а значит, первую и вторую решили 70, а третью и четвертую - 30 человек. Таким образом, награждены были 30 человек.

И напоследок, рассмотрим задачу, которую будем решать с конца.

Задача 6.

Два пирата играли на золотые монеты. Сначала первый проиграл половину своих монет и отдал второму, потом второй проиграл половину всех своих монет, потом снова первый проиграл половину своих. В результате у первого оказалось15 монет, а у второго - 33. Сколько монет было у первого пирата до игры?

Решение. Проведем наши рассуждения с конца игровой ситуации. Перед последней игрой у первого пирата было 30 монет, потому что после проигрыша половины у него осталось 15 монет, а у второго, который выиграл в последней игре, до этой игры было 18. Рассуждая аналогичным образом, получим, что перед второй игрой у первого было 12 монет, а у второго - 36. А значит, вначале игры у каждого пирата было по 24 монеты.

Занятие 48-49. Составление и выпуск брошюры «Математическая шкатулка»

Подборка интересных задач и историческая информация.

Занятие 50-51. Выпуск газеты «Занимательная математика»

Подборка задач и историческая информация. На данном занятии кружка каждая группа представляет свою газету. Участники, у которых получилась самая интересная, красочная, занимательная газета, получают небольшие сувениры. Оставшуюся часть времени все учащиеся путешествуют по газете-победительнице, разгадывая ее ребусы, головоломки, кроссворды, решая интересные задачи.
Занятие 52. «Подведение итогов».




1. Какие числа, записанные в четверичной системе счисления одинаковыми цифрами, записываются в двоичной системе счисления хотя и другими, но тоже одинаковыми цифрами?
2. Шутка из «автобиографии» одного математика: «Учиться я начал очень рано и уже в 33 года перешел в выпускной класс. Последний урок совпал с моим днем рождения, когда мне исполнилось 100 лет». В каком возрасте пошел в школу этот мальчик?
3. В комнате собралось 17 человек. Десять из них знают английский язык, 13 - немецкий и французский, 2 человека владеют сразу тремя языками: немецким, французским и английским. Нет ли ошибки в этих данных?
4. Как с помощью четырех кольев, двух колец и веревок заставить козу пастись на участке прямоугольной формы?
5. В четырех пакетах лежат по 5 шариков, причем в трех пакетах каждый шарик весит по 10 грамм, а в оставшемся пакете по 9 гр. Как одним взвешиванием на точных весах с гирями определить, в каком пакете более легкие шарики?
6. В самолете летят 3 пассажира: Волков, Зайцев и Медведев. Такие же фамилии у пилота, штурмана и радиста. Известно, что пассажир Волков живет в Москве, штурман живет на полпути между Москвой и Ленинградом, а пассажир - однофамилец штурмана - живет в Ленинграде. Пассажир - земляк штурмана - вдвое старше его. Зайцеву 37 лет. Медведев и радист уже 5 лет летают вместе. Установите фамилию пилота, штурмана и радиста.
7. В классе 30 учеников. 15 учеников посещают литературный кружок, 11 - биологический. Из них 4 ученика участвуют в работе обоих кружков. 5 учащихся занимаются в литературном и математическом кружках, а 3 - в биологическом и математическом. Только один ученик посещает все три кружка. Остальные учащиеся занимаются только в математическом кружке. Сколько всего учащихся занимаются в математическом кружке?
8. Найдите наибольшее четное пятизначное число, первые 3 цифры которого образуют куб натурального числа, а последние 3 цифры - квадрат натурального числа.
9. Дробь несократима. Будут ли несократимы дроби: ; ; ?
10. Число записано с помощью 30 единиц и нескольких нулей. Может ли оно быть полным квадратом?
11. Хозяева трех домов пользуются тремя колодцами. Но колодцы время от времени пересыхают. Поэтому каждый хозяин решил проложить дорожки от своего дома ко всем трем колодцам, но так, чтобы эти дорожки не пересекали дорожек соседей. Можно ли это сделать?
12. В одном поселке живет 50 школьников, а в другом 100. Где удобнее всего построить школу с таким расчетом, чтобы общий путь, проходимый всеми школьниками, был наименьшим?

  1. Задача. Два туриста выехали на велосипедах в разное время, а ехали с одинаковой скоростью. Когда второй турист проехал 5 км, первый уже проехал 13 км. Через сколько км пути первый турист проедет расстояние в два раза больше второго?

( через 3 км, I - 16 км, II - 8 км)

(3 очка)

  1. В левой части равенства расставьте знаки действий и скобки, чтобы равенство стало верным:

123456789=1

(1·2+3+4-5+6+7-8):9=1

(2 очка)

3. Отец купил сыну пальто за 19 руб. в уплату он дал только трехрублевки и получил 5 руб сдачи. Сколько дал трехрублевок продавцу?

(8 трехрублевок)

(2 очка)

4. Сколько времени прошло от начала суток, если часы показывают без четверти 10?

(9 ч 45 мин)

(2 очка)

5. Девочку спросили, сколько у нее сестер. Она ответила, что у нее сестре столько, сколько и братьев. А ее брат на этот же вопрос ответил, что у него сестер вдвое больше, чем братьев. Сколько в этой семье мальчиков и сколько девочек?

(4 девочки и 3 мальчики)

(3 очка)

6. сколько всего цифр потребуется, чтобы пронумеровать 24 страницы тетради?

(39 цифр)

(2 очка)

7. Какое число (четное или нечетное) получить, если сложить по порядку 6 натуральных чисел?

(нечетное)

(2 очка)

8. Что больше:

1+2+3+4+0 или 1·2·3·4·0?

(>)

(1 очко)

9. Что будет в 2010 году?

(поживем, увидим)

(1 очко)

Домашнее задание. Сочинить стихотворение о цифрах.

Приложение
3 Раздел. "Логические задачи".
1. На улице, встав в кружок, беседуют четыре девочки: Аня, Валя, Галя, и Надя. Девочка в зеленом платье (не Аня и не Валя) стоит между девочкой в голубом платье и Надей. Девочка в белом платье стоит между девочкой в розовом и Валей. Какое платье носит каждая из девочек?
(Ответ: Аня- в белом платье, Валя- в голубом, Галя- в зеленом, Надя- в розовом ).
2. Три друга: Алеша, Боря, и Витя - учатся в одном классе. Один из них ездит домой из школы на автобусе, один - на трамвае и один - на троллейбусе. Однажды после уроков Алеша пошел проводить своего друга до остановки автобуса. Когда мимо них проходил троллейбус, третий друг крикнул из окна: "Боря, ты забыл в школе тетрадку". Кто на чем ездит домой?
(Ответ: Алеша на трамвае, Боря на автобусе, Витя на троллейбусе).
3. В семье четверо детей, им 5, 8, 13 и 15 лет. Детей зовут Аня, Боря, Вера и Галя. Сколько лет каждому ребенку, если одна девочка ходит в детский сад, Аня старше Бори и сумма лет Ани и Веры делится на три?
(Ответ: Вере- 5 лет, Боре- 8 лет, Ане- 13 лет, Гале- 15 лет).
4. В пионерский лагерь приехали три друга: Миша, Дима и Кирилл. Известно, что каждый из них имеет одну из фамилий: Алиев, Малинин, Кадников. Миша не Кадников, отец Димы инженер.
Дима учится в 6-м классе. Алиев учится в 5-м классе. Отец Алиева слесарь. Какая фамилия у каждого из ребят?
(Ответ: Миша- Алиев, Дима- Малинин, Кирилл- Кадников).
5. На столе лежат в ряд четыре фигуры: треугольник, ромб, круг, квадрат. Цвета этих фигур - зеленый, желтый, синий, красный. В каком порядке лежат фигуры и каков цвет каждой из них, если фигура красного цвета лежит между зеленой и синей, справа от желтой фигуры лежит ромб, круг лежит правее треугольника и ромба, причем треугольник лежит не с краю, и, наконец, фигура синего цвета не лежит рядом с фигурой желтого цвета?
(Ответ: желтый квадрат, зеленый ромб, красный треугольник, синий круг).
6. Четыре ученицы: Мария, Нина, Ольга, и Поля- участвовали в лыжных соревнованиях и заняли четыре первых места. На вопрос, кто какое место занял, они дали три разных ответа:
1. Ольга заняла первое место, Нина- второе;
2. Ольга - второе, Поля-третье;
3. Мария - второе, Поля- четвертое.
Отвечавшие при этом признали, что одна часть каждого ответа верна, а другая - неверна. Какое место заняла каждая из учениц?
(Ответ: Оля- 1 место, Мария- 2 место, Поля- 3 место, Нина- 4 место).
7. На острове два племени: аборигены и пришельцы. Аборигены всегда говорят правду, пришельцы всегда лгут. Путешественник нанял туземца-островитянина в проводники. Они пошли и увидели другого островитянина. Путешественник послал проводника узнать, к какому племени принадлежит этот туземец.
Проводник вернулся и сказал, что туземец назвал себя аборигеном. Кем был проводник: пришельцем или аборигеном?
(Ответ: проводник - абориген)
8. В одной сказочной стране поблизости один от другого находятся города А и В. Все жители города А говорят только правду, а жители города В всегда лгут. Жители этих городов ходят друг к другу в гости. Путешественник попал в один из этих городов, но не знает в какой. Как он может, задав один вопрос первому попавшемуся жителю, узнать, в каком городе он находится?
(Ответ: вы житель этого города?).
9. Коля, Боря, Вова и Юра заняли первые четыре места в соревновании. На вопрос, какие места они заняли, трое из них ответили:
1) Коля ни первое, ни четвертое;
2) Боря второе;
3) Вова не был последним.
Какое место занял каждый мальчик?
10. В кафе встретились три друга: скульптор Белов, скрипач Чернов и художник Рыжов. «Замечательно, что у одного из нас белые, у другого черные, а у третьего рыжие волосы, но ни у кого цвет волос не соответствует фамилии»,- заметил черноволосый.«Ты прав»,- сказал Белов. Какой цвет волос у художника?
11. В лесу проводился кросс. Обсуждая его итоги, одна белка сказала: «Первое место занял заяц, а второй была лиса». Другая белка возразила: «Заяц занял второе место, а лось был первым». На что филин заметил, что в высказывании каждой белки одна часть верная, а другая - нет. Кто был первым и кто вторым в кроссе?
12. Летела стая уток. Одна впереди, две позади; одна позади и две впереди; одна между двумя и три в ряд. Сколько летело уток?
13. Отряд солдат должен переправиться с одного берега реки на другой, пользуясь услугами двух мальчиков и лодкой, в которой могут поместиться или два мальчика, или один солдат. Как это сделать?
14. Дедушка с тремя внуками вышел прогуляться в парк. Знакомый спросил его: сколько каждому из них лет? Ваня сказал: «Я младше Пети и мне больше 5 лет». Петя сказал: «Я младше Саши на три года», а Саша заметил: «Нам всем вместе в три раза меньше лет, чем дедушке, а вместе с дедушкой ровно 100 лет». Сколько лет каждому из внуков?
15. Четыре подруги со своими братьями пришли на каток. Оказалось, что в каждой паре кавалер выше дамы, причем никто не катается со своей сестрой. Самый высокий - Юра Воробьёв, потом - Андрей Егоров, потом - Лена Егорова, Сережа Петров, Оля Петрова, Дима Крымов, Инна Крымова и Аня Воробьева. Кто с кем катался?
16. Коля, Петя и Ваня собирали грибы. Коля нашёл 10 сыроежек и столько белых, сколько подберезовиков нашел Ваня. Ваня нашел лисичек в 2 раза меньше, чем сыроежек Коля, и 3 подберезовика. Петя нашел только лисички, которых у него было больше, чем белых у Коли, но меньше, чем лисичек у Вани. Сколько грибов собрали ребята?
Тема: Решение логических задач. (Числа).

  1. Установите закономерность в ряды чисел: 186,345,***,713. Найдите третье число.

  2. Может ли число, составленное из одних четверок, делится на число, составленное из одних троек? А наоборот?

  3. Сколько чисел существует от единицы до тысяча, неделимых на два и пять?

  4. В двузначном числе в два раза больше единиц, чем десятков. Если к этому числу прибавить 36, то получится число, записанное теми же цифрами. Найдите эти числа.

  5. В законопослушной семье возраст папы представляет собой точный квадрат, а произведение цифр его возраста равно возрасту мамы. Возраст дочери и сына соответственно равны сумме цифр возраста папы и мамы. Сколько лет каждому члену семьи?

  6. Расставьте цифры 1,2,3,4,5,6,7 в кружки так, чтобы каждая сумма тройки чисел на прямых была одинаковой.

  7. Чтобы открыть сейф, нужно ввести код - число, состоящее из семи цифр: двоек и троек. Сейф откроется, если двоек больше, чем троек, а код делится и на три, и на четыре. Придумайте код, открывающий сейф.

  8. В ряд выписали 11 натуральных чисел так, что сумма любых трех соседних чисел равно 21. На первом месте стоит число семь, а на девятом - 6. Какое число стоит на втором месте?

  9. Пятизначное число умножили на девять. При этом получили данное число с обратным порядком цифр. Найти это число.

  10. Таня в 6 раз моложе своего прадедушки. Кроме того, она заметила, что если между цифрами ее возраста поставить ноль, то получится возраст прадедушки. Сколько ей лет?

Задача 1. 186,345,***,713.

Решение: В разряде сотых: 1,3,*,7 - последовательность нечетных чисел, * = 5.

В разряде десятых: 8,4,*,1. 8 : 2 = 4, 4 : 2 = 2, 2 : 2 = 1. * = 2.

В разряде единиц: 6,5,*,3. - убывание. * = 4.

Ответ: Третье число равно 524.

Задача 2. 444444…:33333….=? или 3333…: 4444….=?

Решение: 1. 4 : 3 = 1. 3 : 4 =

2. 44 : 33 = 2. 33 : 44 =

Задача 3. От 1 до 1000 не делимых на 2 и на 5.

Решение: На два делятся четные числа, на 5 числа, которые оканчиваются на 0 и на 5.

Из 1000 500 - четных и 500 - нечетных.

Из 500 на 5 оканчиваются 50 чисел.

Ответ:

Задача 4. В два раза больше единиц, чем десятков.

Решение: Составим такие числа: 12,24,39,48.

12 + 36 = 48 (не подходит), 24 + 36 = 60, 39 + 36 = 75, 48 + 36 = 84.

Ответ: Число 48.

Задача 5. Точный квадрат - возраст папы.

Произведение цифр возраста папы - возраст мамы.

Сумма цифр возраста папы - возраст дочери.

Сумма цифр возраста мамы - возраст сына.

Решение: 4,9,16,25,36,49,64,81…

Для возраста папы подходят числа: 25,36,49,64,81.

Для возраста мамы: 18,36,24.

Составим закономерность возрастов и получаем, что подходят числа: 49 и 36, тогда 13 - возраст дочери, 9 - возраст сына.

Ответ: 49,36,13,9.

Задача 6. 4

2 3 7

6 5 1

Задача 7. * * * * * * * - код сейфа. Состоит из двоек и троек, причем количество двоек больше, чем количество троек и полученное число делится и на три, и на четыре.

Решение: (* +*+*+*+*+*+*) : 3 и ******2 : 4. Пусть двоек будет четыре: 2+2+2+2+3+3+3=17 (не делится на три), если пять, то 2+2+2+2+2+3+3=16 (не делится на три), тогда шесть: 2+2+2+2+2+2+3 = 15.

Ответ: Код сейфа: 2222232.

Задача 8. 7*******6**, 7?******6**.

Решение: 7 + 6 = 13 и 13 + 8 = 21. Это будут числа 6,7,8. 76876876876 - не подходит. 78678678678.

Ответ: На втором месте стоит число восемь.

Задача 9. АБВГД*9 = ДГВБА.

Ответ: 10989*9 = 98901.

Задача 10. Двузначное число. 10*6 = 60, 11*6 =66, 12*6 =72, 13*6 = 78, 14*6 = 84, 15*6 = 90, 16*6 = 96, 17*6 = 102, 18*6 = 108.

Ответ: Тане 18 лет.

"Задачи с числами".
1. На столе лежат 40 камешков. Двое играющих берут поочерёдно со стола камешки, причём за один раз не более 10 камешков. Выигрывает тот, кто берет последний камешек. Как должен поступить начинающий игру, чтобы наверняка выиграть?
2. Играют двое. Начинающий называет одно из чисел: 1, 2, 3, 4. Второй игрок прибавляет к этому числу одно из этих же чисел: 1, 2, 3, 4 и называет вслух получившуюся сумму. То же самое делает потом первый игрок и т.д. Выигравшим считается тот, кто первым назовет число 40. Как, по-вашему, кто выиграет?
3. В ящике лежат 35 шариков. Двое играющих по очереди вынимают их из ящика, причем по условию игры каждый обязан вынуть в свой ход не менее одного шарика и не более пяти. Проигравшим считается тот, кто вынужден будет своим ходом вынуть из ящика последний шар. Может ли игрок, делающий ход первым, обеспечить себе выигрыш? Каким образом?
4. На столе лежат три кучки камешков. В одной кучке 1 камешек, в другой- 2, в третьей-3. Двое играющих берут поочередно камешки, причем за один раз можно взять любое число камешков из одной кучки. Выигрывает тот, кто берет последний камешек. Докажите, что при правильной игре второго начинающий игру обязательно проигрывает.
5. В клетках квадрата переставьте числа так, чтобы по любой вертикали, горизонтали и диагонали их суммы были равны между собой:
3 5 7
9 11 13
15 17 19
6. Даны числа: 5, 10, 15, 20, 25, 30, 35, 40, 45. Впишите их в клетки девятиклеточного квадрата так, чтобы получилось в сумме одно и то же число по любой вертикали, горизонтали и диагонали.
7. Разместите в свободных клетках квадрата ещё числа 3, 4, 5, 6, 8, 9 так, чтобы по любой вертикали, горизонтали и диагонали получилось в сумме одно и то же число:
10
7
1 11
8. Карлсон предложил Малышу следующую игру. На столе лежат две кучки по 7 и 8 спичек. Первый делит одну из кучек на две кучки, затем второй делит одну из кучек на две кучки и т.д. Проигрывает тот, кто не сможет сделать очередного хода. Карлсон начинает. Кто выиграет в этой игре? Зависит ли результат от того, кто как играет, или важно лишь, кто ходит первым?
9. Заполните магический квадрат так, чтобы сумма цифр в каждом ряду по вертикали и горизонтали была равна 1000:
612 198
252
210
10. Игра «Сто»: Играют двое. Первый называет любое число от 1 до 10 включительно, второй прибавляет к этому числу ещё какое-нибудь число, не большее 10, и называет сумму; к этой сумме первый опять прибавляет какое-нибудь число, не большее 10, и т.д. Выигрывает тот, кто первым назовет число 100.

Задача-шутка.

На пастбище паслись телята, гуляли гусята. Общее число ног телят было 392, а общее число лап гусят на 94 меньше числа ног телят. Сколько телят и сколько гусят было на пастбище?

V Продолжи ряд

6, 8, 16, 18, 36, …

9, 11, 31, 33, 53, …

15, 24, 35, 48, 63, …

2, 3, 6, 7, 10, 11, 14, …

Домашнее задание. Задача. Стоимость книги равна 620 р. плюс половина стоимости книги. Сколько стоит эта книга?
Задачи на "Взвешивания".
1. Из трех одинаковых по виду колец одно несколько легче других. Как найти его одним взвешиванием на чашечных весах?
(Ответ: Кладем два кольца на весы. Если весы в равновесии, то оставшееся кольцо более легкое; если же одно кольцо перевесило, то ответ ясен).
2. меются девять пластин и двухчашечные весы. Одна из пластин легче других, но по виду они одинаковы. Как с помощью двух взвешиваний найти более легкую пластину?
(Ответ: Разделить пластинки на три группы по три пластинки в каждой).
3. Среди 27 монет одна фальшивая. Как найти фальшивую монету с помощью трех взвешиваний на весах с чашечками без гирь, если известно, что фальшивая монета тяжелее, чем настоящая?
(Ответ: Разделить на 3 группы по 9 монет и сначала установить, в какой группе фальшивая монета).
4. Имеются двухчашечные весы и гири массой 1, 2, 4, 8, 16 грамм. На одну чашу весов кладут груз, на другую можно класть гири. Докажите, что весы можно уравновесить, если масса груза равна:
а) 13, 19, 23, 31 грамм;
b) любому целому числу граммов от 1 до 31 включительно.
(Ответ: а) 13=8+4+1; 19=16+2+1; 23=16+4+2+1; 31=16+8+4+2+1.
b) с помощью гирь в 1 и 2 гр легко взвесить массы в 1, 2 и3 гр; добавляя гирю в 4 гр, можем взвесить массы от 4-7 гр, добавляя гирю в 8 гр, можно взвесить массы от 8-15 гр и т.д.).
5. Имеются двухчашечные весы и гири массой 1, 3, 9, 27, 81 грамм. На одну чашу весов кладут груз, гири разрешается класть на обе чаши. Докажите, что весы можно уравновесить, если масса груза равна:
a)31, 52, 74, 80 гр;
b) любому целому числу граммов от 1 до 121 включительно.
6. На плохо отрегулированных весах бабушка взвесила два пакета сахарного песку- получилось 500 гр. И 300 гр. Когда же она взвесила на тех же весах оба пакета вместе, то получилось 900 гр. Определите по этим данным вес каждого пакета.
7. Из 3 одинаковых с виду монет одна фальшивая, но неизвестно, она тяжелее или легче остальных. Как определить фальшивую монету, сделав не более двух взвешиваний на чашечных весах без гирь?
4 Раздел. "Геометрические задачи".
1. Фигуру, показанную на рисунке, нужно обвести, не отрывая карандаш от бумаги и не обводя одно и то же ребро дважды. Если допустить, что линии могут пересекаться, то задача решается просто. Решение весьма усложняется, если пересечение линий запрещено.
(Ответ: ).
2. У треугольника, длины сторон которого- целые числа, длина одной стороны равна 5,а другой- 1. Чему равна длина третье стороны?
(Ответ: 5.)
3. Две противоположные стороны прямоугольника увеличили на 1/6 часть, а две другие уменьшили на 1/6 часть. Как изменится площадь прямоугольника?
(Ответ: площадь уменьшится на 1/36 часть.)
4. В бассейне с горизонтальным дном площадью 1 га. Содержится миллион литров воды. Можно ли в этом бассейне проводить соревнования по плаванию?
(Ответ: 1 га.- это 10000 кв.м=1000000 кв.дм, т.е на 1 кв.дм площади поверхности дна приходится 1 л. воды. Но 1л.=1 куб.дм.. Следовательно, глубина слоя воды 1 дм. Соревнования по плаванию проводить нельзя.)
5. Окрашенный куб с ребром в 10см. распилили на кубики с ребром в 1см. Сколько среди них окажется кубиков с одной и двумя окрашенными гранями?
(Ответ: Вдоль каждого из 12 ребер куба образуется 8 кубиков с двумя окрашенными гранями. Таких кубиков 8*12=96. Внутренний квадрат со стороной 8см. на каждой из 6 граней куба образует после распилки куба 8*8=64 кубика с одной окрашенной гранью. Всего их будет 64*6=384 кубика.)
6. Объем деревянного бруска 80 см3, ширина 4 см, высота 2 см. Длину этого бруска уменьшили на 3 см. Определить объем оставшейся части.
7. Маша собралась клеить кубики, для чего она нарисовала различные заготовки. Старший брат посмотрел её работу и сказал, что некоторые заготовки не являются развертками кубика. Из каких заготовок можно склеить кубики?
8. Сторону квадрата увеличили на 4 см и получили второй квадрат, имеющий площадь 81 см2. Найдите площадь первого квадрата.
9. У Маши был аквариум, основание которого - квадрат со стороной 24 см; уровень воды в нем достигал 36 см. Купили новый аквариум длиной 36 см, шириной 24 см. Маша перелила воду в новый аквариум. Определите уровень воды в новом аквариуме.
10. Прямоугольный параллелепипед имеет длину 1250 см, ширину 720 см, высоту 80 см. Его разрезали на кубические дециметры и разместили их в один ряд, положив в плотную друг к другу. Какой длины получился ряд?
5 Раздел. "Комбинаторика".
1. Сколькими способами можно представить число 50 в виде суммы двух четных положительных целых чисел? (Представления, различающиеся порядком слагаемых, считать совпадающими).
(Ответ: 50=2+48=4+46=…=46+4=48+2, всего 24 представления, но т.к представления а+b и b+а совпадающие, то получаем 12 способов.)
2. Сколькими способами можно представить число 6 в виде суммы нечетных слагаемых? (Представления, различающиеся порядком слагаемых, считать одинаковыми).
(Ответ: 6=1+1+1+1+1+1=1+1+1+3=1+5=3+3 всего четыре способа.)
3. Любую ли сумму из целого числа рублей, большего семи, можно уплатить без сдачи денежными купюрами в 3 и 5 рублей?
(Ответ: да, достаточно проверить 8, 9, 10, а затем добавлять по 3 рубля.)
4. Кусок проволоки длиной 102 см. нужно разрезать на части длиной 15 и 12 см., но так, чтобы обрезков не было. Как это сделать? Сколько решений имеет задача?
(Ответ: 102=12+6*15=6*12+2*15, два решения.)
5. Задумано трехзначное число, у которого с любым из чисел 543, 142 и 562 совпадает один из разрядов, а два других не совпадают. Какое число задумано?
(Ответ: 163.)
6. При составлении расписания уроков на вторник трое преподавателей высказали пожелания, чтобы их уроки были: по математике- 1-й или 2-й; по истории- 1-й или 3-й; по литературе- 2-й или 3-й. Сколькими способами и как при составлении расписания можно удовлетворить пожелания всех преподавателей?
(Ответ: 1 способ: математика, литература, история;
2 способ: история, математика, литература.
7. Запишите в строчку три числа так, чтобы сумма любых двух соседних чисел была четная, а сумма всех чисел была нечетная.
(Ответ: например 1,3, 5.)
8. Некто пообещал дать 99 конфет тому, кто сумеет их разделить между четырьмя детьми так, чтобы каждому досталось нечетное число конфет. Почему этот приз до сих пор никому не удалось получить?
9. Саша купил в магазине 20 тетрадей, 2 альбома для рисования, авторучку за 120 рублей, несколько карандашей по 8 руб. и несколько обложек для книг по 30 руб.. Ему сказали, что в кассу следует уплатить 457 руб.. Саша попросил пересчитать стоимость покупки, и ошибка была исправлена. Как он догадался, что была допущена ошибка?
10. Подпольный миллионер Тарас Артемов пришел в Госбанк, чтобы обменять несколько 50- и 100- рублевых купюр старого образца. Ему была выдана 1991 купюра более мелкого достоинства, причем среди них не было 10- рублевых. Докажите, что его обсчитали.
11. Сколькими способами можно уплатить без сдачи 28 рублей, имея монеты 1- и 5- рублевого достоинства?
12. Алеша, Боря, Вася и Гена - лучшие математики класса. На школьную олимпиаду нужно выставить команду из трех человек. Сколькими способами это можно сделать?
13. Восемь подружек решили обменяться фотографиями так, чтобы у каждой из них оказались фотографии остальных подруг. Сколько фотографий для этого требуется?
6 Раздел. "Цифровые задачи".
1. Из книги выпала какая-то её часть. Первая страница выпавшего куска имеет номер 387, а номер последней страницы состоит из тех же цифр, но записанных в другом порядке. Сколько листов выпало из книги?
(Ответ: Номер последней страницы в выпавшем блоке листов- четное число, т.е. 738. Т.о. выпавший блок содержит (738-386):2=176 листов.)
2. В записи 1*2*3*4*5 звездочки замените знаками действий и расставьте скобки так, чтобы получилось выражение, значение которого равно 100.
(Ответ: 1(2+3)4. 5)
3. Расставьте в записи 7*9+12:3-2 скобки так, чтобы значение этого выражения было равно 23 и 75.
(Ответ: (7*9+12):3-2=23; (7*9+12):(3-2)= 75.)
4. Семиклассник Петя переехал в новый пятиэтажный дом, у которого первый и второй этажи во 2-м и 3-м подъездах заняты под магазин. Все заселенные лестничные площадки дома устроены одинаково, на каждой из них находится не более четырех квартир. Номер квартиры Пети- 31. На каком этаже живет Петя?
(Ответ: на 5 этаже.)
5. Братья Алеша и Боря родились в августе. В школе начинают учиться с 7 лет. Номер класса, в котором учится сейчас старший брат Борис, равен возрасту Алеши. В какой класс перейдет Алеша, когда Борис окончит 10 классов?
(Ответ: в 5 класс.)
6. Гриша с папой пошёл в тир. Уговор был такой: Гриша делает 5 выстрелов и за каждое попадание в цель получает право сделать ещё 2 выстрела. Всего Гриша сделал 17 выстрелов. Сколько раз Гриша попал в цель? (Ответ: 6 раз.)
7.Улитка каждый день вползает по стене на 7 метров вверх и ночью опускается на 4 метра вниз. На какой день она, начав от земли, достигнет крыши дома, высота которого 19 метров?
(Ответ: К началу 5-го дня улитка преодолела 12 метров и концу этого дня была на крыше дома.)
8. Заполните клетки так, чтобы сумма чисел, стоящих в любых трех соседних клетках, равнялась 15:

6 4
(Ответ: числа, между которыми лежит по 2 клетки, должны совпадать.)
9. В записи 66666666 поставьте между некоторыми цифрами знак сложения так, чтобы получилось выражение, значение которого равно:
a) 264; б) 13332; в) 67332.
10. Как нужно расставить скобки, чтобы получить верное равенство:
a) 3248: 16- 3*315-156*2=600;
b) 350-15*104-1428:14=320.
11. В клетках таблице расставьте целые числа так, чтобы их сумма в каждой строке была ровна 35, а в каждом столбце 20. Найдите несколько решений.
12. Вася знает четыре числа, сумма которых равна 99. Если первое число увеличить на 2, второе уменьшить на 2, третье умножить на 2, а четвертое разделить на 2, то каждый раз получается одно и то же число. Найдите эти четыре числа.
13. Из некоторого числа вычли сумму его цифр, из полученного числа вычли сумму его цифр и т.д. После одиннадцатого вычитания впервые получили 0. Каким могло быть первое число?
14. Найти двузначное число, которое на 6 меньше квадрата суммы своих цифр.
15. Произведение числа на его обращенное равно 692443.Найти это число.
16. Мать поручила детям - брату и сестре - разложить конфеты так, чтобы на завтра к обеду для гостей была оставлена половина всех конфет и ещё три штуки, к завтраку для всей семьи - половина оставшихся конфет и ещё три штуки и к вечернему чаю - половина оставшихся конфет и ещё три штуки. Дети разложили конфеты в три вазы так, как велела мать, и у них осталось ещё 4 конфеты, которые им разрешили съесть. Сколько было конфет?
17. У Пети 3 брата. Первый старше его на 3 года, второй моложе на 3 года, третий моложе Пети втрое. Зато отец втрое старше Пети. Всем вместе 95 лет. Сколько лет каждому?
18. Шифр замка-автомата - семизначное число, три первые цифры которого одинаковы, остальные четыре цифры также одинаковы. Сумма всех цифр этого числа - число двузначное, первая цифра которого совпадает с первой цифрой шифра, а последняя - с последней. Найдите этот шифр.
"Алгебраические задачи".
1. Я решил определить расстояние от моего дома до дома моего приятеля. Я шел равномерным шагом и полпути считал шаги парами, а полпути- тройками, причем пар получилось на 250 больше, чем троек. Сколько шагов до дома моего приятеля?
(Ответ: 3000 шагов.)
2. Самолет летел из А в B. Сначала он летел со скоростью 180 км/ч, но когда ему осталось лететь на 320 км меньше, чем он пролетел, он увеличил скорость до 250 км/ч. Оказалось, что средняя скорость самолета на всем пути 200 км/ч. Определить расстояние от А до В.
(Ответ: 1120 км.)
3. Пассажир, проезжая в трамвае, заметил знакомого, который шел вдоль линии трамвая в противоположную сторону. Через 10 сек пассажир вышел из трамвая и пошел догонять своего знакомого. Через сколько секунд он догонит знакомого, если он идет в 2 раза быстрее знакомого и в 5 раз медленнее трамвая?
(Ответ: 110 сек.)
4. У двух рыбаков спросили: « Сколько рыбы в ваших корзинах?»- « В моей корзине половина числа рыб, находящихся в корзине у него, да еще 10», - ответил первый. «А у меня в корзине столько рыб, сколько у него, да еще 20»,- сказал второй. Сколько же рыб у обоих?
(Ответ: 100 рыб.)
5. Моему брату через 2 года будет вдвое больше лет, чем ему было 2 года назад, а моя двоюродная сестра через 3 года будет втрое старше, чем 3 года назад. Кто из них старше?
(Ответ: им по 6 лет.)
6. В парке живут воробьи, синицы, голуби и вороны- всего 10000 птичек. Воробьев в 10 раз больше, чем ворон; голубей на 400 больше, чем ворон; синиц на 1400 меньше, чем воробьев. Сколько, каких птичек живет в порке?
(Ответ: 5000-воробьев; 3600- синиц; 900- голубей; 500- ворон.)
7. Андрюша, Боря, Вадик и Гена разговаривали о своих книгах. Андрюша сказал: «У Гены книг в 2 раза больше, чем у меня». Боря сказал: « У меня столько книг, сколько у Андрюши и Вадика вместе». Вадик сказал: « У меня на 3 книги меньше, чем у Гены». Гена сказал: «У меня столько книг, сколько у Бори и Вадика вместе». Сколько книг у каждого мальчика?
(Ответ: Андрей-2 книги; Боря- 3 книги; Вадик-1 книга; Гена- 4 книги.)
8. Москва старше Санкт-Петербурга на 556 лет. В 1981 году Москва была втрое старше Санкт-Петербурга. В каком году основана Москва, и в каком году основан Санкт-Петербург?
9. Рыболов на вопрос, какова масса пойманной им рыбы, ответил: « Масса хвоста 1 кг, масса головы составляет столько, сколько хвост и половина туловища, а масса туловища - столько, сколько голова и хвост вместе». Найти массу рыбы.
(Ответ: 8 кг.)
10. Саша заметил, что когда он ехал в школу на автобусе, а возвращался на троллейбусе, то на весь путь было затрачено35 мин. Когда же он туда и обратно ехал на автобусе, затратил 40 мин. Сколько времени потратит Саша на путь в школу и обратно, если будет ехать на троллейбусе?

7 Раздел. «Мир процентов и среднего арифметического».
1. Собрали 100 кг грибов. Оказалось, что их влажность 99%. Когда грибы подсушили, влажность снизилась до 98%. Какой стала масса грибов после подсушивания?
2. Рост человека археологи могут определить даже по отдельным костям. Например, длина малой берцовой кости составляет 22% роста человека, а локтевой кости составляет 16% роста человека.
a) При раскопках нашли малую берцовую кость длиной 39,3 см. Вычислите, каков был рост человека.
b) Как можно доказать, что локтевая кость длиной 20,3 см не могла принадлежать тому же человеку?
3. Сеня купил три пакета орехов, а Саша - 2 таких пакета. К ним присоединился Костя, и они разделили все орехи поровну. При расчете оказалось, что Костя должен уплатить товарищам 25 коп. Сколько денег из этой суммы должен получить Сеня и сколько Саша? Сколько стоит пакет орехов?
4. Средний возраст 11-ти футболистов команды 22 года. Во время игры один из игроков получил травму и ушел с поля. Средний возраст оставшихся на поле игроков стал 21 год. Сколько лет футболисту, ушедшему с поля?
5. Фокус «Опять пять!».
Задумайте натуральное число, прибавьте к нему следующее за ним по порядку, добавьте к результату 9, разделите на 2, вычтите задуманное число. «А теперь я знаю, сколько у вас получилось!»- «Сколько?»- «5!».
Домашнее задание:
v Цена входного билета на стадион была 180 руб. После снижения входной платы число зрителей увеличилось на 50%, а выручка выросла на 25%. Сколько стал стоить билет после снижения?
v У горного барана массой 150 кг масса рогов равна 30 кг. Сколько процентов составляет масса рогов от массы тела: 20% или 25%?
v Трое жильцов готовят обед на одной печи. Жилица - назовем её для удобства Тройкиной - положила в общую печь 3 полена своих дров, жилица Пятеркина - 5 поленьев, жилец Бестопливный, у которого не было своих дров, получил от обеих гражданок разрешение сварить обед на общем огне. В возмещение расходов он уплатил соседкам 80 коп. Как должны они поделить между собой эту плату?
« Поиски закономерностей».
1. Найдите правило нахождения числа, помещенного в окошке чердака. Вставьте число в свободное окошко.
2. Найдите число на «голове».
3. Вставьте пропущенное число, если числа в табличках составлены по одному и тому же закону.
11 12 15 16 14 6 15 7 ? 8 10 7
4. Игра «Стертая цифра».
Участникам игры предлагается написать какое-нибудь многозначное число, например 6745693, а затем переставить в нем цифры любым образом, например 5937466. Найти разность полученных чисел, в данном примере:
В полученной разности предлагается стереть одну из цифр (кроме нуля) и подсчитать сумму оставшихся цифр, по которой ведущий и угадывает стертую цифру. Например, в рассматриваемом примере решили стереть цифру 7, тогда 8+0+8+2+2=20. Далее, ведущий от 27 отнимает 20 и получает зачеркнутую цифру.
Домашнее задание:
v Найдите правило составления последовательности чисел и вставьте вместо звездочки пропущенное число: 5; 14; 41; 122; * ; 1094.
v Найдите правила размещения чисел в полукругах и вставьте недостающие числа:
v Впишите недостающие числа в таблицу:
2 6 12 20 30 42
«Решение уравнений»
1. 4х + 5 = 7х - 4;
2. 5а - 7 = 3а - 1;
3. 4(у + 2)= 3(3у - 4);
4. Игра «Лесенка». Каждый играющий получает карточку, на которой нарисована лесенка, в строчках ее по две клеточки. По сигналу ведущего все играющие пишут любое двузначное число на верхней ступеньке. Затем сносят последнюю цифру написанного числа в следующую строчку по вертикали. К снесенной цифре приписывают вторую цифру так, чтобы получилось нечетное число. Затем опять сносят последнюю цифру по вертикали в следующую строчку и приписывают одну цифру так, чтобы полученное вновь число делилось на 3, далее на 4, на 5 и т.д до 10. Выигрывает тот, кто первым правильно закончит «лесенку».
Домашнее задание:
v Решите уравнение: 5х +3х - 2 = 2(3х + 5);
v Решите уравнение: 8(х+3) = 75 - (х-3);
v Составьте условие задачи, которая решалась бы с помощью уравнения: 4(х-5)=3х-2. Решите её.
Занятие №4.
«Решение задач с помощью уравнения»
1. На одной чаше весов лежат 6 одинаковых яблок и 3 одинаковые груши, на другой чаше- 3 таких же яблока и 5 таких же груш. Весы находятся в равновесии. Что легче: яблоко или груша?
2. Сын спросил отца, сколько ему лет. Отец ответил: «Если к половине моих лет прибавить 12, то узнаешь, сколько мне было 12 лет назад». Сколько лет отцу?
3. Витя задумал двузначное число, в котором цифра десятков в 2 раза меньше цифры единиц. Если цифры в этом числе переставить, то полученное обращенное число будет на 36 больше задуманного. Найти задуманное Витей число.
4. Игра: пройдите все незаштрихованные клетки квадрата так, чтобы ни в одной не побывать дважды и вернуться к начальной клетке. Обход начните с клетки, в которой стоит крестик. По диагонали ходить нельзя.
5. У Володи и его отца сегодня день рождения. Отец старше сына ровно в 11 раз. Через 6 лет он будет старше сына только в 5 раз. Сколько лет сыну и сколько отцу?
Домашнее задание:
v Турист проехал поездом, на автомобиле и на велосипеде всего 900 км. На автомобиле он ехал со скоростью 45 км /ч, на велосипеде - 15 км /ч. Поездом он проехал на 90 км больше, чем на велосипеде. Сколько часов турист ехал на автомобиле и сколько на велосипеде, если путь, пройденный им на автомобиле, вчетверо больше пути, пройденного на велосипеде?
v Библиотекой куплено на равные суммы несколько одинаковых книг по математике и одинаковых - по литературе. Известно, что книг по литературе на 20 меньше, чем по математике. Сколько куплено тех и других книг, если одна книга по литературе стоит 63 руб., а одна книга по математике - 35 руб.?
v В трех ящиках лежат орехи. В первом ящике на 6 орехов меньше, чем в двух других вместе, а во втором - на 10 меньше, чем в первом и третьем вместе. Сколько орехов в третьем ящике?
«Геометрические головоломки».
1. (Домашняя заготовка: вырезаны 16 одинаковых квадратов четырех цветов - по 4 квадрата каждого цвета). Сложите из них квадрат 4 на 4 так, чтобы одинаковые цвета не повторялись:
a) ни в строчках, ни в столбцах;
b) ни в строчках, ни в столбцах, ни по диагонали.
Зарисуйте решения в тетрадь, используя цветные карандаши или фломастеры.
2. На четырех квадратах каждого цвета напишите цифры 1, 2, 3, 4. Сложите теперь квадрат 4 на 4 так, чтобы одинаковые цифры и одинаковые цвета не повторялись ни в строках, ни в столбцах, ни на диагоналях квадрата.
3. Из спичек построен дом. Переложить две спички так, чтобы дом повернулся другой стороной.
4. Из 10 спичек сделан ключ. Переложить в нем 4 спички так, чтобы получилось три квадрата.
5. Имеются 4 куска проволоки длиной 18 см каждый. Как из них сделать каркасную модель параллелепипеда с размерами 8 см, 4 см и 6 см, не разрезая этих кусков проволоки?
Домашнее задание:
v На коврике изображено 7 роз. Требуется тремя прямыми линиями разрезать коврик на 7 частей, каждая из которых содержала бы по одной розе.
v Не отрывая карандаш от бумаги и не обводя дважды один и тот же участок, вычертить фигуру изображенную на рисунке.
«Сообщения о великих математиках».
Процесс работы над докладом состоит из следующих этапов:
1. выбор темы;
2. составление плана доклада;
3. определение источников, литературы и знакомство с ними;
4. обработка и систематизация собранного материала;
5. написание доклада.
«Площади и объемы».
1. У Маши был аквариум, основание которого - квадрат со стороной 24 см; уровень воды в нем достигал 36 см. Купили новый аквариум длиной 36 см, шириной 24 см. Маша перелила воду в новый аквариум. Определите уровень воды в новом аквариуме.
2.Из листа бумаги, размер которого 950 на 1200 мм2 можно вырезать или квадраты со стороной 64 мм, или квадраты со стороной 46 мм. Какие квадраты надо вырезать, чтобы получилось меньше отходов?
3. Прямоугольный параллелепипед, длина которого 4 см, ширина 3 см, высота 2 см, покрасили со всех сторон и разрезали на кубические сантиметры. Сколько получилось кубических сантиметров, у которых покрашена одна грань, две грани, три грани?
4.Витя Верхоглядкин начертил квадрат и нашел его периметр и площадь. Получилось Р=20 см, S=36 см2. Верно ли он посчитал?
5.В одной старинной математической рукописи шутливо обсуждалась возможность асфальтирования дороги для муравья: длиной 100 км и шириной 1 мм. Сможете ли вы найти площадь этой дороги?
Домашнее задание:
v Разрежьте прямоугольник, длина которого 9 см, а ширина 4 см, на две равные части, из которых можно составить квадрат.
v Объем деревянного бруска 80 см3, ширина 4 см, высота 2 см. Длину этого бруска уменьшили на 3 см. Определить объем оставшейся части.
v Васе купили аквариум в форме куба, вмещающий 64 л воды (1 л = 1 дм3 ). Вася наполнил аквариум водой, не долив 5 см до верхнего края. Сколько литров воды он налил в аквариум?
«Логические задачи»
1.Милиционер обернулся на звук бьющегося стекла и увидел четырех подростков, убегающих от разбитой витрины. Через 5 минут они были в отделении милиции. Андрей заявил, что стекло разбил Виктор, Виктор же утверждал, что виноват Сергей. Сергей заверял, что Виктор лжет, а Юрий твердил, что это сделал не он. Из дальнейшего разговора выяснилось, что лишь один говорил правду. Кто разбил стекло?
2.В бутылке, стакане, кувшине и банке находятся молоко, лимонад, квас и вода. Известно, что вода и молоко не в бутылке, сосуд с лимонадом стоит между кувшином и сосудом с квасом, в банке - не лимонад и не вода. Стакан стоит около банки и сосуда с молоком. Куда налита каждая жидкость?
3.Игра «Хоп!». Играющие по очереди называют последовательные числа натурального ряда, но вместо чисел, делящихся на 3 и оканчивающихся на 3, должны говорить «хоп!». Тот, кто ошибся, выбывает из игры, а остальные продолжают играть.
4.Лёня, Женя и Миша имеют фамилии Орлов, Соколов и Ястребов. Какую фамилию имеет каждый мальчик, если Женя, Миша и Соколов члены математического кружка, а Миша и Ястребов занимаются музыкой.
Домашнее задание:
v Для Вани, Толи и Миши есть три пирога: с рисом, с капустой и яблоками. Миша не любит пирог с яблоками и не ест с капустой; Ваня не любит пирог с капустой. Кто что ест?
v Три подруги вышли в белом, зеленом и синем платьях. Их туфли были одного из тех же трех цветов. Известно, что только у Ани цвета платья и туфель совпадали. Ни платье, ни туфли Вали не были белыми. Наташа была в зеленых туфлях. Определите цвет платья и туфель каждой из подруг.
v Поспорили три мудреца - кто из них самый мудрый. Пришли они к четвертому мудрецу с просьбой их рассудить. Подумал четвертый мудрей и предложил им такое испытание: «У меня есть 5 колпаков - два белых и три черных. Мы зайдем в темную комнату, я надену на ваши головы по колпаку. Затем мы выйдем из этой комнаты, и, кто первый определит цвет своего колпака, тот самый мудрый из вас». Согласились мудрецы и сделали все, как договорились. Через некоторое время один из них воскликнул: «На мне черный колпак!». Как рассуждал самый мудрый из мудрецов?
«Обсуждение олимпиадных задач»
1.Я задумал число, отнял от него 16, умножил результат на 4, разделил на 7. От 144 отнял полученное частное. 288 разделил на полученную разность, прибавил 195, получил 198. Какое число я задумал?
2.Ученик написал на доске пример на умножение двузначных чисел. Затем он стер все цифры и заменил их буквами. Получилось равенство: AB CD=MLNKT
Докажите, что ученик ошибся.
3.Баба Яга поставила на дверь кодовый замок. На замке нужно расставить девять разных цифр (1,2,3,4,5,6,7,8,9) так, чтобы получившиеся равенства были верными.
: = - = 3 + = 1
4.Незнайка начертил три прямых линии и отметил на них 6 точек. Оказалось, что на каждой прямой он отметил 3 точки. Покажите, как он это сделал.
5.Запишите все отрезки, изображенные на рисунке. Сколько получилось всего отрезков?
6.Четыре мальчика выбирали водящего с помощью считалки. Тот, на кого падало последнее слово, выходил из круга, и счет повторялся заново. Считающий мальчик каждый круг начинал с себя и в результате стал водящим, причем счет каждый раз кончался перед ним. Какое наименьшее количество слов могло быть в считалке?
7.Имеются 4 чемодана и 4 ключа к ним. Но ключи перемешались. Сколько испытаний в худшем случае надо сделать, чтобы подобрать для каждого чемодана ключ?
Домашнее задание:
v Три охотника варили кашу. Один положил 2 кружки крупы, второй - 1 кружку, а у третьего крупы не было. Они съели всю кашу поровну. Третий охотник и говорит: «Спасибо за кашу! - и вот вам задача: Я даю вам 5 патронов. Как поделить эти патроны в соответствии с вашим вкладом в мою порцию каши?»
v В школе изучают три иностранных языка: английский, немецкий и французский. Каждый ученик в классе изучает 2 языка. Английский язык изучают 19 человек, немецкий - 8, французский - 11. Сколько учеников в классе?
v На доске 5 на 5 клеток расставьте фишки так, чтобы в каждой строке и в каждом столбце стояли ровно три фишки. В одной клетке может стоять только одна фишка.
«Задачи на части».
1. Два дня пионеры собирали лом, причем того, что собрано в первый день, равна того, что собрано во второй день; во второй день собрано на 690 кг больше, чем в первый. Сколько килограммов лома собрано в каждый из этих дней?
2. Когда велосипедист проехал пути, лопнула шина. На остальной путь пешком он затратил вдвое больше времени, чем на велосипедную езду. Во сколько раз велосипедист ехал быстрее, чем шел?
3. Игра « оттесни шашку ». В крайних клетках полоски 1 на 20 стоят белая и черная шашки. Двое, по очереди, передвигают свою шашку на одну или две клетки вперед или назад, если это возможно (перепрыгивать через шашку нельзя). Проигрывает тот, кто не может двинуть свою шашку. Кто побеждает при правильной игре - первый или второй?
4. В классе число отсутствующих учеников составляет часть от числа присутствующих. После того как из класса вышел один ученик, число отсутствующих стало равно числа присутствующих. Сколько учеников в классе?
5. Два крестьянина вышли из деревни в город. Когда прошли пути, они сели отдохнуть. «Сколько нам еще осталось идти?» - спросил один попутчик другого. «Нам осталось на 12 км больше, чем мы прошли»,- был ответ. Каково расстояние между городом и деревней?
Домашнее задание:
v Когда пассажир проехал половину всего пути, то лег спать и спал до тех пор, пока не осталось ехать половину того пути, что он проехал спящим. Какую часть пути он проехал спящим ?
v На собрании присутствуют около 80 школьников. Треть из них - девочки, половина которых учится в 6-м классе. Из присутствующих мальчиков не учатся в 6-м классе. Сколько учащихся 6-го класса присутствуют на собрании?
v Решив все сбережения поделить поровну между всеми своими сыновьями, помещик составил такое завещание: «Старший из моих сыновей должен получить 1000 руб. и часть остатка; следующий - 2000 руб. и нового остатка; третий сын - 3000 руб. и часть третьего остатка » и т.д. Определить число сыновей и размер завещанного сбережения.
«Десятичные дроби».
Соревнование: (выигрывает тот, кто набирает больше баллов).
Ø Что легче: 0,3 килограмма железа или 0,3 килограмма ваты?
Ø Восстановите координатный луч, т.е отложите на нем единичный отрезок.
Однажды учитель предложил Вите Верхоглядкину сравнить дроби 0,31 и 0,6. «Это очень просто, - начал Витя. - Целые части этих дробей равны. Сравним дробные части: 31 больше 6, значит, и 0,31 больше чем 0,6». Согласны ли вы с таким решением?
Ø Некоторое число удовлетворяет одновременно трем неравенствам. Найдите его:
2,11< <2,5;
2,4 < <2,72;
2,39< <2,42.
Ø В некоторой десятичной дроби все цифры одинаковы. Какое это число, если оно больше 2,21, но меньше 2,221?
Ø Найдите ошибку:
a) 3,27 3,3;
b) 2,99 3,0;
c) 12,34 12,3;
d) 0,75 0,7;
e) 8,18 8,2.
Ø Все числа: 2,5; 2,6; 2,7; 2,8; 2,9; 3,1; 3,2; 3,3; 3,4 обладают одной особенностью, связанной с округлением чисел. Какой?
Ø Витя задумал число. Сначала он округлил его до десятых, получилось 6,4. Потом он округлил задуманное число до целых, получилось 7. Не ошибся ли он?
После подсчета баллов объявляются победители первых трех призовых мест и им вручаются небольшие сувениры.
1. Три друга - Коля, Витя, и Миша - решили купить шайбу, которая стоит 1 рубль. У Коли и Вити было по 0,25 руб., а у Миши - 0,45 рублей. Будут ли они вечером играть в хоккей?
2. Вместо звездочек поставьте знаки «+» или «-» так, чтобы равенства были верными: а) 5,5 * 1,9 * 2,6 =1;
б) 7,9 * 3,4 * 4,2 = 7,1;
в) 6,1* 13,5 * 12,4 =5.
Домашнее задание:
v Вместо квадратиков запишите такие десятичные дроби, чтобы равенства оказались верными:
v Даны числа: 0,8; 1,6; 2,9; 3,7. Разность двух из них равна одному из оставшихся чисел. Запишите это равенство.
v В трех пакетах содержится 1,5 кг крупы, причем массы первого и второго пакетов составляют вместе 1,3 кг, а второго и третьего - 0,9 кг. Сколько крупы в каждом пакете?
Математический час по теме « Десятичные дроби».
Девиз: Знания имей отличные по теме: «Дроби десятичные».
Весь класс разбивается на две команды.
1.Соревнование «Думай и соображай».
Задачи предлагаются всему классу. Отвечает тот, кто первый поднял руку. За правильное решение - 5 баллов. Эти баллы выставляют в таблицу той команде, в которой состоит ученик, решивший задачу.
a) Между числами 5,2 и 5,3 поставьте число, большее 5,2 и меньшее 5,3.
b) Даны числа: 0,3 ; 7,7; 0,125. Поставьте между ними такие знаки, чтобы в результате выполнения указанных ими действий получилась 1.
c) Найдите устно значение выражения: (13- 2,46 : 3,54) (0,5- ).
d) Некоторое число удовлетворяет одновременно трем неравенствам. Найдите его:
e) 3,5 < < 4,1;
3,7< < 4,0;
3,6 < < 3,9.
2. Игра « Заполни клетку».
Две команды получают листочки, текст которых приведен ниже.
Правило заполнения клеток состоит в том, что ответ предыдущего действия становится в первую клетку следующего. Первый участник команды вычисляет первый пример и передает карточку следующему участнику команды и т.д. Выигрывает та команда, которая быстрее и правильно заполнит карточку.
(У первой команды ответ 20, а у второй - 3).
3. Игра «Сравни дроби».
На доске прикреплены две таблицы (по одной для каждой команды). В каждой клетке написана десятичная дробь. Дроби в таблицах одинаковы, но расположены по-разному.
0,3 2,06 5,4
1,48 0,08 0,29
5,39 2,1 1,5
0,08 1,48 1,48
1,5 1,5 0,3
5,4 2,06 2,1
Учащимся предлагают в течение одной минуты рассмотреть числа в таблице, мысленно располагая их в порядке возрастания. Затем учащиеся в командах выстраиваются друг за другом. По знаку ребята, стоящие в команде первыми, бегут одновременно к таблицам и указывают на них самое маленькое число. Каждый следующий игрок указывает большее число. Он выбегает тогда, когда предыдущий возвратится и встанет в конец строя.
Начисление баллов идет по двум критериям: кто быстрее?, кто без ошибок?.
Итог математического часа подводит учитель. Объявляется команда победителей. Вручаются призы. В качестве выигрыша могут быть чертежные инструменты, недорогие, но необходимые учащимся принадлежности, наконец, конфеты, яблоки и т.д.
Домашнее задание:
v Даны числа: 2,67; 3,75; 3,51; 2,43. Сумма двух из них равна сумме оставшихся. Запишите это равенство.
v Масса драгоценных камней измеряется в каратах, причем 1 карат равен 0,2 гр. Геолог нашел два алмаза. Первый - массой 51 карат, а второй - массой 10,1 гр. Какой алмаз ценнее?
v Задумайте две десятичные дроби. Из большего числа вычтите меньшее. Результат запишите. Теперь сложите задуманные числа. Результат запишите. Потом сложите полученные результаты и сумму разделите на 2. Получится одно из задуманных чисел. Объясните почему?
«Принцип Дирихле»
1.В магазин привезли 25 ящиков с яблоками трех сортов, причем в каждом ящике лежат яблоки какого-то одного сорта. Можно ли найти 9 ящиков с яблоками одного сорта?
2.Принесли 5 чемоданов и 5 ключей от этих чемоданов, но неизвестно, какой ключ от какого чемодана. Сколько проб придется сделать в самом худшем случае, чтобы подобрать к каждому чемодану свой ключ?
3.В ящике лежали вперемешку 6 белых и 10 голубых носков. Каково наименьшее число носков надо взять из ящика, не глядя в него, чтобы иметь не меньше одной пары носков одного цвета?
4.Семь грибников собрали 100 грибов, причем все грибники собрали разное число грибов. Докажите, что есть трое грибников, которые собрали не меньше 50 грибов.
5.Игра «минус на плюс». В строке написано несколько минусов. Двое, по очереди, переправляют один или два соседних минуса на плюс. Выигрывает тот, кто переправит последний минус. Кто выиграет при правильной игре?
Домашнее задание:
v В классе 30 человек. В диктанте Витя Малеев сделал 12 ошибок, а каждый из остальных - не больше. Докажите, что по крайней мере трое учеников сделали одинаковое количество(быть может, и ноль) ошибок.
v В шкафу лежат вперемешку 5 пар светлых ботинок и 5 пар темных ботинок одинаковых размера и фасона. Какое наименьшее количество ботинок надо взять наугад из шкафа, чтобы среди них была хоть одна пара (на правую и левую ноги) одинакового цвета?
v Сможет ли Петя разложить 44 монеты по 10 карманам так, чтобы количество монет в каждом кармане было бы различным?
«Арифметика Магницкого».
«Арифметика, сиречь наука числительная, с разных диалектов на славянский язык переведенная и во едино собрана …»
Л.Ф. Магницкий.
1. Доклад о Л.Ф. Магницком.
2. Некий человек нанял работника на год, обещав дать ему 12 руб. и кафтан, но тот, проработав 7 месяцев, восхотел уйти и просил достойной платы с кафтаном; хозяин дал ему по достоицу расчет 5 руб. и кафтан, и ведательно есть, коликой цены оный кафтан был.
3. Послан человек из Москвы в Вологду, и велено ему в хождении своем совершати на всякий день по 40 верст; потом другий человек и другий день послан в след его и велено ему идти на день 45 верст и ведательно есть, коликий день постигнет второй первого?
4. Некто купил 96 гусей. Половину гусей он купил, заплатив по 2 алтына и 7 полушек за каждого гуся. За каждого из остальных гусей он заплатил по 2 алтына без полушки. Сколько стоит покупка?
5. «Математические забавы».
Пронумеровать дни недели, начиная с понедельника по порядку с 1 до 7. Кто-нибудь задумал день. Нужно угадать, какой день он задумал. Угадывающий предлагает выполнить про себя следующие действия:
1. умножить номер задуманного числа на 2;
2. прибавить к произведению 5;
3. умножить сумму на 5;
4. приписать к произведению нуль и назвать результат.
От этого числа угадывающий отнимает 250 и получает номер задуманного дня недели.
Домашнее задание:
v Летели скворцы и встретились им деревья. Когда сели они по одному на дерево, то одному скворцу не хватило дерева, а когда на каждое дерево сели по два скворца, то одно дерево осталось не занятым. Сколько было скворцов и сколько было деревьев?
v Путешественник идет из одного города в другой 10 дней, а второй путешественник тот же путь проходит за 15 дней. Через сколько дней встретятся путешественники, если выйдут одновременно навстречу друг другу из этих городов?
v Один человек купил 112 баранов старых и молодых, заплатив за них 49 рублей и 20 алтын. За старого барана он платил по 15 алтын и по 4 полушки, а за молодого барана по 10 алтын. Сколько каких баранов было куплено?
(гривна - 10 копеек; алтын - 3 коп; полушка- коп.).
Комбинаторные задачи с квадратом.

Цель. Развивать сообразительность, логическое мышление.

Ход занятия

  1. Расставить 3 числа

  2. Расставить 9 чисел

  3. Расставить 4 буквы

  4. Расставить 16 офицеров

Расставить 16 чисел.

Домашнее задание. Записать четыре тремя пятерками.

Выпуск математической газеты «Занимательная математика»

«Викторина».
1. Пассажир такси ехал в село. По дороге он встретил 5 грузовиков и 3 автомашины. Сколько всего машин шло в село?
2. У мальчика столько же сестер, сколько и братьев, а у сестры его братьев в 2 раза больше, чем сестер. Сколько в этой семье братьев и сколько сестер?
3. Как провести прямую, пересекающую все три стороны треугольника?
4. Имеется ров шириной 2 метра. Как переправиться через этот ров с помощью двух досок длиной 2 метра?
5. Два поезда одновременно вышли навстречу друг другу, один со скоростью 60 км/ч, а другой со скоростью 50 км/ч. На каком расстоянии будут поезда друг от друга за час до встречи?
6. Стоят шесть стаканов, первые три из них с водой. Как сделать, чтобы пустой стакан и стакан с водой чередовались? Разрешается брать только один стакан.
7. «Загадка». Чем больше из неё берешь, тем больше она становиться. Что это?
8. Человек рассеянный лег спать в 7.00 вечера на улице Бассейной, предварительно заведя будильник на 8.00 с тем, чтобы встать утром. Сколько он часов спал, пока его не разбудил будильник?
9. Что больше - 48,4% от 28 или 28% от 48,4?
10. В коробке 15 шариков: красных, синих и зеленых. Красных шариков в 7 раз меньше, чем зеленых. Сколько в коробке синих шариков?
11. Из 8 колец одно несколько легче остальных. Найти это кольцо не более чем двумя взвешиваниями на чашечных весах.
12. В ящике 70 цветных карандашей: 20 красных, 20 синих, 15 зеленых, 12 желтых, 3 черных. Какое наименьшее число карандашей надо взять, не видя их, чтобы среди них было не меньше 10 карандашей одного цвета?
«Вечер веселых и смекалистых».
Участвуют две команды. Вечер начинается с приветствия команд.
Команда 1:
Этот турнир ждали мы.
По нему стосковались умы.
Дружно будем задачи решать-
Мы хотим математику знать.
Как же нам не веселиться?
Не смеяться, не шутить?
Ведь сегодня на турнире
Мы решили победить!
Команда 2:
Сегодняшний турнир мы выиграть хотим
И просто вам победу не дадим.
Придется попотеть и постараться.
За каждое очко мы будем драться.
Смекалку мы проявим и отвагу
И просим разгадать сию бумагу.
А если вдруг не повезет? -
Победа всех когда-нибудь найдет.
Команды обмениваются большими свитками из ватмана. Свитки разворачивают и показывают собравшимся. На них большими цветными буквами написаны ребусы. Листы вывешивают на доске. Каждая команда, собравшись в кружок, тихо разгадывает ребусы соперника.
Пока команды трудятся над ребусами, ведущий представляет жюри. Затем ведущий обращается к командам: «Для решения большинства задач недостаточно одних знаний. Необходима ещё и внимательность. С чего начинается решение задачи? Конечно, с условия. Но условие можно читать по-разному: прочтешь невнимательно - вот и утеряна главная ниточка. Проверим, умеют ли команды быстро улавливать условие задачи. Кто из вас быстрее решит задачу Корнея Ивановича Чуковского:
«Шел Кондрат в Ленинград,
А навстречу двенадцать ребят,
У каждого по три лукошка.
В каждом лукошке - кошка,
У каждой кошки - 12 котят,
У каждого котенка
В зубах по 4 мышонка.
И задумался старый Кондрат:
Сколько мышат и котят
Ребята несут в Ленинград?»
Условие кажется очень простым, и некоторые ребята торопятся сосчитать. Ведущий не перебивает их, но затем сообщает, что они плохо выслушали условие и поэтому попали в положение Кондрата:
«Глупый, глупый Кондрат!
Он один и шагал в Ленинград,
А ребята с лукошками,
С мышами и кошками
Шли навстречу ему -
В Кострому».
Разобравшись с Кондратом, ведущий снова обращается к ребусам. Представители команд докладывают о том, в чем они увидели ключ к разгадке. За расшифрованный ребус присуждается 1 балл.
Традиционная часть турнира - веселая рыбалка.
На столе - «озере» разбросаны в разных местах рыбки вырезанные из бумаги и красиво раскрашенные. К каждой рыбке скрепками прикреплена задача. Ребята должны удочкой (палочка и к ней прикреплена нить с магнитом) поймать рыбку. Но ловцы не видят рыбок: с их стороны протянута занавеска. Она загораживает стол от играющих, и они забрасывают свои удочки наугад, как настоящие рыбаки. Рыбки видны болельщикам, которые должны активно помогать своим командам. Если с помощью указаний болельщиков рыбка найдена и её удается подцепить магнитом за скрепки, то удержать не так легко. Требуется осторожность, как при обычной ловле. Наконец, счастливый рыбак держит в руках свою добычу. Ему присуждают 1 балл и дают время для обдумывания задачи. Когда и соперник вытащит рыбку, первый ловец читает вслух свою задачу, сообщает её решение. Если задача решена верно, то ученику присуждают еще 1 балл, если же нет, то он обращается за помощью к команде.
Приведем несколько заданий конкурса «Рыбалка»:
1. Из Москвы в Ленинград вышел поезд со скоростью 50 км/ч, а из Ленинграда в Москву вышел поезд со скоростью 60 км/ч. Какой из поездов будет дальше от Москвы в момент встречи?
2. Тройка лошадей пробежала 30 км. Какое расстояние пробежала каждая лошадь?
3. Какой знак надо поставить между двумя двойками, чтобы получить число больше двух, но меньше трех?
4. По стеблю растения, высота которого 1 метр, ползет улитка. Днем она поднимается на 4 дм, а ночью спускается на 2 дм. На какой день улитка будет на вершине?
5. Мальчик хочет 30 орехов разложить на три кучки, чтобы в каждой кучке было нечетное число орехов. Что вы ему посоветуете?
6. Книга в переплете стоит 1 рубль 20 коп. Сколько стоит книга, если она на 1 рубль дороже переплета?
7. Арбуз весит 2 кг и еще 2/3 арбуза. Какова масса всего арбуза?
Этот конкурс очень возбуждает и участников, и зрителей. Поэтому ведущий предлагает всем успокоиться и выслушать индусскую притчу, которую любил рассказывать один из создателей Московского художественного театра Константин Сергеевич Станиславский.
Магараджа выбирал себе министра. Он объявил, что возьмет того, кто пройдет по стене вокруг города с кувшином, доверху наполненным молоком, и не прольет ни капли. Многие ходили, но по пути их отвлекали, и они проливали молоко. Но вот пошел один. Вокруг него кричали, стреляли. Его всячески пугали и отвлекали. Но он не пролил молоко. «Ты слышал крики, выстрелы? - спросил его потом магараджа. - Ты видел, как тебя пугали?» - «Нет, повелитель, я смотрел на молоко».
«Не слышать и не видеть ничего постороннего - вот до какой степени может быть сосредоточено внимание. Каким мощным оно бывает. Теперь мы проверим внимание членов команд»
От каждой команды на очередной конкурс выходят по одному человеку. Начинаются игры на проверку внимания.
Слушай одновременно нескольких.
Двое говорят одновременно два различных слова, а представители команд должны различить, кто какие слова сказал. Затем трое говорят одновременно три разных слова, следом четверо - четыре слова и т.д. Выигрывает тот, кто различил больше слов.
Каждой руке - свое дело.
Играющим дают лист бумаги и в каждую руку по карандашу. Задание: левой рукой начертить 3 треугольника, а правой 3 окружности.
Шагай - соображай.
Участники этого конкурса стоят рядом с ведущим. Все делают первые шаги, и в это время ведущий называет какое-нибудь число, например 7. При следующих шагах ребята должны называть числа, кратные 7: 14, 21, 28 и т.д. На каждый шаг по числу. Ведущий идет с ними в ногу, не давая замедлить шаг. Как только кто-то ошибся, он остается на месте до конца движения другого.
По результатам всех трех игр жюри определяет команду, победившую в конкурсе «Внимание», и указывает, сколько баллов заработала каждая команда (от 1 до 3 баллов).
С большим интересом ребята ждут конкурса капитанов. И вот, наконец, они предстают в единоборстве.
Вопросы капитанам (на размышление - полминуты).
1. В воде оказалась 10-я ступенька пароходной веревочной лестницы. Начался прилив: вода в час поднимается на 30 см. Между ступеньками лестницы 15 см. Через сколько часов вода скроет 6-ю ступеньку?
2. Два в квадрате - 4, три в квадрате - 9. Чему равен угол в квадрате?
3. В семье у каждого из 6 братьев есть по сестре. Сколько детей в этой семье?
Последнее конкурсное соревнование вечера - для болельщиков. Кто быстрее сосчитает?
1. 44 : 4=
44,044 : 44=
4, 444 : 44=
44,4444 : 444=
2. 55 :5=
5,555 : 0,55 =
55,055 : 5,5 =
555,555 : 55 =
Специально не оговаривается, какие баллы жюри может присуждать капитанам или болельщикам. Договариваться об оценках нужно в начале вечера, а в конце его жюри может согласовывать оценки по ходу работы.
В дополнительное время происходят внеконкурсные состязания. Одно из них - «Аукцион пословиц и поговорок с числами». Например: «Одна голова хорошо, а две - лучше»; «Одна рука узла не вяжет; у семи нянек дитя без глазу; хвастуну цена - три копейки» и т.д.
Заканчивается вечер выступлением представителя жюри, который называет победителей и поздравляет их. Можно вручить небольшие сувениры.
Занятие № 20

Обобщающие занятие.

Цель. Повторить и обобщить изученное.

Ход занятия.

I Повторение.

Задачи-шутки, задачи на переливание.

Упражнения со спичками, с куском бумаги.

Комбинаторные задачи.

II Подведение итогов.

Числовые головоломки

Цель. Развивать нестандартное мышление

1. Числовые головоломки.

  1. Задачи «Раздел лепешки» №50,

Дополнительно.

Решите уравнение:

Юный математик дополнительные занятия в 5 классе(А.Г. Мерзляк)

Занятие № 22

Арифметическая викторина

Цель. Развивать сообразительность, фантазию.

  1. На какое число нужно разделить 2, чтобы получить 4?

  2. Когда делимое и частное равны между собой?

  3. Может ли сумма трех последовательных натуральных чисел быть простым числом? двух? четырех?

  4. Существует ли простое число, являющиеся четным?

  5. Как с помощью одного знака неравенства можно записать, что число а больше -2, но меньше 2.

  6. Сколько га в 1 м2?

  7. За книгу заплатили 60 коп. и еще Юный математик дополнительные занятия в 5 классестоимости ее. Сколько стоила эта книга?

  8. Половина от половины числа равна половине. Какое это число?

  9. наполненные довержу водой сосуд имеет массу 5 кг, а заполненный наполовину 3 кг 250 г. Сколько воды вмещает сосуд?

  10. Сколько будет трижды сорок и пять?

Комбинаторные задачи

Цель. Развивать память, логическое мышление

1. Верные и неверные высказывания

307, стр. 56 (Ф.Ф.Нагибин)

2. Веселые вопросы.

Когда нельзя сокращать сократимую обыкновенную дробь?

Три курицы за три дня снесут три яйца. Сколько снесут 6 куриц за 6 дней? 4 курицы за 9 дней?

Юный математик дополнительные занятия в 5 классечисла равняется Юный математик дополнительные занятия в 5 классе его. Какое это число?

Половина - треть числа. Какое это число?

Дополнительно.

Решите:

Юный математик дополнительные занятия в 5 классе

Геометрическая викторина

Цель. Развивать сообразительность, логическое мышление, пространственное воображение.

  1. Прямая MN лежит внутри угла АВС, который больше нулевого угла, но не больше полного. Какай это угол?

  2. Можно ли из проволоки, длина которой 20 см, согнуть такой треугольник, одна сторона которого была бы равна 8 см? 10 см? 12 см?

  3. Одна сторона равнобедренного треугольника равна 20 см, а другая равна Юный математик дополнительные занятия в 5 классе третье. Чему равен периметр этого треугольника?

  4. Все высоты данного треугольника пересекаются в одной из его вершин. Какой это треугольник?

  5. Имеются 13 равных квадратов. Как составить из них два квадрата?

  6. Листочек бумаги надо разрезать на 8 частей, ограниченных отрезками. Сколько разрезов нужно для этого сделать?

Выпуск газеты «Занимательная математика»

Цель. Разработать и выпустить математическую газету.

Выпуск математической газеты.


© 2010-2022